Download as pdf or txt
Download as pdf or txt
You are on page 1of 89

2017-18 100 &

op kers
Class 12 T
By E ran culty
-JE Fa r
IIT enior emie .
S fP r es
o titut
Ins

PHYSICS
FOR JEE MAIN & ADVANCED
SECOND
EDITION

Exhaustive Theory
(Now Revised)

Formula Sheet
9000+ Problems
based on latest JEE pattern

2500 + 1000 (New) Problems


of previous 35 years of
AIEEE (JEE Main) and IIT-JEE (JEE Adv)

5000+Illustrations and Solved Examples


Detailed Solutions
of all problems available

Topic Covered Plancess Concepts


Tips & Tricks, Facts, Notes, Misconceptions,
Modern Physics Key Take Aways, Problem Solving Tactics

PlancEssential
Questions recommended for revision
24. MODERN PHYSICS

1. INTRODUCTION
The developments in the post-classical/Newtonian physics, also known as modern physics, has given us revelatory
insights into the structure and nature of fundamental forces/particles in the universe. The wave-particle duality/
paradox, which postulates that every elementary particle exhibits the properties of not only particles, but also
waves is one such insight. For example, when electromagnetic radiation is absorbed by matter, it predominantly
displays particle-like properties. It was de-Broglie who propounded the concept of matter waves, i.e. the particles
exhibiting wave properties. We will be dealing here with the energy, wavelength, and frequency of electromagnetic
waves and the relationship between these quantities. We will also be dealing with the photoelectric effect on which
Einstein’s based his photon theory of light. We will be discussing the Bohr atomic model, the hydrogen spectra, and
the laws describing the characteristics of X-rays.

2. DUAL NATURE OF ELECTROMAGNETIC WAVES


Classical physics always defined motions in terms of particles or waves i.e., it treated particle and waves as distinct
entities. An electron is considered as a particle because it possess mass (9.109×10−31 kilograms), electric charge
(−1.602×10−19 coulomb) and they behave according to the laws of particle mechanics. However, we shall see that
an electron in motion is as much a particle as it is a wave manifestation.
Electromagnetic radiation has properties in common with other forms of waves such as reflection, refraction,
diffraction, and interference. It, however, also has particle-like properties in addition to those associated with wave
motion (Photoelectric effect and Einstein’s theory, black body radiation, Compton effect). Therefore, we can say
that they a have dual nature.
hc
Einstein’s Equation: =
E hf
=
λ

2.1 Electromagnetic Spectrum

n
28 26 24 22 20 18 16 14 12 10 8 6 4 f(10 Hz)
TV & FM

AM Long
Visible

UHF

r Rays X-Rays UV IR Microwaves Short radio


Wave Waves

n
-20 -19 -16 -14 -12 -10 -8 -6 -4 -2 0 2 4 (10 m)

Figure 24.1 Electromagnetic spectrum


2 4 . 2 | Modern Physics

2.2 Electron Emission


Electrons are negatively charged particles. Therefore, though they move about arbitrarily in a conductor at room
temperature, they cannot leave the surface of the conductor due to attraction of positively charged particles(protons).
Therefore, some external energy has to be provided so that the electrons can be ejected from the atoms on the
surface of the conductor. To eject the electrons which are just on the surface of the conductor, only minimal energy
is required. This minimal energy or thermodynamic work that is needed to remove an electron from a conductor/
solid body to a point in the vacuum immediately outside the surface of the solid body/conductor is called the work
function (denoted by W) of the conductor. Work function is the property of the metallic surface.
Heat, light, electric energy etc., can be employed to liberate an electron from a metal surface. Depending on the
source of energy, the following methods are possible:
(a) Thermionic emission: In this method, the metal/conductor is heated to overcome binding potential of the
conductor, and consequentially, free the electrons.
(b) Field emission: The emission of electrons induced by an electrostatic field is called field emission. In this
process, the high electric field acting on the conductor exerts an electric force on the free electrons in the
conductor in the opposite direction of field. This force overcomes the binding potential of the conductor and
the electrons start coming out of the metal’s surface
(c) Secondary emission: Ejection of electrons from a solid that is bombarded by a beam of charged particles
(e.g., electrons or ions) is known as secondary emission.
(d) Photoelectric emission: The photoelectric effect refers to the emission/ejection of electrons from the surface
of a metal in response to incident light (or electromagnetic wave). This happens when the incident light or
electromagnetic wave has greater energy then the work function of the metal. The electrons emitted in this
process are called photoelectrons.

3. PHOTOELECTRIC EFFECT
(a) The photoelectric effect was discovered by Wilhelm Ludwig Franz Hallwachs in 1888, the experimental
verification which was done by Hertz.
(b) The photoelectric effect refers to the emission/ejection of electrons from the surface of a metal in response
to incident light (or electromagnetic wave).
(c) The electron ejected due to photoelectric effect is is called a photoelectron and is denoted by e− .
(d) Current produced as a result of the ejected electrons is called photoelectric current.
(e) Photoelectric effect proves quantum nature of light.
(f) Photoelectric effect can not be explained by the classical wave theory of light. The wave theory is incable of
explaining the first 3 obserations of the photoelectric effect.
(g) Photoelectrons, generally, refer to the free electrons that are in the inter-molecular spaces in the metal.
(h) Explanation for Photoelectric effect was successfulyy explained given by Albert Einstein as being the result
of light energy being carried in discrete quantized packets. For this excellent work he was honored with the
Nobel prize in 1921.
(i) The law of conservation of energy forms the basis for photoelectric effect.

Threshold Frequency ( v 0 ): The minimum frequency of the incident light or radiation that will produce a
photoelectric effect i.e., ejection of photoelectrons from a metal surface is known as threshold frequency for that
metal. Its value, though constant for a specific metal, may be different for different metals.
If v = frequency of incident photon & v 0 = Threshold Frequency
Then
(a) If v < v 0 , there will be no ejection of photoelectron and, therefore, no photoelectric effect.
P hysi cs | 24.3

(b) If v = v 0 , photoelectrons are just ejected from metal surface, in this case the kinetic energy of electron is zero.
(c) If v > v 0 , then photoelectrons will come out of the surface along with kinetic energy.

Threshold Wavelength ( λ0 ): The greatest wavelength of the incident light or radiation for a specified surface for
c
the emission of photoelectrons is known as threshold wavelength λ0 = .For wavelengths above this threshold,
v0
there will be no photoelectron emission.
For λ = wavelength of incident photon, then
(a) If λ < λ0 , then photoelectric effect will take place and ejected electron will posses kinetic energy.
(b) If λ = λ0 , then just photoelectric effect will take place and kinetic energy of ejected photoelectron will be zero.
(c) If λ > λ0 , there will be no photoelectric effect.

3.1 Work Function or Threshold Energy (f)


(a) The minimal energy or thermodynamic work that is needed to remove an electron from a conductor/solid
body to a point in the vacuum immediately outside the surface of the solid body/conductor is called the work
function or threshold energy for the conductor.
hc
= φ hv= 0
λ0
(b) Work function is the characteristic of given metal
(c) If E = energy of incident photon, then
(i) If E < φ , no photoelectric effect will take place.
(ii) If E = φ , just photoelectric effect will take place but the kinetic energy of ejected photoelectron will be
zero.
(iii) If E > φ , photoelectric effect will take place along with possession of the kinetic energy by ejected
electron.

3.2 Laws of Photoelectric Effect


Lenard postulated the following laws regarding photo emission on the basis of his experiments:
(a) For a given substance, there is a minimum value of frequency of incident light called threshold frequency
below which no photoelectric emission is possible, howsoever, the intensity of incident light may be. It is given
c
by v 0 =
λ0
(b) The number of photoelectrons emitted per second (i.e. photoelectric current) is directly proportional to the
intensity of incident light provided the frequency is above the threshold frequency.
(c) The maximum kinetic energy of the photoelectrons is directly proportional to the frequency provided the
frequency is above the threshold frequency. However, the relationship between the wavelength and kinetic
energy is inversely proportional. With increasing frequency of incident light, the kinetic energy of photoelectron
increases but with increasing wavelength it decreases. So v ↑ λ ↓ K.E. of emitted electrons ↑ v ↓ λ ↑ K.E. of
emitted electrons ↓
(d) The maximum kinetic energy of the photoelectrons is independent of the intensity of the incident light.
(e) The process of photoelectric emission is instantaneous, i.e., as soon as the photon of suitable frequency falls
on the substance, it emits photoelectrons.
(f) The photoelectric emission is one-to-one. i.e. for every photon of suitable frequency one electron is emitted.
(g) Value of threshold frequency or threshold wavelength depends upon photo sensitive nature of metal.
2 4 . 4 | Modern Physics

−19
Illustration 1: The work function of silver is 5.26 × 10 J . Calculate its threshold wavelength. (JEE MAIN)
hc
Sol: For any metal to eject photo electron the work function of surface is given as φ =
λ0
hc 6.6 × 10−34 × 3 × 108 °
Threshold wavelength = λ0 = ; ∴ λ = = 3.764 × 10−7 m ; λ =3764 A
φ 5.26 × 10 −19

Illustration 2: The work function of Na is 2.3 eV. What is the maximum wavelength of light that will cause photo
electrons to be emitted from sodium? (JEE MAIN)
hc
Sol: For any metal to eject photo electron the work function of surface is given as φ =
λ0
hc hc
The threshold wavelength λ0 = ; (=
φ hv=
0 ) ;& hc = 1.24 × 10−6 (eV) m
φ λ0
1.24 × 10−6 °
λ0 = m ; λ0 = 0.539 × 10−6 m = 539 nm; λo =5930 A
2.3

3.3 Failure of Wave Theory to Explain Photoelectric Effect


Note - The assumptions of the classical wave theory could not explain some observations of the photoelectric
effect. These aspects of the photoelectric effect were later explained by Albert Einstein’s photon theory. The failures
of the classical wave theory in explaining the photoelectric effect are enumerated below:
(a) The wave theory suggests that the intensity of the radiation should have a proportional relationship with
the resulting maximum kinetic energy. However Kmax = eV0 suggests that it is independent of the intensity of
light.
(b) According to the wave theory, the photoelectric effect should occur for any intense light, regardless of
frequency or wavelength. However, the equation suggests that photo emission is possible only when frequency
of incident light is either greater than or equal to the threshold frequency f0 .
(c) The wave theory states that there should be a delay on the order of seconds between the radiation’s contact
with the metal and the initial release of photoelectrons. It was assumed that between the impinging of the
light on the surface and the ejection of the photoelectrons, the electron should be “soaking up” energy from
the beam until it had accumulated enough energy to escape. However, no detectable time lag has ever been
measured.
In reality, due to collision between atoms inside the metal, some energy is lost. Hence kinetic energy emitted
by electrons is K< Kmax. Hence the term Kmax is used for the actual total kinetic energy.

3.4 Einstein’s Photon Theory


Albert Einstein worked his way around the limitations of the classical wave theory by explaining that lights exists
and travels as tiny packets/bundles called photons. The energy E of a single photon is given by E = hf
Applying the photon concept to the photoelectric effect, Einstein wrote:

hf
= W + Kmax (Already discussed)

Discussed below is how Einstein’s photon hypothesis overcomes the three objections raised against the wave
theory interpretation of the photoelectric effect.
Objection 1: Intensity of the radiation should have a proportional relationship with the resulting maximum kinetic
energy. This objection is overcome by Einstein’s photon theory because, doubling the light intensity merely doubles
the number of photons, thereby doubling the photoelectric current. It does not, however, change the energy of the
individual photons.
P hysi cs | 24.5

Objection 2: Photoelectric effect should occur for any intense light, regardless of frequency or wavelength. The
existence of a minimum frequency level ( in Einstein’s photon theory) follows from equation hf = W + Kmax . If
Kmax equals zero, then hf0 = W , which implies that the photon’s energy will be barely adequate to eject the
photoelectron and that there will be no residual energy to manifest as kinetic energy. The quantity W is the work
function of the metal/substance. If the frequency f is reduced below f0 , the individual photons, irrespective of how
numerous they are(in other words, no matter what the intensity of the incident light/radiation is), will not have
enough energy to eject photoelectrons.
Objection 3: There should be a delay on the order of seconds between the radiation’s contact with the metal
and the initial release of photoelectrons. The absence of a time lag follows from the photon theory because the
required energy is supplied in packets/bundles. Unlike in the wave theory, the energy is not spread uniformly over
a large area.
Therefore, as far as photoelectricity goes, the photon/particle theory seems to be in total contradiction of the wave
theory of light. Modern physicists have reconciled this apparent paradox by postulating the dual nature of light, i.e.,
light behaves as a wave under some circumstances and like a particle, or photon, under others.

3.5 Einstein’s Equation of Photoelectric Effect


Einstein (1905) explained photoelectric effect on the basis of quantum theory.
According to Einstein, when photons fallon a metal surface, they transfer their energy to the electrons of metal.
When the energy of photon is larger than the minimum energy required by the electrons to leave the metal surface,
the emission of electrons take place instantaneously.
He proposed that after absorbing the photon, an electron either leaves the surface or dissipates its energy within
the metal in such a short interval that it has almost no chance to absorb second photon
The energy supplied to the electrons is used in two ways:
(a) Removes the electron from the surface of metal
(b) Supplies some part of kinetic energy to the photoelectron. Therefore, Einstein’s equation of photoelectric
effect can be written as:
If v max is the maximum velocity of emitted electrons then by law of conservation of energy
1 1
hv = φ + mv 2 . If v 0 : Threshold frequency ∴ φ0 = 2
hv 0 , So ⇒ hv = hv 0 + mv max .
2 2

Einstein’s equation explains the following concepts


(a) The frequency of the radiation/incident light is directly proportional to the kinetic energy of the electrons
and the wavelength of radiation/incident light is inversely proportional to the kinetic energy of the electrons.
1 2
If v 0 is threshold frequency then maximum kinetic energy Emax = hv − hv 0 ⇒ mv max
= h(v − v 0 )
2
2h(v − v 0 )
So maximum velocity of photoelectrons: ⇒ v max =
m
m - mass of electron; v - frequency of incident light; v 0 - threshold frequency;
1 1  1 2 1 1 
λ0 - threshold wavelength Emax hc  −
λ - incident wavelength ⇒ =  ⇒ mv
= hc  −
λ λ  2 max  λ λ 
 0   0 
(b) If v = v 0 or λ = λ0 then v = 0
v < v 0 or λ > λ0 ⇒ There will be no emission of photoelectrons.
(c)
(d) Intensity of the radiation or incident light refers to the number of photons in the light beam. More intensity
means more number of photons and vice-versa. Intensity has no bearing on the energy of photons. Therefore,
intensity of the radiation is increased, the rate of emission increases but there will be no change in kinetic
energy of electrons. With increasing number of emitted electrons, value of photoelectric current increases.
2 4 . 6 | Modern Physics

Illustration 3: A light beam of wavelength 4000 Å is directed on a metal whose work function is 2 eV. Calculate the
maximum possible kinetic energy of the photoelectrons.  (JEE MAIN)

Sol: According to photoelectric equation the maximum kinetic energy of photoelectron after being ejected from
metal is EK = hν − φ
hc 19.8 × 10−19
Energy of the incident photon = . Energy of the incident photon in eV = = 3.09 eV ;
λ 4 × 1.6 × 10−19
Kinetic energy of the emitted electron EK = hν − φ = 3.09 – 2.00 = 1.09 eV

Illustration 4: Calculate the maximum kinetic energy of photoelectrons emitted from a metal with a threshold
wavelength of 5800 Å, if the wavelength of the incident light is 4500 Å. (JEE ADVANCED)

Sol: The maximum kinetic energy of photoelectron with which it is ejected from metal is EK = hν − φ .
2EK
max
Therefore maximum velocity of photoelectron is v max =
me

hc[λ0 − λ] [5800 × 10−10 − 4500 × 10−10 ]


Ek = = 6.62 × 10−34 × 3 × 108 = 9.9 × 10−20 J
max λ0 λ 5800 × 4500 × 10−20

9.9 × 10−20 2hc(λ0 − λ ) 2 × 0.62 × 1.6 × 10−19


Ek = = 0.62 eV ; ⇒ v max = = = 4.67 × 105 m / s
max
1.6 × 10−19 me λλ0 9.31 × 10−31

3.6 Photoelectric Current


(a) When light/radiation is directed on a cathode, photoelectrons are emitted and these are attracted by an
anode. The electric current, thus generated, flows in the circuit. This is called a photoelectric current.
(b) Value of photoelectric current depends upon following parameters:
(i) Potential difference between electrodes. (ii) Intensity of incident light.

3.6.1 Intensity of Light (I)


(a) It is quantity of light energy falling normally on a uniform surface area in unit time.
E W hc
or I = where I = Intensity of light in E = total energy incident = nhv = n
A.t m2 λ
n = no. of photons; A = cross sectional area; T = time of exposure
(b) Intensity of light is proportional to saturation current
I
(c) For point source of light Ir ∝
r2
I
(d) For the Linear source of light Ir ∝
r
Where r is the distance of the point from the light source.

3.7 Stopping Potential and Maximum Kinetic Energy


When the frequency f of the light/radiation is greater than the threshold frequency of the metal on which the light
is directed, some photoelectrons are emitted from the metal with substantial initial speeds. Let us assume that E
is the energy of light incident on a metal surface and W (<E) the work function of metal. In this case, as minimum
energy is required to extract electrons from the surface, the emitted electrons will have the maximum kinetic
energy which is E – W. Kmax = E − W
P hysi cs | 24.7

As the potential V is increased, the electrons experience greater resistance/repulsion, and consequentially, less
number of electrons reach the plate Q. This leads to a decrease in the flow of current in the circuit. At a certain value
V0 , the electrons having maximum kinetic energy (Kmax) also stop flowing and current in the circuit becomes zero.
This is called the stopping potential.
(a) In photoelectric cell, when (+)ve voltage is applied on cathode and negative voltage is applied on anode
applied, then the magnitude of photoelectric current decreases as the potential difference between the two
points (cathode and anode) increases.
(b) The stopping potential is the negative potential ( V0 ) applied to the anode Light
where the current gets reduced to zero or stops flowing in the circuit.
(c) When the magnitude of negative potential on anode is greater than
or equal to magnitude of stopping potential the current in the circuit
becomes zero. i

(d) If emitted electrons do not reach from cathode to anode then stopping P Q G
potential is given by
1 2 h(v − v 0 )
eV0 = mv max or Emax = eV0 ; eV
=0 h(v − v 0 ) ; V0 = V
2 e
(e) Value of stopping potential depends upon frequency of incident light. Figure 24.2: Photoelectric effect

(f) Stopping potential also depends upon nature of metal (or work function)
(g) Stopping potential does not depend upon intensity of light
1 2
(h) Example: Suppose stopping potential = -3 V, then mv max = 3eV
2
1 2
If we apply – 5 V, then also there will be zero current in the circuit but mv max ≠ 5eV
2
Because stopping potential is not equal to 5V which cannot be used in Einstein’s equation.

3.7.1 Graphs
(a) Kinetic energy V/s frequency: At v = v 0 , Emax = 0
E max

V0 V
Figure 24.3

(b)   Vmax V/s v : At v = v 0 Vmax = 0


V max

V0 V
Figure 24.4

(c) Saturated Current V/s Intensity:


Intensity

Current

Figure 24.5
2 4 . 8 | Modern Physics

(d) Stopping potential V/s frequency:

 eV=
0 hv − hv 0
V0
tanθ = slope
h V
= (constant for all type of metals)
e Figure 24.6
Intercept on x-axis = v 0
Intercept on y-axis = v
(e) Potential V/s current: (v : constant) Intensity
I3
Current I2 I3  I2  I1
I1

-V₀ Voltage
Figure 24.7

⇒ Stopping potential does not depend upon intensity of light.


(f) Photoelectric current V/s Retarding potential:

Current

v1 v3  v2  v1
v2
v3
vo(3)vo(2)vo(1)
Retarding potential
Figure 24.8

Illustration 5: Calculate the value of the stopping potential if one photon has 25 eV energy and the work function
of material is 7 eV. (JEE MAIN)
E − φ0
Sol: The stopping potential required to stop the photoelectrons to reach cathode is V0 =
e
E − φ0 25 − 7 18 eV
Stopping potential is V=
0 = = ⇒ V=
0 18 V
e e e

3.8 Derivation of de-Broglie Wavelength


h
De broglie equation is given by: λ =
p

Derivation: Let us start with the energy of a photon in terms of its frequency v, E = hv
Albert Einstein’s special theory of relativity gives a new expression with reference to the velocity of light. This
expression is E = mc2 , where m refers to the relativistic mass of light which is non-zero as it is travelling with
velocity c. If it were at rest, it’s mass would be zero.
c
Now, by equating both the energy equations we get E = hν = mc2 . Also, as seen earlier ν =
λ
h h
Wavelength of a photon. ∴ = mc and λ =
λ mc
P hysi cs | 24.9

Analogously, de Broglie argued that a particle with non-zero rest mass m and velocity v would have a wavelength
h
given λ =
mv
h
Also, mv = p, where p is the particle’s momentum. Substituting p for mv we get λ =
p

3.8.1 Criterion for Type of Behaviour


Like electromagnetic waves, moving bodies also exhibit the wave-particle duality and the wave and particle aspects
of moving bodies cannot be observed simultaneously. A moving body will exhibt particle behavior if the wavelength
of the body is negligible in comparison to its dimensionwhereas it will exhibit a wave nature if its wavelength is in
order of the dimension of body.

Illustration 6: Determine Broglie wavelengths of (a) a 46g golf ball with a velocity of 30 m/s and (b) an electron
with a velocity of 107 m/s. (JEE ADVANCED)
h
Sol: The de-Broglie wavelength of the particle of mass m and moving with velocity v is given by λ = , where h
is Planck’s constant mv

(a) Since v<<c, we can let (effective mass = Rest mass) m =m0 .
h 6.62 × 10−34 J.s
Hence λ = = = 4.8 × 10−34 m.
mv (0.04kg)(30m / s)

Thus, we see that the wavelength of the golf ball is so negligible compared with its dimensions that we would
not be able to observe expect to find any wave aspects in its behavior.
(b) Again v << c, so with m = m0 = 9.1 × 10−31 kg,

h 6.62 × 10−34 J.s °


we have λ = = = 7.3 × 10−11 m = 0.73 A
mv 9.1 × 10 −31 × 107 (kg m / s)
The dimensions of atoms are comparable with the radius of the hydrogen atom which in reality is 5.93 × 10−11 m.
So, it it is clear that an electron with a wavelength of 7.3 × 10−11 m would demonstrate a wave behavior. Also, we
can see that the wave character of moving electrons is facilitates the understanding atomic structure and behavior.

4. ENERGY, MOMENTUM, AND WAVELENGTH OF PHOTONS


(a) The quantum theory states that the light photons are undivided energy packets.
(b) Energy of photons is denoted by E = hv, where h is Planck constant, v is the frequency of photons, and E is
the energy of photons.
(c) The velocity of photons and the velocity of light are equal (c). Therefore, c = vλ
hc
Here, λ is the wavelength of wave connected to photon. ∴ E = hv =
λ
(d) The mass of photons at rest is zero but it will be non-zero if the photons are moving.. Assuming m to be the
effective mass of photons, energy of photon according to Einstein:
hc
E = mc2 ⇒ E = hv = = mc2
λ
mc2 E 1  hc  h h
(e) Momentum of moving photon p = mc = = =   = ⇒ p=
c c c λ  λ λ
E hv h p
(f) Effective mass m = = = =
c 2
c 2 c λ c
2 4 . 1 0 | Modern Physics

h h hc
(g) Wavelength connected to moving photons λ= = =
p mc E
(h) From Point (e) and (f):- Momentum of photon p ∝ m p ∝ E Energy of photons E ∝ m
1 1 1
Wavelength of wave connected to photons λ ∝ ; λ∝ ; λ∝
p m E
(i) Graphs

(i) P P

m E

(ii)

E
P m

(iii)
1/E
1/p 1/m
Figure 24.9

( j) There is no charge on photons

m
Figure 24.10

Illustration 7: A Determine the velocity of a light wave, given that frequency of the photon is ν, energy is hv, and
h
momentum is p =  (JEE MAIN)
λ
c
Sol: For light of frequency ν, the energy is E = hν and the frequency of light wave is ν = . Hence speed of light
is easily determined. λ

h
As E = hν and P = ,
λ
hc E
E= = Pc ⇒ c =
λ P


Illustration 8: Determine the mass of a photon witha wavelength of 0.01 A . (JEE MAIN)
h
Sol: Using equation of equivalent mass of photon, m = , we can find the mass of proton.

E h 6.62 × 10−34
m= = = ; m = 2.21 × 10−30 kg
c2 cλ 8
3 × 10 × 10 −12
P hysi cs | 24.11

Illustration 9: Determine the momentum of a photon with a of frequency 109 Hz.  (JEE MAIN)

Sol: The momentum of photon is p =
c
h hν 6.62 × 10−34 × 109
p= = = ; p = 2.2 × 10−33 kg m/s
λ c 3 × 108

Illustration 10: Determine the energy and momentum of a γ-ray photon with a wavelength of 0.01 Å.(JEE MAIN)
hc h E
Sol: For wave of wavelength λ the energy and momentum is given by E = and p= =
λ λ c
hc 1240(eV) × 1 × 10−9
E= = ;
λ 0.01Å

1240 × 10−9 E MeV


E= (eV) = 1.24 × 106 eV ; E = 1.24 MeV ; The momentum is P = = 1.24
10 −2
× 10 −10 c c

1.24 × 106 × 1.6 × 10−19


P= = 6.62 × 10−22 kg m/s
3 × 108

5. ENERGY, MOMENTUM, AND WAVELENGTH OF A MOVING PARTICLE


Suppose the mass of a particle at rest is m and it is moving with velocity v.

(a) Mass at rest = m


m
(b) Effective mass (or relativistic mass) =
1 − v 2 / c2
 
(c) Momentum or p = m v or p = mv ∴=
p mv = 2mE Here E : Kinetic energy
1 p2
(d) Kinetic energy E = mv 2 =
2 2m
h h h
(e) If λ is the wavelength of connected wave to the moving particle, then λ= = =
p mv 2mE

Illustration 11: A body of 10 gm is moving with velocity 2 × 103 m/s. Determine the value of its associated de-
Broglie wavelength.  (JEE ADVANCED)
h
Sol: The de-Broglie wavelength associated with particle moving with speed v is calculated as λ = .
mv
h 6.62 × 10−34
de-Broglie wavelength =
λ = ;
mv 10 × 10−3 × 2 × 103
λ = 3.3 × 10−35 m
1 1 1 1
So λ ∝ ; λ∝ λ ∝ λ∝
p E v E

 

Figure 24.11 E v Figure 24.12
P 1/ E
2 4 . 1 2 | Modern Physics

Moving particle

Charged Uncharged
electrons
protons
-particles, etc.
Photons Neutrons,
Atoms etc.

5.1 Energy, Momentum, and Wavelength of Charged Particle Accelerated by V-volt


(a) Potential difference or electric field can be used to accelerate a charged.

(b) The kinetic energy of a charged particle having charge q, mass m, accelerated by V volt, and a velocity v is
1
denoted= by E = mv 2 qV
2
2qV 2E
(c) Velocity V = =
m m

(d) Momentum p = 2mE = 2mqV

h h
(e) Wavelength λ = =
p 2mqV
(Here it is assumed that initial potential given to electron is zero)
h
If the particle is given some initial potential Vi and if final potential is Vf then, λ=
2mq(Vf − Vi )
From above Relation

1
λ∝  
V

V
Figure 24.13

1
λ2 ∝  
V

V
Figure 24.14

1
λ∝  
V

V
Figure 24.15
P hysi cs | 24.13

Cases:

(a) If the moving charged particle is an electron, then

2eV h 12.27
(i) v e = (ii) pe = 2 meeV (iii) λe = = Å
me 2me eV V
(b) If the moving charged particle is a proton, then

2eV h 0.286
(i) vP = (ii) pP = 2mepV (iii) λP = = Å
mp 2mepV V
(c) If the charged particle is an α -particle, then

2(2e)V eV 1
(i) v α = = = vP (ii) Pα = 2mα eα V = 2 × 8mp × eV = 2 2 pp
mα mP 2
h 0.101
(iii)
= λα = Å
16mp eV V

Illustration 12: Determine the potential to be applied to accelerate an electron such that its de-Broglie wavelength
becomes 0.4 Å. (JEE MAIN)
12.27
Sol: The de-Broglie wavelength of an electron in terms of accelerating potential difference is λe = Å
V0
12.27 12.27
Where V is the applied potential on electron to accelerate it. λ = Å ; 0.4 =
V0 V0

(12.27)2 12.27 × 12.27


Squaring on both the sides we get 0.16 = ⇒ V0 = ; ⇒ V0 = 941.0 V
V0 16 × 10−2

5.2 Wavelength of Wave Connected to Uncharged Particle


(Like neutron atoms, molecules etc.)
1
(a) If m is the mass and v is the velocity of particle, then kinetic energy E = mv 2 , momentum p = mv
2
h h h
(b) Wavelength λ= = =
p mv 2mE
(c) If λ is the wavelength of wave connected to matter particle then particle energy E will be

h2 h2
E= J= eV
2mλ2 (2mλ2 )e
(d) Energy E of particle (e.g., electron, neutron, or atom) at equilibrium temperature TE = (3/2) KT

Here K = Boltzman constant


h h
(e)
=  λ = here m : mass of a single atom.
2mE 3mKT

Illustration 13: Determine the associated de-Broglie wavelength if the energy of a thermal neutron is 0.02 eV,
 (JEE MAIN)
2 4 . 1 4 | Modern Physics

h
Sol: For neutron having kinetic energy K, the associated de-Broglie wavelength is found to be λ =
2mK
h 6.6 × 10−34
de-Broglie Wavelength λ = = ; λ = 2 × 10−10 m = 2Å
2mK 2 × 1.6 × 10 −27
× 0.02 × 1.6 × 10 −19

6. EXPERIMENTAL VERIFICATION OF MATTER WAVES


The Davisson–Germer experiment conducted by American physicists Clinton Davisson and Lester Germer confirmed
the De Broglie hypothesis which says that the particles of matter such as electrons have wave-like properties (see
Fig. 24.16).

Detector
Electron
Gun

Incident beam
of electrons Diffracted

Crystal

Figure 24.16: Diffraction of matter waves

(a) Davisson–Germer’s experiment


(i) Experimental confirmation of De Broglie waves was done by scientist Davisson and Germer by firing
slow-moving electrons at a crystalline nickel target. The diffraction pattern of electron beam through the
nickel crystal was same as those predicted by Bragg for X-rays.
(ii) Since diffraction is the property of waves, the diffraction of electronic beam confirmed that a wave is
connected to moving electron beam.
(iii) The electron gun was used to obtain electrons with different energies. This was done by accelerating the
e− by V volt in the electron gun.
(iv) When these accelerated electrons fall on a crystal, they are diffracted in various directions.
(v) Electrons were collected by a detector of Faraday cup which was connected to an electrometer.
(vi) Electron beams with different energies produced different intensities of diffracted electrons.
(vii) Results of the Davisson–Germer’s experiment
•• Intensity at any angle is proportional to the distance of the curve at that angle from the point of
scattering.

500

54V

Figure 24.17
P hysi cs | 24.15

•• Intensity is maximum at 54 V potential difference and 50° diffraction angle.

I =50o

54
V
Figure 24.18

(viii) From Bragg’s Law :- D sin θ = nλ (constructive interference)

incident
d o
=90
D 2

Diffracted Wave

Figure 24.19

θ : Angle between incident ray and nth maxima.


n: Diffraction order
D: Distance between atoms or 2d sin φ = nλ
D: Distance between lattice planes
φ : Angle between diffraction plane and incident ray.

(ix) The critical value of wavelength of an accelerated electron at 54 V = 1.67Å
Experimental value = 1.65 Å
(x) Any wave or particle is diffracted by crystal plane only when wavelength is in order of distance between
lattice planes of an atom.

Illustration 14: In a Davisson-Germer experiment, a, electron beam of wavelength 1.5 Å is normally incident on a
crystal, having 3Å distance between atoms. Determine the angle at which first maximum occurs. (JEE MAIN)

Sol: According to Davison–Germer’s experiment, when electrons accelerating at some potential difference V are
incident on a crystal, they diffract. The angle at which the first maxima of diffraction pattern occurs can be found
by Bragg’s law i.e., D sin θ= nλ
nλ 1 × 1.5 1
D sinθ = nλ ∴ sinθ = = = , θ= 30°
D 3 2

7. ATOMIC MODELS
Model : A model is simply a testable idea or hypothesis based on logical and scientific facts.
Theory : A model becomes a theory when it is verified by rigorous scientific analysis and experiments. . Otherwise,
the model is simply not accepted.
2 4 . 1 6 | Modern Physics

7.1 Dalton’s Atomic Model


(a) All matter is made of tiny particles called atoms. Atoms are indivisible and indestructible.
(b) All atoms of a given element are identical in mass and properties, while atoms of different elements differ in
mass and properties.
(c) All matter is made up of hydrogen atoms. The mass and radius of heaviest atom is about 250X and 10X of the
than that of the hydrogen atom, respectively.
(d) Atoms are stable and electrically neutral.

Reason of Failure of model: The discovery of electron by J.J. Thomson (1897) proved that atoms are not indivisible.
Hence, the model is no longer valid.

7.2 Thomson’s Atomic Model (or Plum-Pudding Model)


In this model, the atom is composed of electrons (which Thomson
still called “corpuscles”) surrounded by a soup of positive charge to Electron
balance the electrons’ negative charges, like negatively charged
“raisins” surrounded by positively charged “pudding”. Uniformly distributed
positively charged matter
Achievements of model: Explained successfully the phenomenon
of thermionic emission, photoelectric emission, and ionization. Figure 24.20: Thomson’s Atomic Model

Failure of the model:


(a) It could not explain the line spectrum of H-atom.
(b) It could not explain the Rutherford’s α − particle scattering (Rutherford gold foil) experiment.

7.3 Rutherford’s Experiment and Atomic Model

α -Scattering Experiment:

Results of Experiment:
(a) It was seen that in the experiment that when the α - particles were fired at the gold foil, some of the particles
(<1 in 8000) bounced off the metal foil in all directions, some right back at the source. This should have
been impossible according to Thomson’s model; the alpha particles should have all gone straight through.
Obviously, those particles had encountered an electrostatic force far greater than Thomson’s model suggested
they would, which in turn implied that the atom’s positive charge was concentrated in a much tinier volume
than Thomson imagined. This was possible only in the case when there exists a solid positive mass confining
in a very narrow space.
(b) However, most of the α - particles just flew straight through the foil. This suggested that those tiny spheres
of intense positive charge were separated by vast gulfs of empty space.

Gold foil
Figure 24.21: Scattering of alpha particles by gold nucleus
P hysi cs | 24.17

1
(c) N ∝ ⇒ If θ ↑ then N ↓ , N = No. of particles scattered per unit time
4 θ
sin  
2

N Hyperbola

Figure 24.22

Equation indicates that at larger deflection (scattering) angle, number of particles deflected are very-very less.

Figure 24.23

Graph for N & θ show that coulomb’s law holds for atomic distances also.

(d) N ∝ (Nuclear charge)2

Illustration 15: In an α - particle scattering experiment using gold foil, the number of particles scattered at 60° is
1000 per minute. What will be the number of particles per minute scattered at 90° angle? (JEE ADVANCED)

Sol: In Rutherford’s experiment, the number of particles deflected at an angle θ by the gold atoms per minute are
1
best represented by relation N ∝
θ
sin4  
2
Let N = No. of α - particles scattered per minute at an angle 90°.
1
∴ N∝  (1) … (i)
 90 
sin4  
 2 
1
Given that 1000 ∝ (2) … (ii)
 60 
sin4  
 2    60 
sin4  
Taking ratio of (i) to (ii) we get N = 1000 ×  2  = 250 /min
 90 
sin4  
 2 
Rutherford’s Atomic Model
Uniform distribution of
mass & charge

Solid sphere

Electrons (Stationary & randomly located)

Figure 24.24: Thomson’s Atomic Model


2 4 . 1 8 | Modern Physics

Non uniform distribution of mass & charge


Hollow sphere
Electrons
(movable in
circular orbits)
Figure 24.25: Ruthorford’s Atomic Model

(a) The whole positive charge and almost whole mass of an atom (leaving aside the mass of revolving e− in
various circular orbits) remains concentrated in nucleus of radius of the order of 10−15 m.

e
r
Hollow
sphere
Circular orbit
Figure 24.26: Motion of electron in atom

(b)  Σq( + )ve on proton in a nucleus = Σq( −)ve on e− in various circular orbits & hence, the atom is electrically
neutral.
(c) The necessary centripetal force for revolving round the nucleus in circular orbit is provided by coulomb’s
mv 2 k(ze)(e)
electrostatic force of attraction =
r r2
Reason of failure of model
(a) It could not explain the line spectrum of H-atom.
Justification: Asper Maxwell’s electromagnetic theory, every accelerated moving charged particle emits
energy in the form of electromagnetic waves and, therefore, the frequency of an Modern Physics - Solution (1)
while moving in a circular orbit around the nucleas will steadily decline, resulting in the continuous emission
of lines thereby mandating that the spectrum of an atom be continuous, but in reality, one obtains line
spectrum for atoms.
(b) It could not explain the stability of atoms.
Justification : Since revolving electron continuously radiates energy, the radii of circular path will continuously
decrease and in a time of about 10−8 s the revolving electron must fall down in a nucleus by adopting a spiral
path.

Application of Rutherford’s model


Determintion of distance of closest approach: When a positively charged particle approaches a stationary
nucleus (which is the positively charged core of the atom), then due to repulsion between the two (like charges
repel), the kinetic energy of positively charged particle gradually decreases, reaching a stage where its kinetic
energy becomes zero and from where it again starts retracing its original path.
Definition: The distance of closest approach is the
minimum distance of a stationary nucleus from a point z1e z2e
where the kinetic energy of a positively charged particle
approaching the nucleas for a head-on collision becomes A B
zero. Suppose a positively charged particle A of charge q1 Stationary necleus
(=z1e) approaches from infinity towards a stationary z0
nucleus of charge z 2 e then,
Figure 24.27: Distance of closest approach
P hysi cs | 24.19

Let at point B, kinetic energy of particle A becomes zero then by the law of conservation of energy at point A & B.
k(z 1e)(z 2e) k(z1e)(z 2e)
TEA = TEB ; KEA + PEA = KEB + PEB ; E+0=0+ (in joule) ∴ r0 = m
r0 E

Illustration 16: Calculate the distance of closest approach where an α -particle with kinetic energy 10 MeV is
heading towards a stationary point-nucleus of atomic number 50. (JEE MAIN)

Sol: The nucleus of tin (atomic number 50) being more massive than the
alpha particle, remains stationary. So the kinetic energy of the alpha particle A B
is converted into electric potential energy at the distance of closest
approach. -particle
K × (Z1e) × (Z 2e) z0
The electric potential energy of alpha particle is TEα =
r0
1
where K = and r0 is the distance of closest approach of alpha particle Figure 24.28
4 πε0
from nucleus of tin.
K × (2e)(50e)
TEA = TEB ; ∴ 10 × 106 eV =
r0

−14 −4
r0 1.44 × 10 m =
= ; r0 1.44 × 10 Å

Illustration 17:Find the distance of closest approach for a proton moving with a speed of 7.45 × 105 m/s towards
a free proton originally at rest. (JEE MAIN)

Sol: As the moving proton approaches the free proton originally at rest, it exerts an electric force of repulsion on
the proton at rest. At the distance of closest approach, both the protons move with same velocity along the line
of impact. The initial kinetic energy of moving proton is equal to the final kinetic energy of both the protons plus
Ke2
the electric potential energy at the distance of closest approach, given by . Here r0 is the distance of closest
approach. r0

→ V = 7.45 × 105 m/s u = 0


O O
Proton Free proton

Originally

V1 V2

r0
Proton free proton
after movement
Figure 24.29

At the time of distance of closest approach


By the law conservation of energy
1 ke2 1 1
mv 2 + 0= + mv12 + mv12  ... (i)
2 r0 2 2
v
By the conservation of momentum mv + 0 = mv1 + mv1 ∴ v1 =
2
2
1 ke2 v 4 4 × (9 × 109 )(1.6 × 10−19 )2
From equation (i) mv 2 = + m  ; r0 = × ke2 = r0 1.0 × 10 −12 m
=
2 r0 2 mv 2 (1.66 × 10−27 )(7.45 × 105 )2
2 4 . 2 0 | Modern Physics

7.4 Bohr’s Model


Bohr combined the concepts of classical physics with quantum mechanics to propose his model for H or H-like
atoms. This model is based on law of conservation of angular momentum.
(a) According to de Broglie, in a stationary orbit the circumference of Bohr’s orbit must be an integral multiple of
the wavelength associated with the moving particle
or 2πr = nλ (Constructive interference)
nh nh
or 2πr = or mvr = which is Bohr’s quantum condition.
mv 2π
(b) In an orbit, waves are always formed in whole numbers..

7.4.1 Concept of Stable, Stationary, Quantized, Fixed Allowed Radii Orbit, or Maxwell’s
Licensed Orbits

According to Bohr, if an electron revolves in these orbits the electron neither radiates nor absorbs any energy.
v
Total energy, (E) (eV)

Unbound (ionised)
r + atom


0
-0.85 n=5
n=4
Figure 24.30: Bohr radius -1.51 n = 3 Excited
states

-3.40 n=2

Ground state
-13.6 n=1

Figure 24.31: Energy level diagram

(b) Emission of energy


Where n = principle quantum no.
En = energy of e− in nth orbit

En2 n2
hc
E = h = = En1 - En2

En1 n1

Figure 24.32: Emission of energy by electron


P hysi cs | 24.21

(c) Absorption of energy


En2 n2

E E + En1= En2

En1 n1

Figure 24.33: Absorption of energy by electron

h
Electron revolves only in those orbits in which its angular momentum is integer multiple of

h
mvr = Iω = n

mv 2 kZe2
=
r r2

7.4.2 Determination of Radius, Velocity & Energy of e- in Bohr’s Orbit


(a) Determination of radius of circular path (orbit)
nh
 mvr =  … (i)

nh
∴ v=  … (ii)
2πmr

mv 2 kZe2
2
kZe2  n2h2  n2 h2
and = ; ∴ m  nh  = ; rn = v   rn =
r r2  2πmr  r  4 π2mkZe2  ; Z
×
4 π2mke2
 
n2 
r=
n × 0.529 A
Z

Results:
(1)2
(i) r1
 = × 0.529 Å ; ∴ rn = n2r1
z r

n2
Figure 24.34

(ii) ∴ r ∝ n2
r Parabolic

n
Figure 24.35

Illustration 18: The radius of the shortest orbit of a single-electron system is 18 pm. This system can be represented
as (JEE MAIN)

Sol: According to Bohr’s model, the radius of orbit of electron is directly proportional to square of principle quantum
0.529 o
number i.e., rn ∝ n2 . When the electron is in ground state (i.e., for principle quantum number =1) r1 = A
Z
2 4 . 2 2 | Modern Physics

(1)2 
For shortest orbit n = 1; rn = n2r1 ; × 0.529 Å = 18 × 10-2 A
Z
3 system is Li2+ since only single e  is present.
⇒Z=

Illustration 19: What will be the ratio of the area of circular orbits in doubly ionized lithium atom in 2nd & 3rd Bohr
orbit? (JEE MAIN)

Sol: According to Bohr’s theory as rn ∝ n2 , but A ∝ r 2 . Therefore A ∝ n4 .


A2 (2)4 16
∴ = =
A3 (3) 4 81

7.4.3 Determination of Velocity of Electron in Circular Orbit


nh
∴ mvr =  ...(i)

nh mv 2 kZe2 2πkZe2 Z 2πke2 Z


r= ; ⇒ = ⇒ v= ; ⇒ v= × ⇒ v = 2.18 × 106 m/s
2πmv r r2 nh n h n

c Z
v= m/s; where c = velocity of light in vacuum = 3 × 108 m/s
137 n

Results:
1
(i) v ∝ (Z = constant)
n
v hyperbolic

n
Figure 24.36

Illustration 20: What will be the ratio of speed of electrons in hydrogen atom in its 3rd & 4th orbit? (JEE MAIN)
Z
Sol: According to the Bohr’s theory v ∝ where v is the speed of electron in its orbit, n is the principle quantum
n
number and Z is the atomic number of the element.

Z v 4
 v∝ ∴ 3 =
n v4 3

Illustration 21: What will be the the ratio of speed of electron in 3rd orbit of He+ to 4th orbit of Li++ atom?
 (JEE MAIN)
Z
Sol: According to the Bohr’s theory v ∝ , where v is the speed of electron in its orbit n is the principle quantum
n
number and Z is the atomic number of the element.

Here the element in consideration differs in atomic number, i.e., Z(He) = 2 and Z (Li) =3
2
( v3 ) He+ 3 8
∴ =  = .
( v 4 ) Li2+ 3 9
4
P hysi cs | 24.23

7.4.4 Determination of Energy of Electron in Bohr’s Circular Orbit


1 kZe2
(a) Kinetic energy of electron KE = mv 2 ; KE =
2 2r
Results: v
(i) KE of an e− = positive quantity
(ii)  r ↑ , KE ↓ + r
(iii) when, r = ∞ , KE = 0

K( + Ze)( −e) KZe2 Figure 24.37:


(b) Potential energy of an electron PE = ; PE = − Bohr’s Orbit
r r
Results:
(i) Potential energy (PE) of an e− = negative quantity (ii) r ↑ , PE ↑ (c) If r = ∞ , PE = 0

(c) Total energy of electron: The total energy of an electron in any orbit equals the sum of its kinetic and
KZ e2 KZ e2 KZ e2
potential energy in that orbit. TE = KE + PE = − ; TE = −
2r r 2r
Results:
(i) TE of an electron in atom = (-)ve quantity. (-)ve sign indicates that electron is in bound state.
(ii) If r ↑, TE ↑
(iii) if r = ∞ , TE = 0
PE
(iv) TE = -KE = in any H-like atom
2

Total energy of terms of n

k Ze2
TE = −
 n2h2 
2× 2 
 4 π mk Ze2 
 

2π2mk 2 Z 2e4 Z2 Z2
TE =
− ⇒ TE =
− R ch ⇒ TE =
−13.6 ev
n2h2 n2 n2
2π2mk 2e4 me4
where R = Rydberg constant = = = 1.097 × 107 m−1
3
ch 8 ∈0 ch3
2

Note: Rydberg constant is not a universal constant. In Bohr calculation, it is determined by assuming the nucleus
to be stationary
For Bohr Rydberg constant,
= R ∞ 1.097 × 107 m−1 , if nucleus is not assumed stationary then
R
R= ,mN = mass of nucleus
 me 
1 +  
 mN 

7.4.5 Results Based on Total Energy Equation


(a) With the increase in principal quantum number n (relative overall energy of each orbital), both total energy
and potential energy of an electron increases, whereas the kinetic energy decreases.
(b) With the increase in principal quantum number, the difference between any two consecutive energy level
2 4 . 2 4 | Modern Physics

decreases.
(c) Total energy of an electron in any orbit in H-like atom = (Total energy of an electron in that orbit in H-atom
×Z 2 )
(d) PE of an electron in any orbit in H-like atom = (PE of an electron in that orbit in H-atom) ×Z 2 (v) KE of an
electron in any orbit in H-like atom= (KE of an electron in that orbit in H-atom) ×Z 2 (vi) ∆En n in any H-like
1 2
atom= ( ∆En n in H-atom) ×Z 2
1 2

7.4.6 Success of Bohr’s Theory


(a) Bohr successfully combined Rutherford’s model with the Planck hypothesis on the quantified energy states
at atomic level
(b) Bohr’s theory explained the atomic emission and absorption spectra
(c) It explained the general characteristics of the periodic table
(d) Bohr’s theory offered the first “working” model for the atom

7.4.7 Short Coming of Bohr’s Model


(a) Bohr’s model holds true only for atoms with one electron. E.g, H, He+, Li+2, Na+1
(b) Bohr’s model posits circular orbits whereas according to Somerfield these are elliptical.
(c) The model could not explain the intensity of spectral lines.
(d) It assume the nucleus to be stationary, but it also rotates on its own axis.
(e) It failed to account for the minute structure in spectrum line.
(f) The model offered no explanation for the Zeeman effect (splitting up of spectral lines in magnetic field) and
Stark effect (splitting up of spectral lines in electric field)
(g) Doublets observed in the spectrum of some of the atoms like sodium (5890 Å & 5896 Å) could not be
explained by Bohr’s model.

7.4.8 de Broglie’s Explanation of Bohr’s Second Postulate of Quantization

In Bohr’s model of the atom, it is stated that the angular momentum of the electron orbiting around the nucleus is
nh
quantized (that is, Ln = ; n = 1, 2, 3,..…). Why is it that the values of angular momentum are only integral

h
multiples of 
2π 
De Broglie, speculated that nature did not
single out light as being the only matter
which exhibits a wave-particle duality. He
proposed that ordinary ``particles’’ such as
electrons, protons, or bowling balls could Nucleus
also exhibit wave characteristics in certain r
circumstances.C.J. Davisson and L.H. German
later experimentally verified the wave nature
of electron in 1927. It was De Broglie’s
contention (like Bohr) was that an electron in
motion around the nucleas must be seen as a
particle wave. Analogous to waves travelling A standing wave is shown in a circular orbit where four de Broglie
wavelength fit into the circumference of the orbit.
on a string, particle waves too can lead to
standing waves under resonant conditions. Figure 24.38: De broglie model
P hysi cs | 24.25

We know that when a string is peturbed, it generates a number of wavelengths along the length of the string.
Of these, only those wavelengths that have nodes at either ends and form standing waves survive, while other
wavelengths get reflected upon themselves resulting in their amplitudes quickly dropping to zero. Therefore,
standing waves are formed when a wave travels the along the enrire length of the string and back in one, two, or
any integral number of wavelengths. For an electron moving in nth circular orbit of radius rn , the total distance is
the circumference of the orbit, 2πrn .
h
Thus, 2πrn =nλ , n = 1, 2, 3 …….. We have, λ = , where p is the magnitude of the electron’s momentum.
p
If the speed of the electron is much less than the speed of light, the momentum is mvn.
h nh nh
Thus, λ = .; 2πrn = or mv nrn =
mv n mv n 2π

This is the quantum condition proposed by Bohr for the angular momentum of the electron. Thus de Broglie
hypothesis provided an explanation for Bohr’s second postulate for the quantization of angular momentum of the
orbiting electron by postulating the wave nature of matter particles like electrons. The quantized electron orbits
and energy states are due to the wave nature of the electron and only resonant standing waves can persist.

7.4.9 Limitations
(a) The Bohr model is applicable to hydrogenic atoms with a single electron. All attempts to use Bohr’s Model
to analyze atoms with more than one electron failed as Bohr’s model deals only with interaction between
the electron and the positively charged nucleus but does not account for the interaction of an electron with
other electrons as would be the case with multi-electron atoms.(ii) While the Bohr’s model correctly predicts
the frequencies of the light emitted by hydrogenic atoms, it cannot predict the relative intensities of spectral
lines. Some frequencies in the hydrogen emission spectrum, for example, have weak intensity while others
have strong intensity. Bohr’s model is unable to account for the intensity variations.

7.4.10 Some Important Definitions and their Meaning


Energy state

Ground state Excited state


or
Most stable state
n=1

Energy state where electrons Energy state where electrons


experience attractive force of do not experience nuclear
nucleus n= m+1 attraction = Ionised state
If m=number of excited state =
Figure 24.39: Energy level classification

(a) Ionization energy and ionization potential: The ionization energy is the energy necessary to remove an
electron from the neutral atom. It is a minimum for the alkali metals which have a single electron outside a
closed shell. The ionization potential is the potential through which an electron is accelerated for removal an
electron from the neutral atom is called ionization potential.
I.E. = E∞ − E1 =−E1 = Binding energy of e− ( e∞ assumed to be zero)
(b) Excitation energy and excitation potential: The minimum energy required to excite an atom i.e., alteration
from the condition of lowest energy (ground state) to one of higher energy (excited state) is called excitation
energy of the particular excited state and corresponding potential is called excitation potential.
2 4 . 2 6 | Modern Physics

E2 n=2 E+E1=E2
E E=E2-E1

E1 n=1 Excitation energy

E3 n=3 E+E1=E3
E E=E3-E1

E1 n=1 2nd Excitation energy

E3 n=3 E+E2=E3
E E=E3-E2

E2 n=2 Excitation energy of e- for 1st excited state

E4 n=4 E+E2=E4
E E=E4-E2

E2 n=2 2ndExcitation energy of e- for 2nd excited state

E 
E+E1=E=0
E
E=-E1
E1 1 (I.E.)

E 
E+E2=E=0
E E=E-E2=-E2
E2 2 (I.E.) of in 1st excited state

Figure 24.40

If excitation energy and ionization energy are represented in eV, then corresponding value in volt is termed as
excitation potential and ionization potential, respectively.
For Example: Excitation energy and ionization energy for H-atom are 10.2 eV and 13.6 eV, respectively and,
therefore, 10.2V and 13.6V are excitation and ionization potential, respectively.

PLANCESS CONCEPTS

Reduced mass: Both the proton and electron revolve in circular orbits about their common centre of
mass. However, we can account for the motion of the nucleus simply by replacing the mass of electron
m by the reduced mass µ of the electron and the nucleus.
Mm
Here µ = ….(i)
M+m
m
Where M = mass of nucleus. The reduced mass can also be written as, µ =
m
1+
M
Note: If motion of the nucleus is also considered, then m is replaced by µ, where µ = reduced mass of
mM Z2 µ
electron – nucleus system = . In this case, En = ( −13.6eV) .
m+M n2 me
Vaibhav Krishnan (JEE 2009, AIR 22)
P hysi cs | 24.27

8. SPECTRUM

8.1 Types of Line Spectrum


Emission line spectrum: When an electric current passes through a gas which is at less than atmospheric pressure,
it gives energy to the gas. This energy is then given out as light of several definite wavelengths (colours). This
is called a emission line spectrum. These are caused when an electron hops from excited states to lower states.
Different The wavelength of emission lines of different elements have emissions of different wavelengths. For one
element the emission spectrum are unique for each element.
Absorption line spectrum: It is the electromagnetic spectrum, broken by a specific pattern of dark lines or bands,
observed when radiation traverses a particular absorbing medium and through a spectroscope. The absorption
pattern of an element is unique and can be used to identify the substance of the medium. When white light is
passed through a gas, the gas is absorbs light of certain wavelength. The bright background on the photographic
plate is then crossed by dark lines that corresponds to those wavelengths which are absorbed by the gas atoms,
resulting in transition of an atom from lower energy states to higher energy states.
(The emission spectrum consists of bright lines on dark background.)
The spectrum of sunlight has dark lines called Fraunhoffer lines. These lines are produced when the light
emanating from the core of the sun passes through the layer of cooler gas. This layer absorbs light of certain
wavelengths corresponding to the elements present in the cooler gas. This results in dark lines (absorption of
certain wavelengths) on a brighter background. Fraunhoffer lines reveal the composition of the star.

8.2 Time Period and Frequency of Electron’s Motion


2πrn n3 h3 n3
(a) Time period of revolution of an electron in the nth Bohr orbit is Tn = = = 1.5 × 10−16 sec
vn Z2 4 π2mk 2e4 Z2
For H-atom, Z = 1 ; then for n = 1, T= 1.5 × 10−16 sec T1 : T2 : T3 = 1 : 8 : 27
1 ,
1 Z2
(b) Frequency of revolution v n = vn ∝
Tn n3
1 1
For H-atom v=
1 6.6 × 1015 Hz , v1 : v 2 : v 3 = 1 : :
8 27
(c) Current and Magnetic field Due to Electron’s Motion: The motion of an electron in a circular orbit, gives
rise to some equivalent current in the orbit. It is equal to (in the nth orbit) M = current × area; Mn= In .πrn2 ;
nhe eL
Mn = ; Mn =
4 πm 2m
nh
Where L = , angular momentum of the electron in its orbit.

What you must memorise is their dependence on Z and n and order of magnitudes in first Bohr orbit.
n3
Tn ∝ ; T1 ≈ 1.5 × 10−16 s
Z2
Z2
vn ∝ ; v1 ≈ 6.6 × 1015 Hz
3
n
Z2
ωn = 2πv n ; ωn ∝
n3
nh
Ln = ; Ln ∝ n

2 4 . 2 8 | Modern Physics

PLANCESS CONCEPTS

1
Total energy of an electron in an atom = × potential energy of electron = − kinetic energy of electron
2
Nivvedan (JEE 2009, AIR 113)

8.3 Determination of Number of Spectral Lines (Theoretical) in Emission and in


Absorption Transitions

8.3.1 Number of Emission Spectra Lines


When an electron is in an excited state with principal quantum number n, then the electron may go to (n – 1)th state,
………., 2nd state or 1st state from the nthstate. Therefore, there could be (n – 1) possible transitions starting from the
nth state. The electron reaching (n – 1)th state may make (n – 2) different transitions. Similarly for other lower states,
the total number of possible transitions is
n (n − 1)
(n – 1) + (n – 2) + (n – 3) + ……… 2 + 1 =
2

8.3.2 Number of Absorption Spectral Line


At ordinary temperatures almost all the atoms remain in their lowest energy level (n = 1) and, therefore. absorption
transition can start only from the lowest energy level i.e., n = 1 level (not from n = 2, 3, 4, …… levels). Hence, only
Lyman series is found in the absorption spectrum of hydrogen atom (which as in the emission spectrum, all the
series are found)
Number of absorpton spectral lines = (n – 1)
Remember: The absorpton spectrum of sun has Balmer series also besides the Lyman series. Many H-atoms
remain in n = 2 also due to very high temperature.

8.4 Explanation of H-Spectrum and Spectral Line Formula


In a single-electron atom, the transition of an electron from any higher energy state n2 to any lower energy state
n1 causes a photon of frequency v or wavelength λ to be emitted.

hc Z2 Z2 n2
Then ∆E= hv = = En − En ; E = −Rch J = −13.6 eV En2
λ 2 1
n2 n2
1 v l
RchZ 2  RchZ 2 
21 
∴ ∆E = − −  ⇒ ∆E Rch Z 
= −  n1
n22  n2   n2 n2  En1
 1   1 2 
Figure 24.41
hc 1 1  1 1 1 
⇒ h=
ν = RchZ 2  −  ⇒ ν= = R Z 2  − 
λ  n2 n2  λ  n2 n2 
 1 2   1 2 

v = wave number = number of wave in unit length v = cv

1 1 1 
For H-atom, Z= 1 & there for,
= R − 
λ n2 2 
 1 n2 

8.5 Hydrogen Spectral Series


(a) Lyman series: n1 = 1 , n2 = 2 , 3, 4, ….. ∞
P hysi cs | 24.29

3
2
1
Absorption line
Figure 24.42


Last line
or series limit
3
2
First line
1
Emission line
Figure 24.43

1 1 1 4
For 1st line or series beginning n1 = 1 , n2 = 2=
; R  −  ; λmax = = 1216 Å
λ 12 2
2  3R

1 1 1  1
For series limit or last line n1 = 1 , n2 = ∞=
; R − ; λmin = =912.68 Å
λ 2 2 R
1 ∞ 
* Remember – Lyman series is found in UV region of electromagnetic spectrum
(b) Balmer series:

Series limit

4
3
First line
2

n=1
Energy level
Figure 24.44

n1 = 2, n2 = 3, 4, 5, 6, …… ∞ Wavelength of first line

1 1 1
i.e. maximum wavelength 6563Å
= R  2 − 2  ; ∴ λmax =
λmax 2 3 
Wavelength of last line or series limit i.e. minimum wavelength

1 1  4
λmin = R  − ; λmin = = 3646 Å
2 2 R
2 ∞ 
* Balmer series is found only in emission spectrum.
* Balmer series lies in the visible region of electromagnetic spectrum. Only the first four lines of Balmer series
lies in visible region. Rest of them lie in the infrared region of EM spectrum.
2 4 . 3 0 | Modern Physics

(c) Paschen series:=


n1 3,=
n2 4, 5, 6 . . . .∞


5
4
3

n=2

n=1
Figure 24.45

1 1 1 
n1 3,=
For first line= n2 4 , then R  −
=× 
2
λmax 3 42 

18751 Å For last line or series limit


λmax =



5
4
n=3
n=2
n=1
Figure 24.46

1 1 1  9
n1 = 3 , n2 = ∞ ; = R  2 − 2  ; λmin = = 8107 Å
λmin 3 ∞  R

* Paschen series is also found only in emission spectrum.


* Paschen series is obtained in infrared region of electromagnetic spectrum.

n1 4,
(d) Brackett series –= = n2 5, 6, 7 . . .∞


6
5
4
3
2
n1=1
Figure 24.47

1 1 1
For first list 40477 Å
= R  2 − 2  ; λmax =
λmax 4 5 
P hysi cs | 24.31

1 1 1  16
For last line or series limit = R  2 − 2  ; λmin = = 14572 Å
λmin 4 ∞  R

* Brackett series is also found only in emission spectrum.

* Brackett series is also obtained in infrared region of electromagnetic spectrum.

(e) Pfund series-

last line

6
first line
5

4
3
2
n=1

Figure 24.48

n1 = 5 , n2 = 6 , 7, 8, …... ∞

1 1 1
For first line = R  2 − 2  ; λmax = 74515 Å For last line or series limit
λmax 5 6 

1 1 1  25
= R  − 2  ⇒ λmin = = 22768 Å

λmin 5 ∞  R

* Pfund series is also obtained only in emission spectrum.

* Pfund series is situated in the infrared region of electromagnetic spectrum.

PLANCESS CONCEPTS

The minimum wavelength of a series (Lyman, Balmer, Paschen, Brackett etc.) correlates with the ionization
potential of the electron from that shell.
Chinmay S Purandare (JEE 2012, AIR 698)

General Point for Spectral Lines in Every Spectral Series


(a) Wavelength of first line is maximum and last line is minimum.
(b) As the order of spectral series increases, wavelength also usually increases
λPF > λBR > λP > λB > λL

(c) Frequency of energy emission in Lyman transitions are highest among all other series.
2 4 . 3 2 | Modern Physics

PLANCESS CONCEPTS

1
Total energy of an electron in an atom = * Potential energy of electron
2

= - Kinetic energy of electron
* If an electron jumps from then ∆=
E Ehigh − Elow
Where Elow is the low-energy state from where the jump begins and Ehigh is the high-energy state where
the jump ends.
Nitin Chandrol (JEE 2012, AIR 134)

Illustration 22: What will be the two highest wavelengths of the radiation emitted when hydrogen atoms make
transitions from higher states to n = 2 states? (JEE ADVANCED)

Sol: For electronic transition from energy state En > E2 (where n = 3, 4, 5…) to E2, the spectral series corresponds
1 1 1
to Balmer series. Therefore the wavelength of this transition is  = R  2 − 2  where n = 3, 4 ,5....∞ and R is
λ 2 n 
Rydberg’s constant.
The highest wavelength corresponds to the lowest energy of transition. This will be the case for the transition n =
3 to n = 2. The second highest wavelength corresponds to the transition n = 4 to n = 2.
E1
The energy of the state n is En =
n2
13.6eV 13.6eV 13.6eV
Thus, E2 =
− −3.4 eV ; E3 =
= − −1.5eV ; and E4 =
= − −0.85 eV
=
4 9 16

hc 1242eV × 1 nm
The highest wavelength is λ1 = = = 654 nm
∆E (3.4eV − 1.5eV)

1242eV × 1 nm
The second highest wavelength is λ2 = = 487 nm.
(3.4 eV − 0.85eV)

Illustration 23: The particle µ-meson has a charge equal to that of an electron and a mass that is 208 times that
of the electron. It moves in a circular orbit around a nucleus of charge +3e. Assume that the mass of the nucleus
is infinite. Supposing that Bohr’s model is applicable to this system, (a) derive an equation for the radius of the nth
Bohr orbit, (b) find the value of n for which the radius of the orbit is approximately the same as that of the first Bohr
orbit for a hydrogen atom (c) find the wavelength of the radiation emitted when the µ-meson jumps from the third
orbit to the first orbit. (JEE ADVANCED)

n2h2 ε0
Sol: According to Bohr’s theory, the radius of nth Bohr’s orbit is rn = and energy of µ-meson in nth orbit
πme2 Z
mZ 2e4
is En = − . If µ-meson jumps from a higher energy orbit to a lower energy orbit, the energy emitted is
8ε20n2h2
1 1
∆E = Z 2 × 13.6 ×  −  eV . To derive the expression for the nth orbit we have to keep in mind that the electrostatic
 nf ni2 
2

force of attraction between µ-meson and the nucleus provides the required centripetal force for circular orbit.
h
According to Bohr’s postulate, the magnitude of angular momentum of µ-meson must be integral multiple of .

P hysi cs | 24.33

2
mv 2 Ze2 2 Ze
(a) We have, = or v r =  … (i)
r 4 πε0r 2 4 πε0m
nh
The quantization rule is vr =
2πm

(vr)2 n2h2 4 πε0m n2h2 ε0


The radius is r = = =  … (ii)
v 2r 4 π2m2 Ze2 Zπme2
n2h2 ε0
For the given system, Z = 3 and m = 208 me .;Thus rµ = .
624 πmee2
h2 ε 0
(b) From (ii), the radius of the first Bohr orbit for the hydrogen atom is rh = .
πmee2
n2h2 ε0 h2 ε0
For rµ = rh , = or, n2 = 624 or, n = 25
2 2
624 πmee πmee
mv 2 Ze2 Ze2
(c) From (i), the kinetic energy of the atom is = and the potential energy is − .
2 8πε0r 4 πε0r
Ze2
The total energy is En = − Using (ii),
8πε0r

9 × 208mee4 1872  mee 


4
Z 2 πme4
En = − = − = −  … (iii)
8πε20n2h2 8ε20n2h2 n2  8ε20h2 

 m e4 
But  − e  is the ground state energy of hydrogen atom and hence is equal to −13.6 eV .
 8ε2h2 
 0 

1872 −25459.2eV
From (iii), En = − × 13.6 eV =
2
n n2
E1
Thus, E1 = - 25459.2 eV and E3 = = -2828.8 eV, The energy difference is E3 − E1 = 22630.4 eV.
9
hc
E3 − E1 =
λ
o
o
hc 12375eV − A
=
⇒λ = = 0.5468 A
E3 − E1 22630.4 eV

Illustration 24: A neutron moving with speed v makes a head-on collision with a stationary hydrogen atom in
ground state. Determine the minimum kinetic energy of the neutron for which inelastic (completely or partially)
collision may take place. The mass of neutron ≈ mass of hydrogen = 1.67 × 10−27 kg . (JEE ADVANCED)

Sol: It is important to remember the hydrogen atom will absorb the kinetic energy lost in an inelastic collision,
causing the atom to reach one of its excited states. The quantum of energy thus absorbed by hydrogen atom will
be equal to what is required to reach a possible excited state, and not more. Since the hydrogen atom is initially
in ground state (n = 1), the minimum energy it can absorb will be equal to that required to reach the first excited
state (n = 2). If the colliding neutron’s kinetic energy is less than this minimum energy, no energy will be absorbed,
i.e., inelastic collision may not take place.
Let us assume that the neutron and the hydrogen atom move at speeds v1 and v 2 after the collision. The collision
will be inelastic if a part of the kinetic energy is used to excite the atom. Suppose an energy ∆E is used in this way.
Considering collision to be inelastic, using conservation of linear momentum and energy,
mv mv1 + mv 2 ...(i)
=
2 4 . 3 4 | Modern Physics

1 1 1
And mv 2
= mv12 + mv 22 + ∆E …(ii)
2 2 2
2∆E 2∆E
From (i), v 2 = v12 + v 22 + 2v1 v 2 ; From (ii), v 2 = v12 + v 22 + Thus, 2v1 v 2 =
m m
4 ∆E 4 ∆E
v2 −
Hence, (v1 − v 2 )2 = (v1 + v 2 )2 −4v1 v 2 = ; As v1 − v 2 must be real, ; v 2 − ≥0;
m m
1
or mv 2 > 2∆E .
2
The minimum energy that can be absorbed by the hydrogen atom in ground state to go in an excited state is 10.2 eV.
1 2
Thus, the minimum kinetic energy of the neutron needed for an inelastic collision is mv min 2 × 10.2eV =
= 20.4 eV
2

1
Illustration 25: The potential energy U of a small moving particle of mass m is mω2r 2 , where ω is a constant and
2
r is the distance of the particle from the origin. Assuming Bohr’s model of quantization of angular momentum and

circular orbits, show that radius of the nth allowed orbit is proportional to n .  (JEE ADVANCED)
dU
Sol: The force acting on the particle in the radial direction Fr = − provides the necessary centripetal acceleration
dr
for the particle to move in a circular orbit.

dU
The force at a distance r is Fr =− =−mω2r  … (i)
dr
mv 2
Suppose the particle moves along a circle of radius r. The net force on it should be along the radius.
r
mv 2
Comparing with (i), =mω2r ⇒ v =rω  ... (ii)
r
nh nh
The quantization of angular momentum gives mvr = or, v =  … (iii)
2π 2πmr
1/2
 nh 
From (ii) and (iii), r =   .
 2πmω 
Thus, the radius of the nth orbit is proportional to n.

9. BINDING ENERGY
Binding energy,is amount of energy required to separate a particle from a system of particles or to disperse all
the particles of the system. Conversely it also defined as the energy released when particles are brought together
to form a system of particles. For example, if an electron and a proton are initially at rest and brought from large
distances to form a hydrogen atom, 13.6 eV energy will be released. The binding energy of a hydrogen atom is,
therefore, 13.6 eV, same as its ionization energy.

10. CONCEPT OF RECOILING OF AN ATOM DETERMINATION OF


MOMENTUM & ENERGY FOR RECOIL ATOMS
When a nuclear particle is emitted or ejected at high velocity from an atom the remainder of the atom recoils with a
velocity inversely proportional to its mass. This happens when an electron makes transition from any higher energy
state to any lower energy state. The atom is recoiled by sharing some energy from the energy evolved during
electronic transition.
P hysi cs | 24.35

If m = mass of recoiled atom, V = velocity of recoiled atom  En n2


2

1 hc
Then mv 2 + =En − En = ∆E
2 λ 2 1 l
h
Recoil momentum of atom = = momentum of photon En n1
λ 1

p2 Figure 24.49
Recoil energy of atom =
2m

Illustration 26: Given that the excitation energy of a hydrogen-like ion in its first excited state is 40.8 eV, determine
the energy needed to remove the electron from the ion. (JEE MAIN)
 1 
Sol: The excitation energy for hydrogen like ion for (n-1)th excited state (nth orbit) is E = hc × R × Z 2  1 −  where
n = 2, 3, 4, ….etc. The energy needed to remove the electron from the ion is E = hc × R × Z 2 .  n2 

The excitation energy in the first excited state (n=2) is


1 1  2 3
=E RhcZ 2  −  = (13.6 eV) × Z × .
12
22  4
Equating this to 40.8 eV, we get Z = 2. So, the ion in question is He+ .
RhcZ 2
The energy of the ion in the ground state is E = − = −4 × (13.6 eV) = −54.4 eV
12
Thus 54.4 eV is required to remove the electron from the ion.

PLANCESS CONCEPTS

The energy of a photon and its wavelength are inversely proportional.


B Rajiv Reddy JEE 2012, AIR 11

11. THE WAVE FUNCTION OF AN ELECTRON


Quantum mechanics has enabled physicists to develop a mathematically and logically rigorous theory which
describes the spectra in a much better way. The following is a very brief introduction to this theory.

We have already seen that to understand the behavior of light, we understand it as both a wave (the electric field E

(as well as a particle (the photon). The energy of a particular ‘photon’ is related to the ‘wavelength’ of the E wave.
Light going in x direction is represented by the wave function.= E(x,t) E0 sin(kx − ωt) In general, if light can go in
   
any direction, the wave function is = E(r , t) E0 sin(k . r − ωt)  ... (i)
 
Where r is the position vector; k is the wave vector.

11.1 Quantum Mechanics of the Hydrogen Atom



The wave function Ψ(r , t) of the electron and the possible energies E of a hydrogen atom or a hydrogen-like ion
are obtained from the Schrodinger’s equation.

−h2  ∂ 2 Ψ ∂ 2 Ψ ∂ 2 Ψ  Ze2 Ψ
 + + − EΨ 
= ... (ii)
8π2m  ∂x2 ∂y 2 ∂z 2  4 πε0r

Here (x, y, z) refers to a point with the nucleus as the origin and r is the distance of this point from the nucleus. E
refers to energy. The constant Z is the number of protons in the nucleus. For hydrogen, we have to put Z = 1. There
2 4 . 3 6 | Modern Physics


are infinite number of functions Ψ(r ) which satisfy equation (ii). These functions, which are solutions of equation
(ii), may be characterized in terms of three parameters n, l and ml With each solution Ψnl m , there is associated
l
a unique value of the energy E of the atom or the ion. The energy E corresponding to the wave function Ψnl m
l
mZ 2e4
depends only on n and may be written as En = −
8ε20h2n2

12. LASER

12.1 Basic Process of Laser


The basic strategy to get light amplification by stimulated emission is as follows: 
E2 E2
A system is chosen which has a metastable state at having an energy E2 (See Fig. 24.50).
There is another allowed energy E1 which is less than E2. The system could be any of the Incident
following: a gas or a liquid in a cylindrical tube or a solid in the shape of a cylindrical rod. photon
Let us assume that the number of atoms in the metastable state E2 is increased to more
than that in E1. Let us also assume that a photon of light of energy E2-E1 is incident on one
of the atoms in the metastable state E2. Then this atom drops to the state E1 i.e., emitting a
photon in the same phase, energy, and direction as the first one. Then these two photons E1 E1
interact with two more atoms in the state E2 and so on. Therefore, the number of photons Stimulated
keeps on increasing. All these photons will have the same phase, the same energy, and the absorption
same direction. Thus, the amplification of light is achieved.
Figure 24.50: Laser

12.2 Working
When power is suppliedy and the electric field is established, E2
some of the atoms of the mixture get ionized. These ionized Metastable state
atoms release some electrons which are accelerated by the high
electric field. Consequentially, these electrons collide with helium
atoms to take them to the metastable state at energy E3 . These
atoms collide with a neon atom and transfer the extra energy to E1
it. As a result, the helium atom returns to its ground state and the
Figure 24.51: metastable state of electron
neon atom is excited to the state at energy E2. This process keeps
looping so that the neon atoms are continuously pumped to the
state at energy E2, keeping the population (of atoms) of this state large.

12.3 Uses of Laser


(a) Spectroscopy: Most lasers being inherently pure source of light, emit near monochromatic light with a very
clear range of wavelengths. This makes the laser ideal for spectroscopy.
(b) Heat Treatment: In laser heat treating, energy is transmitted to the material’s surface in order to create
a hardened layer by metallurgical transformation. The use of lasers results in little or no distortion of the
component and, as such, eliminates much of part reworking that is currently done. Therefore, the laser heat
treatment system is cost-effective.
(c) Lunar laser ranging: The Apollo astronauts planted retroreflector arrays on te moon to make possible the
Lunar Laser Ranging Experiment. In this experiment laser beams are focused, through large telescopes on
Earth, on the arrays, and the time taken for the beam to be reflected back to Earth is measured to determine
the distance between the Earth and Moon with high accuracy.
(d) Photochemistry: Extremely brief pulses of light – as short as picoseconds or femtoseconds (10 – 12 to 10 – 15
s) – produced by some laser systems are used to initiate and analyze chemical reactions. This technique is
known as photochemistry.
(e) Laser Cooling: This technique involves atom trapping, wherein a number of atoms are enclosed in a specially
shaped arrangement of electric and magnetic fields.
P hysi cs | 24.37

(f) Nuclear Fusion: Powerful and complex arrangements of lasers and optical amplifiers are used to produce
extremely high-intensity pulses of light of extremely short duration. These pulses are arranged to impact
pellets of tritium-deuterium, simultaneously, from all directions, hoping that the compression effect of the
impacts will induce atomic fusion in the pellets.

13. X-RAYS
X-radiation is a form of electromagnetic radiation. Most X-rays have a wavelength ranging from 0.01 to 10
nanometers, corresponding to frequencies in the range 30 petahertz to 30 exahertz (3×1016 Hz to 3×1019 Hz) and
energies in the range 100 eV to 100 keV. X-radiation is also referred to as Röntgen radiation, after Wilhelm Röntgen,
who is usually credited as its discoverer, and who had named it X-radiation to signify an unknown type of radiation
produced when electron collided with the walls of the tube.
The wave nature of X-rays, was established by Laue who demonstrated that they are diffracted by crystals.

13.1 Production
High P.D.
The modern X-ray tube, called Coolidge tube, is shown in the
Fig. 24.52. A heated element emits electronswhich are
accelerated towards a cooled copper anode under a high Target
potential difference. A target metal of high atomic number Filament
Cooled
and high melting point is lodged on the anode. copper anode

The intensity of the X-ray beam is controlled by The filament


current controls the intensity of the X-ray beam by regulating Concave focusing
the number of electrons striking the target per unit time. The cathode
potential difference between the cathode and anode controls X-rays
the penetrating power of the beam.
Figure 24.52: X- Ray tube

13.2 X-Ray Spectra


A typical X-ray spectrum given by a target is shown in the Fig.24.53. The spectrum is basically continuous range of
wavelengths starting from a minimum value. A line spectrum having sharp wavelengths is superimposed on this.

Characteristic spectrum
Intensity

Continuous spectrum

min 
Figure 24.53: X-Ray spectra

13.3 Origin Of Characteristic Spectrum


If an incoming electron knocks out an electron in one of the inner shells, the exiting electron creates a vacancy
in that shell. This vacancy gets fille by another electron from a higher shell that makes a transition to this shell,
creating another vacancy in the higher shell. This hopping of electrons from higher to lower shells continues till the
inner shells are filled up. This process produces a series of radiations, some of which pertain to the X-ray region.
These radiations are typical of the target element. The X-ray spectrum of a substance is classified into K-series,
L-series, M-series etc.
2 4 . 3 8 | Modern Physics

13.4 Moseley’s Experiment and the Concept of Atomic Number


Moseley’s experiment involved the analysis of the X-ray spectra of 38 different
elements, ranging from aluminum to gold. He measured the frequency of v
principal lines of a particular series (the α-lines in the K-series) of the spectra
and was able to show that the frequencies of certain characteristic X-rays
emitted from chemical elements are proportional to the square of a number
which was close to the element’s atomic number (Z). He presented the
following relationship: =
ν a (Z − b)
z
where v = frequency of X-rays, Z = atomic number, a and b are constants. On -b
plotting the values of square root of the frequency against atomic numbers
Figure 24.54
of the elements producing X-rays on a graph, a straight line was obtained.
When electron is knocked out from n1 energy state and it is filled with electron from n2 energy state wavelength
1 1 1
of X-ray emitted is R (Z − b)2  − 
=
λ  n1 n22 
2

For K-series b = 1, n1 = 1 and n2 = 2,3,......... For L-series : b = 7.4, n1 = 2 , n2 = 3, 4,.......


For K α -line, the electron jumps from L-level to a vacancy in the K-kevel. So for the L electron, there are Z protons
in the nucleus and an electron in the K-shell which screens off the positive charge. So the net charge the L electron
faces can be taken as (Z – 1)e.

2 3
Now, E = Rhc ( Z − 1 ) = ⇒ h ν R hc ( Z − 1 ) ×  
2
=
4
3R c
=v (Z − 1)
4

13.5 Origin of Continuous Spectrum


When an incident electron comes very close to a target nucleus, it is suddenly accelerated due to the electrostatic
field around the nucleus (Coulomb field) and emits electromagnetic radiation. This radiation is referred to as
breaking radiation and is continuous. The minimum wavelength (and maximum frequency) correlates with an
electron losing all its energy in a single collision with a target atom.
hc hc 12420
If V is the acceleration p.d., then hνmax = = eV or λmin = = Å
λmin eV V

13.6 Properties of X-rays


(a) X-rays ionize the material which they penetrate.
(b) They produce the same effect on photographic plates as visible light.
(c) They cause fluorescence when they act on certain chemical compounds like zinc sulphide.
(d) X-rays penetrate matter and get absorbed as they pass through it. If I0 is the intensity of incident relation

and I is the intensity after travelling through a distance x, then I = I0 e−µx where µ is called the absorption

coefficient of the material. The atomic number of the material and its absorption coefficient are directly
proportional. This is the basis of radiography.
(e) They cause photoelectric emission.
(f) Electric and magnetic fields have no effect on X-rays as they contain no charged particles.

Note: X-Rays are not affected by electric or magnetic fields. Intensity of X-rays depends on number of electrons
in the incident beam.
P hysi cs | 24.39

PROBLEM-SOLVING TACTICS

(a) This section of Physics is more fact-based. The key to answering questions of these sections is establish a lonk
between the known and asked quantities
(b) One has to be very conversant with the formulae and standard scientific constants.
(c) In this section, graphical questions seeking relationship between various fundamental quantities are usually
asked. Assign the dependent variable as y and the independent variable as x and then look for a relation
between them.
(d) One must not get confused about approaching the questions from a wave nature or particle nature or try
to combine both. Just solve questions on the basis of the known and asked quantities and the relationship
between the two.
(e) It is important to learn the scientific constants in various units to avoid unnecessary unit conversion. (e.g.,
if energy of a photon is in eV units and wavelength asked in angstrom, one can directly use the relation =
12400/E, here 12400 is the product of Planck’s constant and speed of light.)
(f) Analytical questions pertaining to H-atom can be solved easily if one knows proportionality relation between
quantities. They need not be learnt by heart. They can be derived without bothering about constants appearing
in these relations. (e.g., radius of nth shell is directly proportional to n2 , keeping Z constant.)

FORMULAE SHEET

Speed of E.M.W. in vacuum c = 3 × 108 m / s = νλ


hc
Each photon having a frequency ν and energy E = hν = where h = 6.63 × 10−34 Js is Planck’s Constant
λ

Einstein’s Photo Electric Equation:


Photon energy = K.E. of electron + work function.
1

= mv 2 + φ
2
φ = Work function = energy needed by the electron in freeing itself from the atoms of the metal.
φ =hv 0
The minimum value of the retarding potential to prevent electron emission is:

eVcut off = (KE)max


h
De Broglie wave length given by λ = (wave length of a particle)
p
mv 2 kZe2
The electron in a stable orbit does not radiate energy i.e. =
r r2
A stable orbit is that in which the angular momentum of the electron about nucleus is an integral (n) multiple of
h h
i.e. mvr = n ; n = 1, 2, 3, ….. (n ≠ 0) .
2π 2π
For Hydrogen atom : (Z = atomic number = 1)
h
(i) Ln = angular momentum in the nth orbit = n

2 4 . 4 0 | Modern Physics

n2h2 ε0 −10 2
(ii) rn = radius of nth circular orbit = rn = 2
(0.529 Å) n2 ; (1 Å = 10 m) ; rn ∝ n
πme

−13.6eV 1
(iii) En energy of the electron in the nth orbit = i.e. En ∝
2
n n2
2
e
(iv) nth orbital speed v n =
2ε0nh

Note: Total energy of the electron in an atom is negative, indicating that it is bound.
13.6eV
Binding Energy (BE)n = −En =
n2
(iv) En − En = Energy emitted when an electron jumps from n2 orbit to n1 orbit (n2 > n1 ) .
2 1

1 1
∆E = (13.6 eV)  − 
 n12 n22 

∆E = hν ; ν= frequency of spectral line emitted.

1 1 1
Wave number= ν = = [no. of waves in unit length (1m)] = R  − 
λ  n12 n22 

Where R = Rydberg’s constant for hydrogen = 1.097 × 107 m−1

(v) For hydrogen like atoms of atomic number Z:

Bohr radius 2 n2
= rnz × n = (0.529Å) ;
Z Z

Z2
EnZ = ( −13.6) ev
n2

Note: If motion of the nucleus is also considered, then m is replaced by µ.

mM
Where µ = reduced mass of electron – nucleus system =
m+M

Z2 µ
In this case, En = ( −13.6 eV) .
2 me
n
En − En
2 1
Excitation potential for quantum jump from n1 → n2 =
electron charge

From Mosley’s Law ν= a(z − b) where b (shielding factor) is different for different series.

1 1 1 
For x-rays =R× (Z − b)2 ×  − 
λ n2 2 
 1 n2 
R = R 0 A1/3 . Where R 0 = empirical constant = 1.1 × 10 −15 m ; A = Mass number of the atom.
P hysi cs | 24.41

Solved Examples

JEE Main/Boards KEmax 0.57 × 1.6 × 10−19


V= = = 0.57 V.
e 1.6 × 10−19
Example 1: Calculate the energy of α -particle in the
event of its head-on collision with gold nucleus if the
Example 3: Determine the de Broglie wavelength of an
closest distance of approach is 41.3 Fermi.
electron having kinetic energy of 500 eV?
Sol: The kinetic energy of α -particle is converted into
the electric potential energy at the distance of closest Sol: The de-Broglie wavelength of electron moving
approach in the event of a head-on collision. The kinetic h
with Kinetic energy K is given as λ =
qq 2mK
energy of alpha particle is thus E = 1 2
4 πε0r h h
Using =λ = we get
mv 2mK
Given r0 = 41.3 × 10−15 m, Z = 70, q1= Ze= 79e and q2
= 2e,
6.6 × 10−34
λ=
Ze(2e) 9 × 109 × 79 × 2(1.6 × 10−19 )2 2 × 9.1 × 10−31 × 500 × 1.6 × 10−19
As E = =
4 πε0r 41.3 × 10−15
λ =  0.5467 × 10−10 m .
9 × 79 × 2 × 1.6 × 1.6 × 10−14
= J
41.3 Example 4: If an X-ray tube produces a continuous
spectrum of radiation with its short wavelength end
8.814 × 10−13
= 8.814 × 10−13 J = eV = 5.51 MeV 0.65Å, what is the maximum energy of a photon in the
1.6 × 10−19 radiation?

Example 2: If the wavelength of the incident light is Sol: The energy of radiation having wavelength λ is
5000 Å and the photoelectric work function of the hc
metallic plate is 1.90 eV, find E=
λ
(a) Energy of the photon in eV Given λmin = 0.65Å = 0.65 × 10−10 m,
(b) Kinetic energy of the photoelectrons emitted h = 6.63 × 10−34 Js, c = 3 × 108 ms−1
(c) Stopping potential We know, maximum energy of X-ray photon is
hc
Sol: The energy of photon is E = hν = , where λ is hc 6.63 × 10−34 × 3 × 108
λ Emax =ν
h max = =
the wavelength of the light. This photon knocks out λmin 0.65 × 10−10 × 1.6 × 10−19
photoelectron from the surface of metal with the
= 19.13 × 103 eV = 19.13 keV
maximum kinetic energy Emax = hν − φ0 = e V where
f0 is the work function of metal and V is the stopping
potential. Example 5: If ultra-violet light of λ =2600 Å is incident
on a silver surface with a threshold wavelength for
(a) Energy of the incident photon, photoelectric emission of λ =3800 Å , calculate:
hc 6.6 × 10−34 × 3 × 108 (i) Work function
E = hν= =
λ 5000 × 10−10 (ii) Maximum kinetic energy of the emitted
−19 photoelectrons.
= 3.96 × 10 joule = 2.47 eV
(iii) Maximum velocity of the photoelectrons.
(b) Kinetic energy of the photo-electrons emitted KEmax
1 hc
= mv 2 = hν − φ0 = (2.47 – 1.90) eV = 0.57 eV Sol: The work function of metal is φ = hν th = . The
2 λ th
(c) e V = KEmax Where V is stopping potential kinetic energy with which the photoelectron is ejected
2 4 . 4 2 | Modern Physics

1
from the metal surface is E = hν − φ = mv 2 hc (6.6 × 10−34 ) × (3 × 108 )
2 =
∴ λ2 =
4.8 × 10−19 4.8 × 10−19
hc 6.63 × 10−34 × 3 × 108
(i) φ = hν th = = J
λ th 3800 × 10−10 λ2 4.125 × 10−7 m = 4125 Å
=

5.23 × 10−19
= 5.23 × 10−19 J = eV = 3.27 eV Example 7: A hydrogen-like atom (atomic number Z) in
1.6 × 10−19 a higher excited state of quantum number n can make
a transition to the first excited state by successively
(ii) Incident wavelength λ =2600 Å
emitting two photons of energies 10.20 eV and 17.00
Then KEmax of emitted photoelectrons = hν − φ ; eV, respectively. On the other hand, the atom from the
same excited state can make a transition to the second
hc 6.63 × 10−34 × 3 × 108 excited state by successively emitting two photon of
here h=
ν =
λ 2600 × 10−10 energies 4.25 eV and 5.95 eV, respectively. What are the
values of n and Z. (Ionization energy of hydrogen atom
7.65 × 10−19 = 13.6 eV)?
=7.65 × 10−19 J = =4.78 eV
1.6 × 10−19
Sol: For any hydrogen-like atom, the energy released
KEmax= (4.78 – 3.27) eV; = 1.51 eV in transition from a higher excited state to a lower
1 1
KEmax × 2 1.51 × 1.6 × 10−10 × 2 excited state is ∆E = Z 2 × 13.6 ×  −  eV where nf
(iii) Vmax = = m/s.
m 9.1 × 10−31  n2f ni2 
and ni are principle quantum numbers of final (lower)
= 7.29 × 105 m / s and initial (higher) energy states respectively.
In first case, the excited atom makes a transition from
Example 6: The photocurrent generated when a nth state to n = 2 state and two photons of energies 10.2
surface is irradiated with light of wavelength 4950 Å, eV and 17.0 eV are emitted. Hence, if Z is the atomic
vanishes if a stopping potential greater than 0.6 V is number of H-like atom, then using
applied across the photo tube. When a different source 1 1
of light is used, it is found that the stopping potential ∆E = Z 2 × 13.6 ×  −  eV ;
has changed to 1.1 V. Determine the work function of  n2f ni2 
the emitting surface and the wavelength of second 1 1
source. (10.2 + 17.0)eV =Z 2 × 13.6 ×  −   ...(i)
2
2 n2 
Sol: The maximum kinetic energy of emitted In second case, the excited atom makes a transition
photoelectron is the product of stopping potential and from nth state to n = 3 state and two photons of
electron charge, given by KEmax = eV = hν − φ , where energies 4.25 eV and 5.95 eV are emitted.
ϕ is the work function of the metal. For two different
stopping potentials we have two different wavelengths 1 1
(4.25 + 5.95) eV =Z 2 × 13.6 ×  −   ...(ii)
2
of light used. 3 n2 
Let λ1 = 4950Å, V1 = 0.6 V Dividing equation (i) and (ii), we get
hc hc 27.2 9(n2 − 4) n2 − 4 2n2 − 13
KEmax
= − φ ;  KEmax= eV1 ⇒ =
φ − eV=
1 = or = 1.185 or
λ1 λ1 10.2 2 5
4(n2 − 9) n −9
(6.6 × 10−34 ) × (3 × 108 ) 2.185
− 0.6 =
1.9 eV =
4950 × 10−10 × 1.6 × 10−19 0.185
hc or n2 = 36 or n = 6
(b) = φ + eV2 ;
λ2
Putting in equation (i), we get
hc
λ2
( )
=3.04 × 10−19 + 1.6 × 10−19 × 1.1 = 4.8 × 10−19 J 1 1 
27.2 = Z 2 × 13.6  − 2 27.2 9
 or Z = × = 9
 4 36  13.6 2
or, Z = 3.
P hysi cs | 24.43

Example 8: In a hydrogen sample, if the atoms are


(6.63 × 10−34 ) × (3 × 108 )
excited to states with principal quantum number n, = m = 113.74 Å.
how many different wavelengths may be observed in 108.8 × 1.6 × 10−19
the spectrum?
Example 10: For a hypothetical hydrogen-like atom,
Sol: The hydrogen atom excited to the principal the wavelength in Å for the spectral lines for transitions
quantum number n will emit radiations as the electron
1500p2
hop back to lower energy states. Each transition from n = p to n = 1 are given by λ = , where p
to a lower energy state emits radiation of different p2 − 1
wavelength. Thus, we get a radiation spectrum. = 2, 3, 4, ….

From the nth state, the atom may go to (n – 1) (i) Find the wavelength of the least energetic and the
th
state, ……, n = 2 state or n = 1 state. So there are most energetic photons in this series.
(n – 1) possible transitions starting from the nth state. (ii) Construct an energy level diagram for this element
The atoms reaching (n – 1)th state may make (n – 2) representing at least three energy levels.
different transitions to reach n = 1 state. In the same
(iii) Determine the ionization potential of this element?
way, for other lower states, the total number of possible
transitions is (n – 1) + (n – 2) + (n – 3) + …. 2+1
n(n − 1) Sol: If wavelength of spectral lines for transitions from
= . n = p to n = 1 are given, then the energy of radiation for
2
hc hc 1
each transition is given as = E = (1 − ) . The
Example 9: For a hydrogen-like, doubly ionized λ 1500 p2
lithium atom with atomic number Z=3, determine least energy is obtained from transition from p = 2 to
the wavelength of the radiation required to excite the p = 1 and maximum energy is obtained from transition
electron in Li2+ from the first to the third Bohr orbit. from p=∞ to p=1. The ionization corresponds to the
The ionization energy of hydrogen atom is 13.6 eV. maximum energy in the spectrum.

Sol: The energy required by the hydrogen-like atom for 1500p2 hc


Given λ = and energy is E=
transition from ground state (n=1) to any of the excited 2
p −1 λ
1 hc  1 
is ∆E 13.6 Z 2 (1 − ) .
states (nth orbit)= E
Substituting for λ we get=  1 − 2  × 1010 J
n2 1500  p 
hc
Wavelength of radiation having energy E is, λ = .
E hc  1 
The energy of nth orbit of a hydrogen like atom is given =  1 −  × 1010 eV
(1500)(1.6 × 10−19 )  p2 
13.6
as En = −  1 
n2 = 8.28  1 −  eV.
 p2 

Thus for Li2+ atom, as Z = 3, the electron energies for
8.28
the first and third Bohr orbits are: Hence energy of nth state is given b En = eV
n2
13.6 × (3)2 (i) Maximum energy is released for transition from p
For n = 1, E1 = − eV = −122.4 eV
12 = ∞ to p = 1; hence wavelength of most energetic
photon is 1500 Å.
13.6 × (3)2
For n = 3, E3 = − eV = – 13.6 eV
(3)2 Least energy is released for transition from n = 2 to n =
1 transition. For p = 2 l = 2000Å
Thus the energy required to transfer an electron from
E1 level to E3 level is, (ii) The energy level diagram is shown in the Fig. 24.60.

E E 1 − E3 = – 13.6 – (– 122.4) = 108.8eV


= (iii) The ionization potential corresponds to energy
required to liberate an electron from its ground state.
Therefore, the radiation needed to cause this transition
should have photons of this energy. i.e., ionization energy = 8.28 eV
hc Hence, ionization potential = 8.28 V
hν = 108.8 eV. The wavelength of this radiation is
λ n=3 -0.92 eV
hc
= 108.8 eV or λ = n=2 -2.07 eV
108.8eV
n=1 -8.28 eV
2 4 . 4 4 | Modern Physics

Example 11: A single electron orbiting a stationary 1 1 1 1 


nucleus of charge +Ze, where Z is a constant and e is E4 − E3 = 13.6 Z 2  −  = E1  − 
2 2 2
4 3  3 42 
the magnitude of the electronic charge, requires 47.2
eV to excite the electron from the second Bohr orbit to  1 1
= 340  −  eV = 16.53 eV
the third Bohr orbit. Find  32 4 
(i) The value of Z (iii) Minimum energy required to remove electron from
(ii) Energy required to excite the electron from the third first orbit = 340 eV.
to the fourth Bohr orbit. hc
⇒ = 340 × 1.6 × 10−19
(iii) Wavelength of the electromagnetic radiation λ
required to remove the electron from the first Bohr  6.6 × 10−34 × 3 × 108 
orbit to infinity. or λ =   m = 36.40 Å
−19
 340 × 1.6 × 10 
(iv) Kinetic energy, potential energy, and angular
momentum of the electron in the first Bohr orbit. (iv) KE of the electron in the 1st orbit
(v) The radius of the first Bohr orbit.
KE1 = −E1 = 340 eV ; PE1 = 2E1 = − 680 eV
(The ionization energy of hydrogen atom = 13.6 eV.
Angular momentum of the electron in the 1st Bohr orbit
Bohr radius = 5.3 × 1011 m)
h 6.63 × 10−34
= = = 1.055 × 10−34 kg m / s
Sol: For a hydrogen-like atom, the total energy of 2π 2π
13.6 × Z 2 (v) Radius of the 1st Bohr orbit for the given atom
electron in nth orbit is E = − eV and radius of
nth orbit is n2 Bohr radius 5.3 × 10−11
= = = 1.06 × 10−11 m
Z 5
5.3 × 10−11 n2
rn =
Z
The kinetic energy in nth orbit is equal to the magnitude JEE Advanced/Boards
of total energy in nth orbit. The potential energy in nth
orbit is equal to twice the total energy in nth orbit. Example 1: If a hydrogen atom in its ground state is
excited by means of a monochromatic radiation of
The energy required to excite the atom from n1 state
wavelength 975 Å
1 1 
to n2 state
= is E 13.6Z 2  −  eV . To remove the (a) How many different lines are possible in the resulting
n2 2 
 1 n2  spectrum?
hc (b) Calculate the longest wavelength amongst them.
E = 13.6 × Z 2 .
electron from n = 1 state to infinity, =
λ
hc The ionization energy for hydrogen atom is 13.6 eV.
So λ =
13.6 × Z 2
This atom is hydrogen like Sol: First calculate the energy of the incident photon of
 1 
Z = atomic number of the nucleus given wavelength. From the formula
= E 13.6  1 −  eV ,
 n2 
En = Energy of the electron in the nth orbit. find the value of n, i.e., the maximum excited state the
= (Z)2 (energy of the electron in the nth orbit of the hydrogen atom will reach after absorbing the photon of
given wavelength. Longest wavelength in the resulting
13.6 E1 spectrum will correspond to transition from nth orbit to
hydrogen atom) = −(Z)2 eV =
2
n n2 (n-1)th orbit.
Where E1 = Energy of the electron in the 1st Bohr orbit Energy of the ground state
of the given atom.
(n = 1) = − (ionization energy) = −13.6 eV
 1 1 
(i) Given (Z 2 ) (13.6)  −  =47.2 eV
2 2
32  The wavelength of the incident radiation
⇒ Z = 5. = λ = 975Å ∴ Energy of the incident photon
13.6
(ii) E1 = −(25) = −340 eV hc 6.63 × 10−34 × 3 × 108
1 = = = 12.75 eV
λ 975 × 10−10 × 1.6 × 10−19
P hysi cs | 24.45

Let electron be exerted to nth orbit 1


1 1  (b) Kinetic energy of incident electron mv 2 = eV
⇒ 12.75 = 13.6  −  ⇒n=4 2
2
1 n2  2eV
The quantum transitions to the less excited states give Or v =
m
six possible lines as follows:
n = 4 : (4 → 3), (4 → 2), (4 → 1) 2 × 1.6 × 10−19 × 40 × 103
= = 1.19 × 108 m / s
−31
n = 3 : (3 → 2), (3 → 1); n = 2 : (2 → 1) 9.1 × 10

The longest wavelength emitted is for the transition (4


→ 3) where energy difference is minimum Example 3: If one milliwatt of light of wavelength
4560 Å is incident on a cesium surface, calculate the
n=4 photoelectric current liberated assuming a quantum
n=3 efficiency of 0.5%.
n=2 Planck’s constant h = 6.62 × 10−34 Js and velocity of
light 3 × 108 m/s.
n=1
hc
Sol: The energy of one photon of light is E = . The
1 1  λ
Emin (E4 − E3 ) = 13.6  − 
2
3 42  number of photons incident on the surface per second
can be determined by dividing power by energy of one
hc photon. The number of photons multiplied by quantum
= 0.661eV; Thus λmax =
Emin efficiency gives the number of photoelectrons emitted
per second.
6.63 × 10−34 × 3 × 108
= ≈ 18807Å
0.661 × 1.6 × 10−19 The energy of each photon of incident light
hc (6.63 × 10−34 )(3 × 108 )
E= = = 4.35 × 10−19 J
Example 2: An X-ray tube operating at a potential λ 4560 × 10−10
difference of 40 kV produces heat at the rate of 720 W. Number of photons in one milliwatt source
Assuming 0.5% of the energy of the incident electrons
is converted into X-rays, calculate Power of light
=
Energy of one photon
(a) The number of electrons per second striking the
target.
10−3
(b) The velocity of the incident electrons. = = 2.29 × 1015 /s.
−19
4.35 × 10
Sol: When X-Rays are produced in an X-Ray tube, the 0.5
power consumed is denoted by P = IV. Some of this Number of electrons released = 2.29 × 1015 ×
100
power is wasted as heat and the rest is converted to
= 1.14 × 1013 /s
X-Rays. The electron incident per second on target is
n = I/e ∴ Photoelectric current
As 0.5% of energy is converted into X-ray, therefore = Photo charge flowing per second
heat produced per second at the target is P = 0.995
= Total electrons emitted per sec. × charge of one
VI
electron
where, I is current inside tube
= (1.14 × 1013 ) × (1.6 × 10−19 ) = 1.824 µ A
P 720
⇒I= = = 0.018Å
0.995 V 0.995 × 40 × 103 Example 4: Consider the following data: Incident beam:
wavelength 3650 Å; intensity 10−8 W/m2. Surface:
Number of electrons per second incident of the target
Absorption coefficient 0.8; work function 1.6 eV.
I 0.018 Determine the time rate of number of electrons emitted
n= = = 1.1 × 1017 electrons/s.
e 1.6 × 10−19 per m2 , power absorbed per m2 , and the maximum
kinetic energy of emitted photoelectrons.
2 4 . 4 6 | Modern Physics

hc First line of Balmer series corresponds to transition


Sol: The energy of one photon of light is E = . from orbit n = 3 to orbit n = 2. Energy emitted is
λ
Number of photons incident on the surface per m2 1 1
per second is the intensity divided by energy of one ∆E = Z 2 × 13.6 ×  −  . The momentum imparted to
photon. Number of photons × absorption coefficient
4 9
= the number of photons absorbed by the surface. the heavy target during elastic collision is twice the
The remaining number of photons is equal to the momentum of the striking particle.
photoelectrons emitted per m2 per second. Suppose ZA and ZB are the atomic number and mA and
If N is the number of photons crossing per unit area per mB are the mass numbers of hydrogen like atoms A and
unit time, B, respectively.

Number of photons falling per second on unit area Z 2Rhc −Z 2 × 13.6


En =− = eV
n2 n2
Intensity Iλ Energy emitted for first Balmer line of atom A
= =
Energy of one photon hc
 1 1 
−8 −10
∆EA =−Z 2A × 13.6  2 − 2  eV
10 × 3650 × 10 2 3 
= = 18.35 × 109 / m2s
6.62 × 10−34 × 3 × 108 Similarly, energy emitted for first Balmer line of atom B
The number of photons absorbed Nab by the surface  1 1 
∆EB =−ZB2 × 13.6 ×  −  eV
per unit area per unit time 2 2
32 
Nab = absorption coefficient of surface × N According to question, ∆EA − ∆EB = 5.667 eV
9 10 2
= 0.8 × 18.35 × 10 = 1.47 × 10 / m s
 1 1 
Now, assuming that each photon ejects one electron, or 5.667 eV = (ZB2 − Z 2A ) × 13.6  2 − 2  eV
2 3 
the rate of electrons emitted per unit area is given by
5.667 × 36
N − Nab = 1.835 × 1010 − 1.47 × 1010 = 0.37 × 1010 / m2 − s or ZB2=
− Z 2A = 3 ... (i)
13.6 × 5
Power absorbed/ m2
Suppose you represent the initial velocity of each atom
= Absorption coefficient × Intensity of light falling A and B as u.
on surface= 0.8 × 10−8 = 8 × 10−9 W/ m2 . Maximum
Momentum imparted by A to target = 2mAu
kinetic energy is of emitted photoelectron is given by
hc Momentum imparted by B to target = 2mBu
(K.E.)max = −φ
λ Then according to questions,
(6.62 × 10−34 )(3 × 108 ) 2mAu = 2mBu ⇒ 2mA = mB ... (ii)
= eV–1.6eV 
−10 −19
3650 × 10 × 1.6 × 10 In case of both the atoms A and B, number of protons
and neutrons is same separately, hence mB = 2ZB and
= 3.4 eV – 1.6 eV = 1.80 eV mA = 2Z A
Putting mA and mB in equation (ii)
Example 5: Consider two hydrogen-like atoms A and
B of different masses but having equal number of 2ZB = 2(2Z A ) or ZB = 2Z A  ... (iii)
protons and neutrons. The difference in the energies Solving (i) and (iii) Z A = 1 and ZB = 2
between the first Balmer lines emitted by A and B is
5.667 eV. When these atoms, moving with the same i.e., atom A contains 1 proton and 1 neutron, i.e., atom
velocity, strike a heavy target elastically, the atom B A is deuterium (1 H2 ) .
imparts twice the momentum to the target than the Similarly, atom B contains 2 protons and 2 neutrons,
atom A. Identify the atoms A and B. i.e., atom B is singly ionized Helium.

Sol: The energy of hydrogen-like atom for nth orbit is


Example 6: A traveling hydrogen atom in the ground
Z 2 × 13.6 state makes a head-on inelastic collision with a
given by E = − , where Z = atomic number.
n2 stationary hydrogen atom in the ground state. After
collision, they move together. What is the minimum
P hysi cs | 24.47

velocity of the traveling hydrogen atom if one of the


4 × 16.32 × 10−19
atoms is to gain the minimum excitation energy after u2 = = 39.02 × 108
the collisions? 1.0078 × 1.66 × 10−27

u 6.246 × 10 4 m / s
⇒=
Sol: Here we need to consider that the kinetic energy
lost in the inelastic collision will be absorbed by one of
the hydrogen atoms to reach to its next excited state. Example 7: Assuming the potential energy between
As both the hydrogen atoms are initially in ground state
ke2
(n=1), the minimum energy absorbed will be equal to electron and proton at a distance r to be U = ,
that required by one of the atoms to reach the first 3r3
excited state (n=2). If the kinetic energy of the colliding use Bohr’s theory to obtain energy levels of such a
hydrogen atom is less than this minimum energy, no hypothetical atom.
energy will be absorbed, i.e. inelastic collision may not
take place. Sol: The negative of gradient of potential energy
is equal to force on the electron. This force provides
Let u be the velocity of the hydrogen atom before the necessary centripetal acceleration to the electron
collision and v the velocity of the two atoms moving to move in a circular orbit around the proton. The
together after collision. By the principle of conservation magnitude of angular momentum of electron is
of momentum, we have: Mu + M × 0 = 2Mv quantized. The mass of the proton is very large as
compared to the mass of electron, so it will not be
u
or v = . The loss in kinetic energy ∆E due to collision accelerated due to the force exerted on it by the
2
1 1 electron, hence it is assumed to be stationary.
is given by =∆E Mu2 − (2M)v 2
2 2 As we know that negative of potential energy gradient
u is force for a conservative field.
As v =
2 dU ke2
2 − = F. It is given that U =  … (i)
1 1 u dr 3r3
∆E
we have = Mu2 − (2M)  
2 2 2 dU d  ke2  ke2
Hence, force F = − = −   =
=
1 1 1
Mu2 − Mu2 = Mu2 dr dr  3r3  r4
2 4 4
According to Bohr’s theory this force provides the
This loss in energy is due to the excitation of one of the
necessary centripetal force for orbital motion.
hydrogen atoms. The ground state (n = 1) energy of a
hydrogen atom is: ke2 mv 2
=  … (ii)
r4 r
E1 = −13.6eV
Also quantizing angular momentum,
The energy of the first excited level (n = 2) is:
nh
E2 = −3.4 eV mvr =  ... (iii)

nh
Thus the minimum energy required to excite a hydrogen Hence, v =  ... (iv)
atom from ground state to first excited state is: E2 − E1 2πmr
= [– 3.4 – (–13.6)] eV = 10.2 eV = 10.2 × 1.6 × 10−19 J Substituting this value in Eq.(ii), we get
= 16.32 × 10−19 J mn2h2 ke2 4 π2me2k
= or r =
As per problem, the loss in kinetic energy in collision is 4 π2m2r3 r4 n2h2
due to the energy used up in exciting one of the atoms.
Thus. ∆E = E2 − E1 Substituting this value or r in Eq. (iv), we get

1 n3h3
Or Mu2 = 16.32 × 10−19 v=
4 8π3km2e2
4 × 16.32 × 10−19 Total energy E = KE + PE
Or u2 =
M
The mass of the hydrogen atom is 1.0078 amu or 1.0078 1 ke2
× 1.66 × 10−27 kg = mv 2 −
2 3r3
2 4 . 4 8 | Modern Physics

2 3 Example 9: Assuming that the short series limit of the


m  n3h3  ke2  n2h2  Balmer series for hydrogen is 3646 Å, calculate the
=   −  
2  8π3km2e2  3  4ke2mπ2  atomic number of the element, given X-ray wavelength
down to 1.0 A. Identify the element.
(n )6  h
= where   = 
2 2
6(ke ) m 3
 2π  Sol: Balmer series spectra is obtained when an electron
transitions from higher energy orbit to the second orbit
Example 8: When an electron in a tungsten (Z = 74) (n =2). The wave number of radiation emitted is given
target drops from an M shell to a vacancy in the K shell, 1  1 1 
calculate the wavelength of the characteristic X-ray as ν= = R −  . The shortest wavelength will
λ  n 2 n2 
emitted there of.  f i 
correspond to highest energy, i.e. n = ∞.
Sol: In multi electron atoms, the nucleus is shielded The short limit of the Balmer series is given by
from the outer most electron by the inner shell electron
such that the outer most electron experience Zeff charge 1  1 1  R
ν= = R − =
from nucleus. The energy of this outermost electron in λ 2 2
∞2  4
13.6 × Z eff2
nth shell is E = − 4  4  10 −1
n2 ∴ R= =   × 10 m Further the wavelengths
λ  3646 
Tungsten is a multi-electron atom. Due to the shielding
of the nuclear charge by negative charge of the inner of the k a series are given by the relation
core electrons, each electron is subjected to an effective 1 1 1 
nuclear charge Z eff which is different for different shells. ν= = R(Z − 1)2  − 
2
λ 1 n2 
Thus, the energy of an electron in the nth level of a
multi-electron atom is given by
The maximum wave number correspondence to n = ∞
13.6Z 2eff and, therefore, we must have
En = − eV For an electron in the K shell (n = 1),
n2 1
Z eff = (Z – 1) ν= = R(Z − 1)2
λ
Thus, the energy of the electron in the K shell is: 1 3646 × 10−10
Or (Z − 1)2 = =
(74 − 1)2 × 13.6 Rλ 4 × 1 × 10−10
EK = − ≈ −72500 eV
12
For an electron in the M shell (n = 3), the nucleus is = 911.5 (Z – 1) = 911.5 ≅ 30.2
shielded by one electron of the n = 1 state and eight
Or Z = 30.2 ≅ 31
electrons of the n = 2 state, a total of nine electrons, so
that Z eff= Z − 9 Thus, the energy of an electron in the Thus, the atomic number of the element concerned is
M shell is: 31.
(74 − 9)2 × 13.6 The element having atomic number Z = 30 is Gallium.
EM = − ≈ −6380 eV
2
3
Therefore, the emitted X-ray photon has an energy
given by Hv = EM − EK
=–6380 eV −( −72500eV) = 66100 eV
hc
Or = 66100 × 1.6 × 10−19 J
λ
hc
∴ λ= m
66100 × 1.6 × 10−19

(6.63 × 10−34 ) × (3 × 108 )


= m =0.0188 × 10−9 m.
66100 × 1.6 × 10−19
P hysi cs | 24.49

JEE Main/Boards

Exercise 1 Q.7 Draw a schematic diagram of the experimental


arrangement used by Davisson and Germer to establish
the wave nature of electrons. Explain briefly how the
Q.1 Define the terms: (i) work function, (ii) threshold
de-Broglie relation was experimentally verified in case
frequency and (iii) stopping potential, with reference to
of electrons.
photoelectric effect.
Calculate the maximum Kinetic energy of electron
emitted from a photosensitive surface of work function Q.8 Two lines, A and B, in the plot given below show the
3.2 eV, for the incident radiation of wavelength 300nm. 1
variation of de-Broglie wavelength, λ , versus ,
V
Q.2 Derive the expression for the de Broglie wavelength where V is the accelerating B
of an electron moving under a potential difference of V
volt. potential difference. For two
particles carrying the same 
A
Describe Davisson and Germer experiment to establish charge, which one of the two
the wave nature of electrons. Draw a labelled diagram represents a particle of smaller 1/V
of the apparatus used. mass?

Q.3 Two metals A and B have work functions 2eV and Q.9 The following graphs
5eV respectively. Which metal has lower threshold shows the variation of P Q
wavelength? stopping potential V0 with V0
the frequency v of the
Q.4 de-Broglie wavelength associated with an electron incident radiation for two
accelerated through a potential difference V is λ . What photosensitive metals P 0.5 1.0 (x1015s-1)
v
will be its wavelength when the accelerating potential and Q:
is increased to 4 V? (i) Explain which metal has smaller threshold wavelength.
(ii) Explain, giving reason, which metal emits
Q.5 Sketch a graph between frequency of incident photoelectron having smaller kinetic energy.
radiations and stopping potential for a given
photosensitive material. What information can be (iii) If the distance between the light source and metal
obtained from the value of the intercept on the P is doubled, how will the stopping potential change?
potential axis?
A source of light of frequency greater than the Q.10 The stopping potential in an experiment on
threshold frequency is placed at a distance of 1 m from photoelectric effect is 1.5V. What is the maximum
the cathode of a photo-cell. The stopping potential is kinetic energy of the photoelectrons emitted?
found to be V. If the distance of the light source from
the cathode is reduced, explain giving reasons, what Q.11 An α -Particles and a proton are accelerated from
change will you observe in the rest by the same potential. Find the ratio of their de-
(i) Photoelectric current Broglie wavelengths.

(ii) Stopping potential


Q.12 Write Einstein’s photoelectric equation.
State clearly the three salient features observed in
Q.6 Ultraviolet radiations of different frequencies v1 photoelectric effect, which can be explained on the
and v 2 are incident on two photosensitive materials basis of the above equation.
having work functions W1 and W2 ( (W1 > W2 )
respectively. The Kinetic energy of the emitted electron
Q.13 Define the term ‘stopping potential’ in relation to
is same in both cases. Which one of the two radiations
photoelectric effect.
will be of higher frequency?
2 4 . 5 0 | Modern Physics

Q.14 Draw a plot showing the variation of photoelectric Q.23 The ground state energy of hydrogen atom is
current with collector plate potential for two different –13.6eV. What are the kinetic and potential energies of
frequencies, v1 > v 2 ,of incident radiation having the electron in this state?
same intensity. In which case will the stopping potential
be higher? Justify your answer. Q.24 In a Geiger-Marsden experiment, calculate the
distance of closest approach to the nucleus of Z = 80,
Q.15 A proton and an electron have same kinetic when an α -particle of 8 MeV energy impinges on it
energy. Which one has greater de-Broglie wavelength before it comes momentarily to rest and reverse its
and why? direction.
How will the distance of closest approach be affected
Q.16 Define the terms (i) ’cut-off voltage’ and (ii) when the kinetic energy of the α -particle is doubled?
’threshold frequency’ in relation to the phenomenon of
photoelectric effect.
Q.25 A photon and electron have got the same de-
Using Einstein’s photoelectric equation show how the Broglie wavelength. Which has the greater total energy?
cut-off voltage and threshold frequency for a given Explain.
photosensitive material can be determined with the
help of a suitable plot/graph. Q.26 If the intensity of incident radiation of a metal
surface is doubled, what happens to the kinetic energy
Q.17 Derive the expression for the radius of the ground of the electrons emitted?
state orbit of hydrogen atom, using Bohr’s postulates.
Calculate the frequency of the photon, which can excite Q.27 The wavelength of a spectral line is 4000 Å.
the electron to –3.4 eV from –13.6 eV. Calculate its frequency and energy. Given,
c = 3 × 108 ms−1 and =
h 6.6 × 10−34 Js .
Q.18 A stream of electrons travelling with speed ‘v’ m/s
at right angles to a uniform electric field ‘E’, is deflected Q.28 Calculate the longest wavelength of the incident
e v2 radiation, which will eject photoelectrons from a metal
in a circular path of radius ‘r’. Prove that = .
m rE surface, whose work function is 3 eV.

Q.19 In a hydrogen atom, an electron of change ‘e’


revolves in a orbit of radius ‘r’ with a speed ‘v’. Prove Exercise 2
that the magnetic moment associated with the electron
evr Single Correct Choice Type
is given by .
2
Q.1 Let nr and nb be respectively the number of
Q.20 Draw a labeled diagram of experimental setup of
photons emitted by a red bulb and a blue bulb of equal
Rutherford’s alpha particle scattering experiment. Write
power in a given time.
two important inferences drawn from this experiment.
(A) nr = nb (B) nr < nb
Q.21 The ground state energy of hydrogen atom is (C) nr > nb (D) Data insufficient
–13.6eV.
(i) What is the potential energy of an electron in the 3rd Q.2 In a photo-emissive cell, with exciting wavelength
excited state? λ , the maximum kinetic energy of electron is K. If
(ii) If the electron jumps to the ground state from third 3λ
the exciting wavelength is changed to the Kinetic
excited state, calculate the wavelength of the photon 4
emitted. energy of the fastest emitted electron will be:
3K 4K
(A) (B)
Q.22 Drawn a schematic arrangement of the Geiger– 4 3
Marsden experiment. How did the scattering of α 4K 4K
-particle by a thin foil of gold provide an important way (C) Less than (D) Greater than
3 3
to determine an upper limit on the size of the nucleus?
Explain briefly.
P hysi cs | 24.51

Q.3 If the frequency of light in a photoelectric Q.8 By increasing the intensity of incident light keeping
experiment is doubled, the stopping potential will frequency (v > v 0 ) fixed, on the surface of metal
(A) Be doubled (A) Kinetic energy of the photoelectrons increase
(B) Be halved (B) Number of emitted electrons increases
(C) Become more than doubled (C) Kinetic energy and number of electrons increase
(D) Become less than doubled (D) No effect

Q.4 The stopping potential for the photoelectron Q.9 A proton and an electron accelerated by same
emitted from a metal surface of work function 1.7 eV is potential difference have de-Broglie wavelength λp
10.4 V. Identify the energy levels corresponding to the and λe
transition in hydrogen atom which will result in emission
(A) λe =λp (B) λe < λp
of wavelength equal to that of incident radiation for the
above photoelectric effect. (C) λe > λp (D) None of these
(A) n = 3 to 1 (B) n = 3 to 2
Q.10 An electron with initial kinetic energy of 100eV is
(C) n = 2 to 1 (D) n = 4 to 1 accelerated through a potential difference of 50V. Now
the de-Broglie wavelength of electron becomes
Q.5 Radiation of two photon energies twice and
(A) 1Å (B) 1.5 Å (C) 3Å (D) 12.27Å
five times the work functions of metal are incident
successively on the metal surface. The ratio of the
maximum velocity of photoelectrons emitted is the two Q.11 If h is Planck’s constant in SI system, the
cases is momentum of a photon of wavelength 0.01 Å is:

(A) 1 : 2 (B) 2 : 1 (C) 1 : 4 (D) 4 : 1 (A) 10−2 h (B) h (C) 102 h (D) 1012 h

Q.6 Cut off potentials for a metal in photoelectric effect Q.12 Let K1 be the maximum kinetic energy of
for light of wavelength λ1 , λ2 and λ3 is found to be photoelectrons emitted by a light of wavelength λ1
V1 , V2 and V3 volts if V1 , V2 and V3 are in Arithmetic and K 2 corresponding to λ2 . If λ1 = 2λ2 , then:
Progression and λ1 , λ2 and λ3 will be: K
(A) 2K1 = K 2 (B) K1 = 2K 2 (C) K1 < 2 (D) K1 > 2K 2
2
(A) Arithmetic Progression
(B) Geometric Progression Q.13 Imagine a Young’s double slit interference
experiment performed with waves associated with fast
(C) Harmonic Progression moving electrons produced from an electron gun. The
(D) None distance between successive maxima will decrease
maximum if
Q.7 In a photoelectric experiment, the collector plate (A) The accelerating voltage in the electron gun is
is at 2.0V with respect to the emitter plate made of decreased
copper φ=4.5eV). The emitter is illuminated by a source (B) The accelerating voltage is increased and the
of monochromatic light of wavelength 200nm. distance of the screen from the slits is decreased
(A) The minimum kinetic energy of the photoelectrons (C) The distance of the screen from the slits is increased
reaching the collector is 0.
(D) The distance between the slits is decreased.
(B) The maximum kinetic energy of the photoelectrons
reaching the collector is 3.7ev.
Q.14 If the electron in a hydrogen atom was in the
(C) If the polarity of the battery is reversed then answer energy level with n = 3, how much energy in joule would
to part A will be 0. be required to ionize the atom? (Ionization energy of
(D) If the polarity of the battery is reversed then answer H-atom is 2.18 × 10−18 J):
to part B will be 1.7eV. (A) 6.52 × 10–16 J (B) 2.86 × 10–10 J
(C) 2.42 × 10–19 J (D) 3.56 × 10–19 J
2 4 . 5 2 | Modern Physics

Q.15 In hydrogen and hydrogen like atoms, the ratio of theory of hydrogen atom,
difference of energies E4n − E2n and E2n − En varies with
(A) fr 2L is constant for all orbits
its atomic number z and n as:
(B) frL is constant for all orbits
z2 z4 z
(A) (B) (C) (D) z 0n0
n2 n4 n (C) f 2rL is constant for all orbits
(D) frL2 is constant for all orbits
Q.16 In a hydrogen atom, the electron is in nth excited
state. It may come down to second excited state by
Q.22 Radius of the second Bohr orbit of singly ionized
emitting ten different wavelengths. What is the value
helium atom is
of n?
(A) 0.53 Å (B) 1.06 Å (C) 0.265 Å (D) 0.132 Å
(A) 6 (B) 7 (C) 8 (D) 5

Q.23 An electron in Bohr’s hydrogen atom has an energy


Q.17 Monochromatic radiation of wavelength λ
of –3.4 eV. The angular momentum of the electron is
is incident on a hydrogen sample in ground state.
Hydrogen atoms absorb the light and subsequently h h
(A) (B)
emit radiations of ten different wavelengths. The value π 2π
of λ is nh 2h
(C) (n is an integer) (D)
(A) 95nm (B) 103nm (C) 73nm (D) 88nm 2π π

Q.18 In a sample of hydrogen like atoms all of which Q.24 An electron is in an excited state in hydrogen-like
are in ground state, a photon beam containing photons atom. It has a total energy of –3.4eV. If the kinetic energy
of various energies is passed. In absorption spectrum, of the electron is E and its de-Broglie wavelength is λ ,
five dark lines are observed. The number of bright then
lines in the emission spectrum will be (Assume that all (A) E = 6.8eV, λ =6.6 × 10−10 m
transitions take place)
(B) E = 3.4eV, λ = 6.6 x 10−10 m
(A) 5 (B) 10 (C) 15 (D) None of these
(C) E = 3.4eV, λ = 6.6 x 10−11 m

Q.19 When a hydrogen atom, initially at rest emits, a (D) E = 6.8eV, λ = 6.6 x 10−11 m
photon resulting in transition n = 5 → n = 1, its recoil
speed is about Q.25 If radiation of all wavelengths from ultraviolet
−4 −2 to infrared is passed through hydrogen a gas at room
(A) 10 m/s (B) 2 × 10 m/s
temperature, absorption lines will be observed in the:
−2
(C) 4.2 m / s (D) 3.8 × 10 m/s
(A) Lyman series (B) Balmer series

Q.20 The electron in a hydrogen atom makes a (C) Both (A) and (B) (D) Neither (A) nor (B)
transition from an excited state to the ground state.
Which of the following statement is true? Q.26 In the hydrogen atom, if the reference level of
potential energy is assumed to be zero at the ground
(A) Its kinetic energy increases and its potential and
state level, choose the incorrect statement.
total energies decrease.
(A) The total energy of the shell increases with increase
(B) Its kinetic energy decreases, potential energy
in the value of n.
increases and its total energy remains the same.
(B) The total energy of the shell decrease with increase
(C) Its kinetic, and total energies decrease and its
in the value of n.
potential energy increases.
(C) The difference in total energy of any two shells
(D) Its kinetic, potential and total energies decrease.
remains the same.

Q.21 The magnitude of angular momentum, orbit (D) The total energy at the ground state becomes
radius and frequency of revolution of electron in 13.6 eV.
hydrogen atom corresponding to quantum number
n are L, r and respectively. Then according to Bohr’s
P hysi cs | 24.53

Q.27 Choose the correct statement(s) for hydrogen and Q.3 If elements with principal quantum number n >
deuterium atoms (considering motion of nucleus) 4 were not allowed in nature, the number of possible
elements would be  (1983)
(A) The radius of first Bohr orbit of deuterium is less
than that of hydrogen (A) 60 (B) 32 (C) 4 (D) 64
(B) The speed of electron in the first Bohr orbit of
deuterium is more than that of hydrogen. Q.4 Consider the spectral line resulting from the
transition n = 2 → n = 1 in the atoms and ions given
(C) The wavelength of first Balmer line of deuterium is
below. The shortest wavelength is produced by (1983)
more than that of hydrogen
(A) Hydrogen atom
(D) The angular momentum of electron in the first Bohr
orbit of deuterium is more than that of hydrogen. (B) Deuterium atom
(C) Singly ionized helium
Q.28 In a Coolidge tube experiment, the minimum
wavelength of the continuous X-ray spectrum is equal (D) Doubly ionized lithium
to 66.3 pm, then 4
Q.5 Equation: 411 H →2 He2+ + 2e− + 26 MeV
(A) Electron accelerate through a potential difference of
12.75 kV in the Coolidge tube represents (1983)
(B) Electrons accelerate through a potential difference (A) b – decay (B) g – decay
of 18.75 kV in the Coolidge tube
(C) Fusion (D) Fission
(C) de-Broglie wavelength of the electrons reaching the
anticathode is of the order of 10µm. Q.6 For a given plate voltage, the plate current in a
(D) de-Broglie wavelength of the electrons reaching the triode valve is maximum when the potential of (1985)
anticathode is 0.01Å. (A) The grid is positive and plate is negative.

Q.29 The potential difference applied to an X-ray tube (B) The grid is zero and plate is positive.
is increased. As a result, in the emitted radiation: (C) The grid is negative and plate is positive
(A) The intensity increases (D) The grid is positive and plate is positive
(B) The minimum wave length increases
Q.7 The X-ray beam coming from an X-ray tube will be
(C) The intensity decreases
 (1985)
(D) The minimum wave length decreases
(A) Monochromatic
(B) Having all wavelengths smaller than a certain
Previous Years’ Questions maximum wavelength

Q.1 The shortest wavelength of X-rays emitted from an (C) Having all wavelengths larger than a certain
X-ray tube depends on  (1982) minimum wavelength

(A) The current in the tube (D) Having all wavelengths lying between a minimum
and a maximum wavelength
(B) The voltage applied to the tube
(C) The nature of the gas in tube Q.8 Statement-I: If the accelerating potential in
(D) The atomic number of the target material an X-ray tube is increased, the wavelengths of the
characteristic X-rays do not change.
Q.2 Beta rays emitted by a radioactive material are Statement-II: When an electron beam strikes the target
 (1983) in an X-ray tube, part of the kinetic energy is converted
(A) Electromagnetic radiations into X-ray energy.  (2007)

(B) The electrons orbiting around the nucleus (A) If Statement-I is true, statement-II is true; statement-
II is the correct explanation for statement-I.
(C) Charged particles emitted by the nucleus
(B) If Statement-I is true, statement-II is true; statement-
(D) Neutral particles II is not a correct explanation for statement-I.
2 4 . 5 4 | Modern Physics

(C) If statement-I is true; statement-II is false. Q.14 If a strong diffraction peak is observed when
electrons are incident at an angle ‘i’ from the normal to
(D) If statement-I is false; statement-II is true.
the crystal planes with distance ‘d’ between them (see
figure), de Broglie wavelength λdB of electrons can be
Q.9 To produce characteristic X-rays using a tungsten calculated by the relationship (n is an integer)  (2008)
target in an X-ray generator, the accelerating voltage
should be greater than…… V and the energy of the (A) d sin i = nλdB (B) 2d cos i = nλdB
characteristic radiation is ………. eV. (1983) (C) 2d sin i = nλdB (D) d cos i = nλdB
(The binding energy of the innermost electron in
tungsten is 40 keV). Q.15 In an experiment, electrons are made to pass
through a narrow slit of width ‘d’ comparable to their
Q.10 The radioactive decay rate of a radioactive element de Broglie wavelength. They are detected on a screen
is found to be 103 disintegration/second at a certain at a distance ‘D’ from the slit (see figure).  (2008)
time. If the half-life of the element is one second, the
decay rate after one second is ………… and after three
seconds is………  (1983)
y=0
d
Q.11 The maximum kinetic energy of electrons emitted D
in the photoelectric effect is linearly dependent on the
……….. of the incident radiation. (1984)
Which of the following graph can be expected to
Q.12 In the uranium radioactive series the initial represent the number of electrons ‘N’ detected as a
nucleus is 238 206 function of the detector position ‘y’(y = 0 corresponds
92 U and the final nucleus is 82 Pb . When
the uranium nucleus decays to lead, the number to the middle of the slit)?
y
of α -particles emitted is ………. and the number of y
β -particles emitted is………. (1985)
(A) d (B) d
N N
Directions : Q.13, Q.14 and Q.15 are based on the
following paragraph.
y y
Wave property of electrons implies that they will show
diffraction effects. Davisson and Germer demonstrated
this by diffracting electrons from crystals. The law (C) d (D) d
N
governing the diffraction from a crystal is obtained by N
requiring that electron waves reflected from the planes
of atoms in a crystal interfere constructively (see in
figure). Q.16 Two points P and Q are maintained at the
potentials of 10V and -4V respectively. The work done
Incoming
Electrons
Outgoing in moving 100 electrons from P to Q is  (2009)
Electrons
i
(A) –19×10–17 J (B) 9.60×10–17 J

d (C) –2.24×10–16 J (D) 2.24×10–16 J

Q.17 The surface of a metal is illuminated with the light


Crystal plane of 400 nm. The kinetic energy of the ejected photo
electrons was found to be 1.68 eV. The work function of
Q.13 Electrons accelerated by potential V are diffracted the metal is (hc = 1240 eV nm)  (2009)
from a crystal. If d = 1Å and i = 30°, V should be about (A) 3.09 eV (B) 1.41 eV
(h = 6.6 ×10−34 Js, me= 9.1 ×10−31 kg, e = 1.6 ×10−19 C)
(C) 151 eV (D) 1.68 ev
 (2008)
(A) 2000 V (B) 50 V (C) 500 V (D) 1000 V
Q.18 Statement-I: When ultraviolet light is incident
on a photocell, its stopping potential is V0 and the
P hysi cs | 24.55

maximum kinetic energy of the photoelectrons is Kmax. Statement-II: If electrons have wave nature, they can
When the ultraviolet light is replaced by X-rays, both V0 interfere and show diffraction.  (2012)
and Kmax increase.
(A) Statement-I is false, statement-II is true
Statement-II: Photoelectrons are emitted with speeds
(B) Statement-I is true, statement-II is false
ranging from zero to a maximum value because of
the range of frequencies present in the incident light. (C) Statement-I is true, statement-II is the correct
 (2010) explanation for statement-I
(A) Statement-I is true, statement-II is true; statement-II (D) Statement-I is true, statement-II is true, statement-II
is the correct explanation of statement-I. is not the correct explanation for statement-I.
(B) Statement-I is true, statement-II is true; statement-II
is not the correct explanation of statement-I. Q.22 A diatomic molecule is made of two masses
m1 and m2 which are separated by a distance r. If we
(C) Statement-I is false, statement-II is true. calculate its rotational energy by applying Bohr’s rule
(D) Statement-I is true, statement-II is false. of angular momentum quantization, its energy will be
given by (n is an integer) (2012)
Q.19 If a source of power 4 kW produces 1020 photons/ (m1 + m2 )2 n2h2 n2h2
second, the radiation belong to a part of the spectrum (A) (B)
called  (2010) 2m12m22r 2 2(m1 + m2 )r 2

(A) X–rays (B) Ultraviolet rays 2n2h2 (m1 + m2 )n2h2


(C) (D)
(C) Microwaves (D) γ – rays (m1 + m2 )r 2 2m1m2r 2

Q.20 This question has Statement-I and Statement-II. Q.23 The anode voltage of a photocellis kept fixed.
Of the four choices given after the statements, choose The wavelength λ of the light falling on the cathode is
the one that best describes the two statements. gradually changed. The plate current I of the photocell
varies as follows :  (2013)
Statement-I: A metallic surface is irradiated by a
monochromatic light of frequency v > v0 (the threshold I I
frequency). The maximum kinetic energy and the
stopping potential are Kmax and V0 respectively. If the (A) (B)
frequency incident on the surface doubled, both the
Kmax and V0 are also doubled (2011)
O  O 

Statement-II: The maximum kinetic energy and the I


I
stopping potential of photoelectrons emitted from
a surface are linearly dependent on the frequency of
(C) (D)
incident light.
(A) Statement-I is true, statement-II is true; statement-II
is the correct explanation of statement-I. O  O 

(B) Statement-I is true, statement-II is true; statement-II Q.24 In a hydrogen like atom electron make transition
is not the correct explanation of statement-I. from an energy level with quantum number n to another
with quantum number (n – 1). If n>>1, the frequency of
(C) Statement-I is false, statement-II is true.
radiation emitted is proportional to : (2013)
(D) Statement-I is true, statement-II is false.
1 1 1 1
(A) (B) (C) (D)
n n 2 3
n n3
Q.21 This question has statement-I and statement-II. Of
2
the four choices given after the statements, choose the
one that best describes the two statements Q.25 The radiation corresponding to 3 → 2 transition
of hydrogen atoms falls on a metal surface to produce
Statement-I: Davisson – germer experiment established
photoelectrons. These electrons are made to enter a
the wave nature of electrons.
magnetic field of 3 × 10–4 T. If the radius of the largest
circular path followed by these electrons is 10.0 mm,
2 4 . 5 6 | Modern Physics

the work function of the metal is close to  (2014)


(iii) Davison, Germer (r) Wave nature of
(A) 1.8 eV (B) 1.1 eV (C) 0.8 eV (D) 1.6 eV experiment electron
(s) Structure of atom
Q.26 Hydrogen (1H1), Deuterium (1H2), singly ionised
Helium (2He4)+ and doubly ionised lithium (3Li6)++ all have (A) (i) → (p), (ii) → (s), (iii) → (r)
one electron around the nucleus. Consider an electron
(B) (i) → (q), (ii) → (s), (iii) → (r)
transition from n= 2 to n= 1. If the wave lengths of
emitted radiation are λ1, λ2, λ3 and λ4 respectively then (C) (i) → (q), (ii) → (p), (iii) → (r)
approximately which one of the following is correct?
(D) (i) → (s), (ii) → (r), (iii) → (q)
 (2014)
(A) 4 λ1 = 2λ2 = 2λ3 = λ4 (B) λ1 = 2λ2 = 2λ3 = λ4 Q.28 Radiation of wavelength λ , is incident on a
(C) λ1 = λ2 = 4λ3 = 9λ4 (D) λ1 = 2λ2 = 3λ3 = 4λ4
photocell. The fastest emitted electron has speed v If

Q.27 Match List-I (Fundamental Experiment) with List-II the wavelength is changed to , the speed of the
4
(its conclusion) and select the correct option from the
fastest emitted electron will be :  (2016)
choices given below the list : (2015)
1 1
 4 2  4 2
List - I List - II (A) < v   (B) = v  
3 3
(i) Franck-Hertz (p) Particle nature of
1 1
experiment light  3 2  4 2
(C) = v   (D) > v  
(ii) Photo-electric (q) Discrete energy 4 3
experiment levels of atom

JEE Advanced/Boards

Exercise 1 collector. In the same setup, light of wavelength 220


nm, ejects electron which require twice the voltage
V0 to stop them in reaching a collector. Find the
Q.1 When a monochromatic point source of light is at
numerical value of voltage V0 . (Take plank’s constant,
a distance of 0.2 m from a photoelectric cell, the cut off
h = 6.6 × 10−34 J s and 1 eV = 1.6 × 10−19 J)
voltage and the saturation current are respectively 0.6
V and 18.0 mA. If the same source is placed 0.6 m away
from the photoelectric cell, then find Q.4 A small 10W source of ultraviolet light of Wavelength
99 nm is held at a distance 0.1m from a metal surface.
(a) The stopping potential The radius of an atom of the metal is approximately
(b) The saturation current 0.05 nm. Find
(i) The average number of photons striking an atom per
Q.2 663 mW of light from of 540 nm source is incident second.
on the surface of a metal. If only 1 of each 5 x 109
(ii) The number of photoelectrons emitted per unit
incidents photons is absorbed and causes an electron
area per second if the efficiency of liberation of
to be ejected from the surface, the total photocurrent
photoelectrons is 1%.
in the circuit is ________.

Q.5 The surface of cesium is illuminated with


Q.3 Light of Wavelength 330 nm falling on a piece
monochromatic light of various wavelengths and the
of metal ejects electrons with sufficient energy which
stopping potentials for the wavelengths are measured.
requires voltage V0 to prevent a electron from reading
The results of this experiment is plotted as shown in
P hysi cs | 24.57

the figure. Estimate the value of work function of the Q.10 In a photoelectric effect set-up, a point source
cesium and Planck’s constant. of light of power 3.2 × 10−3 W emits mono energetic
photons of energy 5.0 eV. The source is located at a
distance of 0.8 m from the center of a stationary metallic
sphere of work function 3.0 eV & of radius 8.0 × 10−3 .
Stopping potential (volt)

The efficiency of photoelectrons emission is one for


2 0.49 every 106 incident photons. Assume that the sphere is
1 isolated and initially neutral, and that photoelectrons
15
0 v x10 Hz are instantly swept away after emission.
-1 0.5 1.0 1.5
-2 (a) Calculate the number of photoelectrons emitted per
seconds.
(b) Find the ratio of the wavelength of incident light to
the de-Broglie wave length of the fastest photoelectrons
Q.6 A small plate of a metal (work function = 1.17 eV) is emitted.
placed at a distance of 2m from a monochromatic light
(c) It is observed that the photoelectron emissions stops
source of wave length 4.8 × 10−7 m power 1.0 watt. The
at a certain time t after the light source is switched on.
light falls normally on the plate. Find the number of
Why?
photons striking the metal plate per square meter per
second. If a constant uniform magnetic field of strength (d) Evaluate the time t.
10−4 tesla is applied parallel to the metal surface, find
the radius of the largest circular path followed by the Q.11 When photons of energy 4.25eV strike the surface
emitted photoelectrons. of a metal A, the ejected photoelectrons have maximum
kinetic energy Ta eV and de Broglie wavelength λa . The
Q.7 Electrons in hydrogen like atoms (Z = 3) make maximum kinetic energy of photoelectrons liberated
transition from the fifth to the fourth orbit & from from another metal B by photons of energy 4.7eV is
the fourth to the third orbit. The resulting radiations Tb = ( Ta – 1.5) eV. If the de Broglie wavelength of these
are incident normally on a metal plate & eject photoelectrons is λb = 2λa , then find
photoelectrons. The stopping potential for the
(a) The work function of a
photoelectrons ejected by the shorter wavelength is
3.95 V. Calculate the work function of the metal, & the (b) The work function of b
stopping potential for the photoelectrons ejected by (c) Ta and Tb
the longer wavelength. (Rydberg constant = 1.094 ×
107 m−1 ).
Q.12 An electron of mass “m” and charge “e” initially at
rest gets accelerated by a constant electric field E. The
Q.8 A beam of light has three wavelength 4144 Å, 4972 rate of change of de Broglie wavelength of this electron
Å & 6216Å with a total intensity of 3.6 × 10−3 W.m−2 at time t is……………….
equally distributed amongst the three wavelengths.
The beam falls normally on an area 1.0 cm2 of a clean
metallic surface of work function 2.3 eV. Assume that Q.13 A hydrogen atom in a state having a binding
there is no loss of light by reflection and that each energy 0.85eV makes a transition to a state of excitation
energetically capable photon ejects one electron. energy 10.2eV. The wave length of emitted photon is
Calculate the number of photoelectrons liberated in ……………..nm.
time t = 2 s.
Q.14 A hydrogen atom is in 5th excited state. When the
Q.9 A small 10 W source of ultraviolet light of electrons jump to ground state the velocity of recoiling
wavelength 99nm is held at a distance 0.1 m from a hydrogen atom is ……………….m/s and the energy of the
metal surface. The radius of an atom of the metal is photon is ……………eV.
approximately 0.05 nm. Find:
Q.15 The ratio of series limited wavelength of Balmer
(i) The number of photons striking an atom per seconds.
series to wavelength of first line of
(ii) The number of photoelectrons emitted per seconds
Paschen series is …………………..
if the efficiency of liberation of photoelectrons is 1%.
2 4 . 5 8 | Modern Physics

Q.16 A neutron with kinetic energy 25 eV strikes a transition are λ1 , λ2 and λ3 . The value of λ3 in terms
stationary deuteron. Find the de Broglie wavelengths of λ1 and λ2 is given by _______.
of both particles in the frame of their center of mass.

E3 n=3
Q.17 Assume that the de Broglie wave associated with
1
an electron can form a standing wave between the E2 n=2
atoms arranged in a one dimensional array with nodes
2
at each of the atomic sites. It is found that one such
3

E1 n=1
standing wave is formed if the distance ‘d’ between the
atoms of the array is 2 Å. A similar standing wave is
Q.23 Imagine an atom made up of a proton and a
again formed if ‘d’ is increased to 2.5 Å but not for any
hypothetical particle of double the mass of an electron
intermediate value of d. Find the energy of the electrons
but having the same charge as the electron. Apply the
in electron volts and the least value of d for which the
Bohr atom model and consider a possible transition of
standing wave of the type described above can form.
this hypothetical particle to the first excited level. Find
the longest wavelength photon that will be emitted λ
Q.18 A stationary He+ ion emitted a photon (in terms of the Rydberg constant R.)
corresponding to the first line its Lyman series. That
photon liberated a photoelectron from a stationary
Q.24 In a hydrogen atom, the electron moves in an orbit
hydrogen atom in the ground state. Find the velocity of
of radius 0.5 Å making 1016 revolutions per second. The
the photoelectron.
magnetic moment associated with the orbital motion
of the electron is __________.
Q.19 A gas of identical hydrogen like atom has some
atom in the lowest (ground) energy level A & some
Q.25 A hydrogen like atom has its single electron
atoms in a particular upper (excited) energy level B
orbiting around its stationary nucleus. The energy
& there are no atoms in any other energy level. The
excite the electron from the second Bohr orbit to the
atoms of the gas make transition to a higher energy
third Bohr orbit is 47.2 eV. The atomic number of this
level by the absorbing monochromatic light of photon
nucleus is ________________.
energy 2.7eV. Subsequently, the atom emit radiation of
only six different photon energies. Some of the emitted
photons have energy 2.7eV. Some have energy more Q.26 A single electron orbits a stationary nucleus of
and some have less than 2.7eV. charge Ze where Z is a constant and e is the electronic
charge. It requires 47.2eV to excite the electron from
(i) Find the principle quantum of the initially excited
the 2nd Bohr orbit to 3rd Bohr orbit. Find
level B.
(i) The value of Z
(ii) Find the ionization energy for the gas atoms.
(ii) Energy required to excite the electron from third to
(iii) Find the maximum and the minimum energies of
the fourth orbit
the emitted photons.
(iii) The wavelength of radiation required to remove the
electron from the first orbit to
Q.20 A hydrogen atom in ground state absorbs a
photon of ultraviolet radiation of wavelength 50nm. (iv) Infinity the Kinetic energy, potential energy and
Assuming that the entire photon energy is taken up by angular momentum in the first Bohr
the electron, with what kinetic energy will the electron
(v) Orbit the radius of the first Bohr orbit.
be ejected?

Q.27 A hydrogen like atom (atomic number Z) is in


Q.21 An electron joins a helium nucleus to form a He+
higher excited state of quantum number n. This excited
ion in ground state. The wavelength of the photon
atom can make a transition to the first excited state by
emitted in this process if the electron is assumed to
successive emitting two photons of energy 22.95eV and
have had no kinetic energy when it combines with
5.15eV respectively. Alternatively, the atom from the
nucleus is _________ nm.
same excited state can make transition to the second
excited state by successive emitting two photons of
Q.22 Three energy levels of an atom are shown in the energies 2.4eV and 8.7eV respectively. Find the value
figure. The wavelength corresponding to three possible of n and Z.
P hysi cs | 24.59

Q.28 Find the binding energy of an electron in the Q.6 An electrons collides with a fixed hydrogen atom
ground state of a hydrogen like atom in whose in its ground state. Hydrogen atom gets excited
spectrum the third of the corresponding Balmer series and the colliding electron loses all its kinetic energy.
is equal to 108.5 nm. Consequently the hydrogen atom may emit a photon
corresponding to the largest wavelength of the Balmer
Q.29 Which level of the doubly ionized lithium has series. The min. K.E. of colliding electron will be
the same energy as the ground state energy of the (A) 10.2eV (B) 1.9 eV
hydrogen atom? Find the ratio of the two radii of
(C) 12.1 eV (D) 13.6 eV
corresponding orbits.

Q.7 A neutron collides head on with a stationary


Q.30 A 20 KeV energy electron is brought to rest
hydrogen atom in ground state
in an X-ray tube, by undergoing two successive
bremsstrahlung events, thus emitting two photons. (A) If kinetic energy of the neutron is less than 13.6 eV,
The wavelength of the second photon is 130 x 10−12 collision must be elastic.
m greater than the wavelength of the first emitted
(B) If kinetic energy of the neutron is less than 13.6 eV,
photon. Calculate the wavelength of the two photons.
collision may be inelastic.
(C) Inelastic collision takes place when initial kinetic
Exercise 2 energy of neutron is greater than 13.6 eV.

Single Correct Choice Type (D) Perfectly inelastic collision cannot take place.

Q.1 10−3 W of 5000 Å light is directed on a photoelectric Q.8 An electron in hydrogen atom first jumps from
cell. If the current in the cell is 0.16 µA, the percentage second excited state to first excited state and then,
of incident photons which produce photoelectrons, is from first excited state to ground state. Let the ratio of
wavelength, momentum and energy of photons in the
(A) 0.4% (B) 0.04% (C) 20% (D) 10% two cases be x, y and z, then select the wrong answer(s):
1 9 5 5
Q.2 Photons with energy 5eV are incident on a cathode (A) z = (B) x = (C) y = (D) z =
x 4 27 27
C, on a photoelectric cell. The maximum energy of the
emitted photoelectrons is 2 eV. When photons of energy
6 eV are incident on C, no photoelectrons will reach the Multiple Correct Choice Type
anode A if the stopping potential of A relative to C is
Q.9 In photoelectric effect, stopping potential depends on
(A) 3V (B) –3V (C) –1 V (D) 4 V
(A) Frequency of the incident light
Q.3 In a hydrogen atom, the binding energy of the (B) Intensity of the incident light by varying source
electron of the nth state is En , then the frequency of (C) Emitter’s properties
revolution of the electron in the nth orbit is:
(D) Frequency and intensity of the incident light
2En 2Enn En Enn
(A) (B) (C) (D)
nh h nh h
Q.10 Two electrons are moving with the same speed v.
One electron enters a region of uniform electric field
Q.4 Difference between nth and (n+1)th Bohr’s radius
while the other enters a region of uniform magnetic
of ‘H’ atom is equal to it’s (n – 1)th Bohr’s radius. The
field, then after sometime if the de-Broglie wavelengths
value of n is:
of the two are λ1 and λ2 , then:
(A) 1 (B) 2 (C) 3 (D) 4
(A) λ1 =λ2 (B) λ1 > λ2
(C) λ1 < λ2 (D) λ1 > λ2 or λ1 < λ2
Q.5 Electron in a hydrogen atom is replaced by an
identically charged particle muon with mass 207 times
that of electron. Now the radius of K shell will be Q.11 A neutron collides head-on with a stationary
hydrogen atom in ground state. Which of the following
(A) 2.56 × 10−3 Å (B) 109.7 Å
statements are correct (Assume that the hydrogen
(C) 1.21 × 10−3 Å (D) 22174.4 Å atom and neutron has same mass):
2 4 . 6 0 | Modern Physics

(A) If kinetic energy of the neutron is less than 20.4 eV (A) A minimum wavelength
collision must be elastic.
(B) A continuous spectrum
(B) If kinetic energy of the neutron is less than 20.4 eV
(C) Some discrete comparatively prominent wavelength
collision may be inelastic.
(D) Uniform density over the whole spectrum
(C) Inelastic collision may be take place only when initial
kinetic energy of neutron is greater than 20.4 eV.
Assertion Reasoning Type
(D) Perfectly inelastic collision cannot take place.

Q.16 Statement-I: Figure shows Vs


Q.12 A free hydrogen atom in ground state is at rest. A graph of stopping potential
neutron of kinetic energy ‘K’ collides with the hydrogen and frequency of incident light
atom. After collision hydrogen atom emits two photons in photoelectric effect. For v
in succession one of which has energy 2.55 eV. (Assume values of frequency less than (0,0) v0
that the hydrogen atom and neutron has same mass) threshold frequency ( v0 )
(A) Minimum value of ‘K’ is 25.5 eV. stopping potential is negative.
(B) Minimum value of ‘K’ is 12.75 eV. Statement-II: Lower the value of frequency of incident
light (for v > v 0 ) the lower is the maxima of kinetic
(C) The other photon has energy 10.2eV.
energy of emitted photoelectrons.
(D) The upper energy level is of excitation energy
(A) Statement-I is true, statement-II is true and
12.75 eV.
statement-II is correct explanation for statement-I.

Q.13 A particular hydrogen like atom has its ground (B) Statement-I is true, statement-II is NOT the correct
state binding energy 122.4eV. It is in ground state. explanation for statement-I.
Then: (C) Statement-I is true, statement-II is false.
(A) Its atomic number is 3 (D) Statement-I is false, statement-II is true.
(B) An electron of 90eV can excite it.
(C) An electron of kinetic energy nearly 91.8eV can be Q.17 Statement-I: Two photons having equal
almost brought to rest by this atom. wavelengths have equal linear momenta.

(D) An electron of kinetic energy 2.6eV may emerge Statement-II: When light shows its photons character,
from the atom when electron of kinetic energy 125eV h
each photon has a linear momentum λ = .
collides with this atom. p
(A) Statement-I is true, statement-II is true and
statement-II is correct explanation for statement-I.
Q.14 A beam of ultraviolet light of all wavelengths
pass through hydrogen gas at room temperature, in (B) Statement-I is true, statement-II is NOT the correct
the x-direction. Assuming all photons emitted due explanation for statement-I.
to electron transition inside the gas emerge in the (C) Statement-I is true, statement-II is false.
y-direction. Let A and B denote the lights emerging
from the gas in the x and y directions respectively. (D) Statement-I is false, statement-II is true.

(A) Some of the incident wavelengths will be absent in A.


Q.18 Statement-I: In the process of photoelectric
(B) Only those wavelengths will be present in B which emission, all the emitted photoelectrons have same K.E.
are absent in A.
Statement-II: According to Einstein’s photoelectric
(C) B will contain some visible light. equation
(D) B will contain some infrared light. KEmax = hv − φ .
(A) Statement-I is True, statement-II is True, statement-II
Q.15 X-rays are produced by accelerating electrons is a correct explanation for statement-I.
across a given potential difference to strike a meta
target of high atomic number. If the electrons have (B) Statement-I is True, statement-II is True, statement-II
same speed when they strike the target, the X-ray is NOT a correct explanation for statement-I.
spectrum will exhibit
P hysi cs | 24.61

(C) Statement-I is True, statement-II is False Previous Years’ Questions


(D) Statement-I is False, statement-II is True
Q.1 A single electron orbits around a stationary nucleus
of charge +Ze where Z is a constant and e is the
Q.19 Statement-I: Work function of aluminum is 4.2
magnitude of the electronic charge. It requires 47.2 eV
eV. If two photons each of energy 2.5 eV strikes on a
to excite the electron from the second Bohr orbit to the
piece of aluminum, the photoelectric emission does
third Bohr orbit. (1981)
not occur.
Find:
Statement-II: In photoelectric effect a single photon
interacts with a single electron and electron is emitted (a) The value of Z.
only if energy of each incident photon is greater than
(b) The energy required to excite the electron from the
the work function.
third to the fourth Bohr orbit.
(A) Statement-I is True, statement-II is True, statement-II
(c) The wavelength of the electromagnetic radiation
is a correct explanation for statement-I.
required to remove the electron from the first Bohr
(B) Statement-I is True, statement-II is True, statement-II orbit to infinity.
is NOT a correct explanation for statement-I.
(d) The kinetic energy, potential energy and the angular
(C) Statement-I is True, statement-II is False momentum of the electron in the first Bohr orbit.
(D) Statement-I is False, statement-II is True (e) The radius of the first Bohr orbit.
(The ionization energy of hydrogen atom = 13.6 eV, Bohr
Q.20 Statement-I: An electron and a proton are radius = 5.3 × 10−11 m , velocity of light = 3 × 108 m / s .
accelerated through the same potential difference. The Planck’s constant = 6.6 × 10−34 J s ).
de-Broglie wavelength associated with the electron is
longer.
Q.2 Hydrogen atom in its ground state is excited by
Statement-II: de-Broglie wavelength associated means of monochromatic radiation of wavelength
h 975° Å. How many different lines are possible in the
with a moving particle is λ = where, p is the linear resulting spectrum? Calculate the longest wavelength
p
momentum and both have same K.E. amongst them. You may assume the ionization energy
for hydrogen atom as 13.6 eV. (1982)
(A) Statement-I is True, statement-II is True, statement-II
is a correct explanation for statement-I.
Q.3 How many electrons, protons and neutrons are
(B) Statement-I is True, statement-II is True, statement-II there in a nucleus of atomic number 11 and mass
is NOT a correct explanation for statement-I. number 24? (1982)
(C) Statement-I is True, statement-II is False (a) Number of electrons =
(D) Statement-I is False, statement-II is True (b) Number of protons =
(c) Number of neutrons =
Q.21 Statement-I: In a laboratory experiment, on
emission from atomic hydrogen in a discharge tube,
Q.4 A uranium nucleus (atomic number 92, mass
only a small number of lines are observed whereas
number 238) emits an alpha particle and the resulting
a large number of lines are present in the hydrogen
nucleus emits β -particle. What are the atomic number
spectrum of a star.
and mass number of the final nucleus? (1982)
Statement-II: The temperature of discharge tube is
(a) Atomic number =
much smaller than that of the star.
(b) Mass number =
(A) Statement-I is True, statement-II is True, statement-II
is a correct explanation for statement-I.
Q.5 Ultraviolet light of wavelengths 800 Å and 700 Å
(B) Statement-I is True, statement-II is True, statement-II
when allowed to fall on hydrogen atoms in their ground
is NOT a correct explanation for statement-I.
state is found to liberate electrons with kinetic energy
(C) Statement-I is True, statement-II is False 1.8 eV and 4.0 eV respectively. Find the value of Planck’s
constant.  (1983)
(D) Statement-I is False, statement-II is True
2 4 . 6 2 | Modern Physics

Q.6 the ionization energy of a hydrogen like Bohr atom Q.11 The wavelength of light emitted in the visible
is 4 Rrydberg. region by He+ ions after collisions with H-atoms is
 (2008)
(a) What is the wavelength of the radiation emitted
when the electron jumps from the first excited state to (A) 6.5 × 10−7 m (B) 5.6 × 10−7 m
the ground state?  (1984)
(C) 4.8 × 10−7 m (D) 4.0 × 10−7 m
(b) What is the radius of the first orbit for this atom?
1 rH
Now, as r ∝ Radius of first orbit of this atom, r1 = 1 Q.12 The ratio of the kinetic energy of the n = 2 electron
Z Z for the H atom to that of He+ ion is  (2008)
0.529
= = 0.2645 Å
2 1 1
(A) (B) (C) 1 (D) 2
4 2
Q.7 A doubly ionized lithium atom is hydrogen-like
with atomic number 3. Q.13 Some laws/processes are given in column I. Match
(a) Find the wavelength of the radiation required to these with the physical phenomena given in column II.
excite the electron in Li2+ from the first to the third  (2006)
Bohr orbit. (Ionization energy of the hydrogen atom
equals 13.6 eV.)
Column I Column II
(b) How many spectral lines are observed in the emission (A) Nuclear fusion (p) Converts some
spectrum of the above excited system? (1985) matter into energy.
(B) Nuclear fission (q) Generally possible
Q.8 There is a stream of neutrons with a kinetic energy for nuclei with low
of 0.0327 eV. If the half-life of neutrons is 700s, what atomic number
fraction of neutrons will decay before they travel a
(C) (r) Generally possible
distance of 10 m? (1986) β -decay
for nuclei with higher
atomic number
Q.9 A particle of charge equal to that of an electron (D) Exothermic (s) Essentially proceeds
–e, and mass 208 times of the mass of the electron nuclear reaction by weak nuclear
(called a mu-meson) moves in a circular orbit around a forces
nucleus of charge +3e. (Take the mass of the nucleus to
be infinite). Assuming that the Bohr model of the atom
is applicable to this system. (1988) Q.14 The threshold wavelength for photoelectric
emission from a material is 5200 Å. Photoelectrons
(a) Derive an expression for the radius of the nth Bohr will be emitted when this material is illuminated with
orbit. monochromatic radiation from a  (1982)
(b) Find the value of n for which the radius of the orbit (A) 50W infrared lamp (B) 1 W infrared lamp
is approximately the same as that of the first Bohr orbit
(C) 50W ultraviolet lamp (D) 1W ultraviolet lamp
for the hydrogen atom.
(c) Find the wavelength of the radiation emitted when
Q.15 The allowed energy for the particle for a particular
the mu-meson jumps from the third orbit to the first
value of n is proportional to  (2009)
orbit. (Rydberg’s constant = 1.097 × 107 m−1 )
(A) a-2 (B) a-3/2 (C) a-1 (D) a2
+
Paragraph 1: (Q.10-Q.12) In a mixture of H − He gas
( He+ is single ionized He atom), H atoms and He+ Q.16 If the mass of the particle is m = 1.0 ×10−30 kg and
ions excited to their respective first excited states. a = 6.6 nm, the energy of the particle in its ground state
Subsequently, H atoms transfer their total excitation is closest to  (2009)
energy of He+ ions (by collisions). Assume that the
(A) 0.8 MeV (B) 8 MeV (C) 80 MeV (D) 800 MeV
Bohr model of atom is exactly valid.

Q.17 The speed of the particle that can take discrete


Q.10 The quantum number n of the state finally
values is proportional to (2009)
populated in He+ ions is  (2008)
(A) n−3/2 (B) n-1 (C) n1/2 (D) n
(A) 2 (B) 3 (C) 4 (D) 4
(A) (B)
P hysi cs | 24.63
 
Q.18 An α -particle and a proton are accelerated from V0 V0
rest by a potential difference of 100V. After this, their
de-Broglie wavelength are λ α and λp respectively. The
(C) (D)
λp
ratio , to the nearest integer, is : (2010)
λα 1/ 1/

Q.19 The wavelength of the first spectral line in the



Balmer series of hydrogen atom is 6561 A . The Q.25 In a historical experiment to determine Planck’s
wavelength of the second spectral line in the Balmer constant, a metal surface was irradiated with light of
series of singly-ionized helium atom is  (2010) different wavelengths. The emitted photoelectron
    energies were measured by applying a stopping
(A) 1215 A (B) 1640 A (C) 2430 A (D) 4687 A
potential. The relevant data for the wavelength ( λ ) of
incident light and the corresponding stopping potential
Q.20 A proton is fired from very far away towards a (V0) are given below:  (2016)
nucleus with charge Q = 120 e, where e is the electronic
charge. It makes a closest approach of 10 fm to the V0 (Volt)
λ(µ m)
nucleus. The de Broglie wavelength (in units of fm) of
the proton at its start is: (take the proton mass, mp =
0.3 2.0
1
(5/3) × 10-27 kg; h/e = 4.2 × 10-15 J.s / C; = 9 × 109
4πε0 0.4 1.0
m/F; 1 fm = 10-15)  (2012) 0.5 0.4

Q.21 A pulse of light of duration 100 ns is absorbed Given that c = 3 × 108 m s-1 and e = 1.6 × 10-19 C,
completely by a small object initially at rest. Power of Planck’s constant (in units of J s) found from such an
the pulse is 30 mW and the speed of light is 3 × 108 experiment is
m/s. The final momentum of the object is (2013)
(A) 6.0 × 10-34 (B) 6.4 × 10-34
(A) 0.3 × 10 -17
kg ms -1
(B) 1.0 × 10
-17
kg ms -1
(C) 6.6 × 10-34 (D) 6.8 × 10-34
(C) 3.0 × 10-17 kg ms-1 (D) 9.0 × 10-17 kg ms-1
Q.26 Highly excited states for hydrogen-like atoms
Q.22 The work functions of Silver and Sodium are 4.6 (also called Rydberg states) with nuclear charge Ze are
and 2.3 eV, respectively. The ratio of the slope of the defined by their principal quantum number n, where n
stopping potential versus frequency plot for Silver to >> 1. Which of the following statement(s) is(are) true?
that of Sodium is  (2013)  (2016)
(A) Relative change in the radii of two consecutive
Q.23 Consider a hydrogen atom with its electron in the orbitals does not depend on Z
nth orbital. An electromagnetic radiation of wavelength
(B) Relative change in the radii of two consecutive
90 nm is used to ionize the atom. If the kinetic energy
orbitals varies as 1/n
of the ejected electron is 10.4 eV, then the value of n is
(hc = 1242 eV nm) (2015) (C) Relative change in the energy of two consecutive
orbitals varies as 1/n3
Q.24 For photo-electric effect with incident photon (D) Relative change in the angular momenta of two
wavelength λ , the stopping potential is Vo. Identify the consecutive orbitals varies as 1/n
correct variation(s) of V0 with λ and 1/ λ . (2015)
Q.27 A hydrogen atom in its ground state is irradiated
V0 V0 by light of wavelength 970A. Taking hc/e = 1.237 × 10-6
eVm and the ground state energy of hydrogen atom as
(A) (B) –13.6 eV, the number of lines present in the emission
spectrum is (2016)
 
Q.28 A glass tube of uniform internal radius (r) has
V0 V0
a valve separating the two identical ends. Initially,

(C) (D)
2 4 . 6 4 | Modern Physics

the valve is in a tightly closed position. End 1 has a


1  h2  1  h2 
hemispherical soap bubble of radius r. End 2 has sub- (A)   (B)  
hemispherical soap bubble as shown in figure. Just n2  8π2 I  n  8π2 I 
after opening the valve,  (2008)
 h2   h2 
Figure: (C) n   (D) n2  
 8π2 I   8π2 I 
   

Q.31 It is found that the excitation frequency from


ground to the first excited state of rotation for the CO
2 1 4
molecule is close to × 1011 Hz . Then the moment of
(A) Air from end 1 flows towards end 2. No change in π
the volume of the soap bubbles inertia of CO molecule about its centre of mass is close

(B) Air from end 1 flows towards end 2. Volume of the to (Take h = 2π × 10−34 Js)
soap bubble at end 1 decreases (A) 2.76 ×10−46 kg m2 (B) 1.87 ×10−46 kg m2
(C) No changes occurs (C) 4. 67 ×10−47 kg m2 (D) 1.17 ×10−47 kg m2
(D) Air from end 2 flows towards end 1. volume of the
soap bubble at end 1 increases Q.32 In a CO molecule, the distance between C (mass =
12 a.m.u) and O (mass = 16 a.m.u.),
5
Q.29 Photoelectric effect experiments are performed where 1 a.m.u. = × 10−27 kg, is close to
3
using three different metal plates p, q and r having
work functions φp = 2.0 eV, φq = 2.5 eV and φr = 3.0 eV, (A) 2.4 ×10−10 m (B) 1.9 ×10−10 m
respectively. A light beam containing wavelengths of (C) 1.3 ×10−10 m (D) 4.4 ×10−11 m
550 nm, 450 nm and 350 nm with equal intensities
illuminates each of the plates. The correct I-V graph for
Q.33 A silver sphere of radius 1 cm and work function
the experiment is (Take hc = 1240 eV nm) (2009)
4.7 eV is suspended from an insulating thread in free
l l space. It is under continuous illumination of 200 nm
P
q wavelength light. As photoelectrons are emitted, the
r sphere gets charged and acquires a potential. The
(A) (B) Pq maximum number of photoelectrons emitted from the
r sphere is A ×10z (where 1 < A < 10). The value of ‘Z’ is
V V  (2011)
l r l
q Q.34 Two bodies, each of mass M, are kept fixed with a
(C) P (D) rq separation 2L. A particle of mass m is projected from the
midpoint of the line joining their centres, perpendicular
P to the line. The g ravitational constant is G. The correct
V V statement(s) is (are) (2013)

Paragraph for questions 30 to 32 (2010) (A) The minimum initial velocity of the mass m to

The key feature of Bohr’s theory of spectrum of hydrogen escape the gravitational field of the two bodies is 4
GM
atom is the quantization of angular momentum when L
an electron is revolving around a proton. We will extend (B) The minimum initial velocity of the mass m to escape
this to a general rotational motion to find quantized
the gravitational field of the two bodies is GM
rotational energy of a diatomic molecule assuming it to 2
L
be rigid. The rule to be applied is Bohr’s quantization
condition. (C) The minimum initial velocity of the mass m to escape
2GM
the gravitational field of the two bodies is
Q.30 A diatomic molecule has moment of inertia I. By L
Bohr’s quantization condition its rotational energy in
the nth level (n = 0 is not allowed) is (D) The energy of the mass m remains constant.
P hysi cs | 24.65

Q.35 A metal surface is illuminated by light of two


different wavelengths 248 nm and 310 nm. The Light
maximum speeds of the photoelectrons corresponding
to these wavelengths are u1 and u2, respectively. If
the ratio u1 : u2 = 2 : 1and hc = 1240 eV nm, the work
function of the metal is nearly  (2014) Electrons

(A) 3.7 eV (B) 3.2 eV


(C) 2.8 eV (D) 2.5 eV
v
- +
Q.36 Light of wavelength λph falls on a cathode plate
inside a vacuum tube as shown in the figure. The work (A) For large potential difference (V >> φ /e), λe is
function of the cathode surface is φ and the anode is a approximately halved if V is made four times
wire mesh of conducting material kept at a distance d
from the cathode. A potential difference V is maintained (B) λe decreases with increase in φ and λph
between the electrodes. If the minimum de Broglie (C) λe increases at the same rate as λph for λph < hc/ φ
wavelength of the electrons passing through the anode
(D) λe is approximately halved, if d is doubled
is λe , which of the following statement(s) is(are) true?
 (2016)

PlancEssential Questions
JEE Main/Boards JEE Advanced/Boards
Exercise 1 Exercise 1
Q.10 Q.17 Q.23 Q.4 Q.8 Q.10
Q.27 Q.28 Q.19 Q.22 Q.30

Exercise 2 Exercise 2
Q.2 Q.4 Q.7 Q.1 Q.6 Q.8
Q.12 Q.13 Q.14
Q.16

Previous Years’ Questions


Q.10
2 4 . 6 6 | Modern Physics

Answer Key

JEE Main/Boards
Exercise 1
λ
Q.1 (iii)1.49 × 10−19 J Q.4 Q.8 Line B represent a particle of
smaller mass. 2

Q.9 (i) Metal Q, (ii) Metal P, (iii) Stopping potential remains unchanged.
Q.10. 1.5 eV Q.11 1 : 2 2 Q.17 2.46 × 1015 Hz
Q.21 (i) –1.7 eV (ii) 972.54 Å Q.23 +13.6 eV, –27.2 eV Q.24 28.8 × 10−15 m
Q.27 7.5 × 1014 Hz, 3.094 eV Q.28 4137.5 Å

Exercise 2

Single Correct Choice Type


Q.1 C Q.2 D Q.3 C Q.4 A Q.5 A Q.6 C
Q.7 B Q.8 B Q.9 C Q.10 A Q.11 D Q.12 C
Q.13 B Q.14 C Q.15 D Q.16 A Q.17 A Q.18 C
Q.19 C Q.20 A Q.21 B Q.22 B Q.23 A Q.24 B
Q.25 A Q.26 B Q.27 A Q.28 B Q.29 D

Previous Years’ Questions


Q.1 B Q.2 C Q.3 A Q.4 D Q.5 C Q.6 D
Q.7 C Q.8 B Q.9 30×103V, 30×103eV Q.10 500 dps, 125 dps
Q.11 Frequency Q.12 8, 6 Q.13 B Q.14 B Q.15 D Q.16 D
Q.17 B Q.18 D Q.19 A Q.20 C Q.21 C Q.22 D
Q.23 D Q.24 D Q.25 B Q.26 C Q.27 C Q.28 D

JEE Advanced/Boards
Exercise 1
15
Q.1 (a) 0.6 V, (b) 2.0 mA Q.2 5.76 × 10−11 A Q.3 V
8
5 1019
Q.4 , Q.5 2eV, 6.53 × 10−34 J s Q.6 4.8×1016, 4.0 cm
16 8 π

Q.7 2 eV, 0.754 V Q.8 1.1 × 1012
5 5
Q.9 (i) photon/s, (ii) electrons/s Q.10 (a) 105 s−1 ; (b) 286.76 ; (d) 111.1s
16 1600
P hysi cs | 24.67

h
Q.11 (a) 2.25 eV, (b) 4.2 eV, (c) 2.0 eV, 0.5 eV Q.12 −
eEt2
Q.13 487 nm Q.14 4.26 m/s, 13.2 eV Q.15 7 : 36

Q.16 λdeutron = 8.6 pm


λneutron = Q.17 KE ≅ 148.4 eV, dleast = 0.5 Å Q.18 3.1 × 106 m / s

Q.19 (i) 2 ; (ii) 23.04 × 10−19 J ; (iii) 4 → 1, 4 → 3 Q.20 11.24 eV
λ1 λ2 18
Q.21 22.8 nm Q.22 Q.23
λ1 + λ2 5R

Q.24 1.257 × 10−23 Am2 Q.25 5

Q.26 (i) Z = 5, (ii) E= 16.5 eV, (iii) λ=36.4 A, (iv) K.E = 340 eV, P.E = –680 eV, (v) Radius r = 0.1058 Å

Q.27 Z = 3, n = 7 Q.28 54.4 eV Q.29 n = 3, 3 : 1


 
Q.30 λ1 =0.871 A and λ2 =2.17 A

Exercise 2

Single Correct Choice Type

Q.1 B Q.2 B Q.3 A Q.4 D Q.5 A Q.6 C

Q.7 A Q.8 B

Multiple Correct Choice Type

Q.9 A, C Q.10 A, D Q.11 A,C Q.12 A, C, D Q.13 A, C, D

Q.14 A, C, D Q.15 A, B, C

Assertion Reasoning Type

Q.16 D Q.17 C Q.18 D Q.19 D Q.20 A Q.21 A

Previous Years’ Questions

kgm2
Q.1 (a) 5 (b) 16.53 eV (c) 36.4 Å (d) 340 eV, –680 eV, –340 eV, 1.05 × 10−34 (e) 1.06 × 10−11 m
s

Q.2 Six, 1.875 µm Q.3 0, 11, 13 Q.4 (a) 91 (b) 234

Q.5 6.6 × 10−34 J s Q.6 (a) 300 Å (b) 0.2645 Å Q.7 (a) 113.74 Å (b) 3

n2h2 ε0 
Q.8 3.96 × 10−6 Q.9 (a) rn = (b) n = 25 (c) 0.546 A
624 πmee2

Q.10 C Q.11 C Q.12 A Q.13 A → p, q; B → p, r; C → p, s; D → q

Q.14 C,D Q.15 A Q.16 B Q.17 D Q.18 3 Q.19 A

Q.20 7 fm Q.21 B Q.22 1 Q.23 2 Q.24 A Q.25 B

Q.26 A, B, D Q.27 6 Q.28 B Q.29 A Q.30 D Q.31 B

Q.32 C Q.33 7 Q.34 B Q.35 A Q.36 A


2 4 . 6 8 | Modern Physics

Solutions

JEE Main/Boards

Exercise 1 l2 =
h
=
h
=
λ1
=
λ
2mKE2 2me × 4v 2 2

Sol 1: (i) Work function – It is the minimum energy


of incident photon below which no ejection of Sol 5: For graph, refer theory.
photoelectron from a metal surface will take place is
If the distance is reduced, intensity of light will increase
known as work function or thresholds energy for that
and number of electrons will increase and current will
metal
increase. Stopping potential will not be affected by
φ = hV0 distance.
(ii) Threshold frequency – it is the minimum frequency
of incident photon below which no ejection of Sol 6: hν1 = W1 + KE
photoelectron from a metal surface will take place is hν2 = W2 + KE
known as threshold frequency for that metal.
W1 > W2
(iii) Stopping potential – The negative potential (V0)
applied to the anode at which the current gets reduced ⇒ ν1 > ν2
to zero is called stopping potential.
KEmax = E – φ Sol 7: Refer theory

12400 h
E= = 4.13 eV Sol 8: λ =
3000 2meV

KEmax = 4.13 – 3.2 = 0.93 eV = 1.49 × 10–19 J h


Slope =
2me
Sol 2: KE = eV Slope of B > Slope of A

h h h ⇒ mB < m A
ld = =. .=
p 2mKE 2meV
Sol 9: (i) Metal P has greater threshold wavelength
6.6 ×10−34 12.27 because P has lower threshold frequency.
=. = Å
2 × 9.1 ×10−31 ×1.67 ×10−19 × V V (ii) KEmax = hν – hν0
For Davisson and Germer’s experiment, refer theory. Metal P emits electrons with less kinetic energy as P has
less threshold frequency.
hc
Sol 3: φA = 2 eV = (iii) If distance is doubled, there is no change in stopping
λA potential.
hc
φB = 5 eV =
λB Sol 10: KEmax = eV = 1.5 eV
λA > λB
λα h h
Sol 11: = ÷
Sol 4: K.E. = eV λp 2mα (KE)α 2mp (KE)p
h h
l1 = = =λ
2mKE1 2meV mp (KE)p 1 eV 1
= = · =
mα (KE)α 4 2eV 2 2
P hysi cs | 24.69

Sol 12: hν = hn0 + KE Sol 20: Refer theory


Refer theory
Sol 21: (i) PE = 2 T.E.
Sol 13: Refer theory −13.6 × 2
PE = = – 0.85 × 2 = – 1.7 eV
16
Sol 14: Refer theory for the graph
1 1  15
hν1 = hν0 + eV; hν2 = hν0 + eV (ii) E = 13.6  −  = ×13.6 eV
2 2
1 4  16
Stopping potential V is higher for ν1 frequency by the
above equations. E = 12.75 eV
12400
h h λ= = 972.54 Å
Sol 15: λp = ; λe = 12.75
2mp (KE) 2me (KE)
mp > me Sol 22: Refer theory

lp < le
Sol 23: T.E. = – 13.6 eV

Sol 16: (i) Cut-off voltage is the negative potential KE = |T.E.| = 13.6 eV
applied to the anode at which the current gets reduced P.E = 2T.E = – 13.6 × 2 = –27.2 eV
to zero.
K × (2e)(80e)
Refer theory for second part of question. Sol 24: 8 × 106 =
r

Sol 17: Refer theory 9 ×109 ×160 ×1.6 ×10−19


r= = 28.8 × 10–15 m
6
8 ×10
E = – 3.4 eV – (– 13.6 eV)
If kinetic energy is doubled then closest distance will
E = 10.2 eV
become half of the original.
hν = 10.2 × 10–19 × 1.6 J
h
ν = 2.46 × 1015 Hz Sol 25: λ =
2m(KE)
2
mv h h
Sol 18: eE = . . =λe =
r
2me (KE)e 2mp (KE)p
V
me(KE)e = mp(KE)p
eE
m p > me
⇒ (KE)e > (KE)p

Sol 26: Kinetic energy = hν – hn0

e v2 It is independent of the intensity of light.


=
m rE c 3 ×108
Sol 27: ν = = = 0.75 × 1015 = 7.5 × 1014 Hz
λ −7
Sol 19: Magnetic moment of a charged particle moving 4 ×10
in a circle is given by Energy = hν

µ = IA = 6.6 × 10–34 × 7.5 × 1014 = 49.5 × 10–20 J= 3.094 eV

e
I = charge flowing per sec
= ω Sol 28: Longest wavelength ⇒ minimum frequency

photon = n0
eω evr
=
µ × πr 2 = 12400
2π 2 = 4137.5 Å
3
2 4 . 7 0 | Modern Physics

Exercise 2 [As energy is greater than 12.1 eV so 1 state has to be


ground state]
Single Correct Choice Type 1 1
12.1 = 13.6  − 
 12 n22 
Sol 1: (C) Intensity of both bulb is same i.e. I1 = I2
12.1 1
Er = Energy of red colour photon = 1−
13.6 n22
Eb = Energy of blue colour photon
1 12.1 1.5
Eb > Er 1−
= =
n22 13.6 13.6
I1
nr = 13.6
Er n22 =
1.5
I2
nb = n2 = 3
Eb
⇒ nb < nr
Sol 5: (A) E1 = 2φ
E2 = 5φ
hc
Sol 2: (D) = φ +K  … (i)
λ E1 = φ + KE1
4hc 2 φ – φ = KE1
= φ + K2  … (ii)
3λ 1
By (i) and (ii) KE1 = φ = mv12
2
4 E2 = φ + KE2
[φ + K] = φ + K2
3
5 φ – φ = KE2
4φ 4k
+ = φ + K2 1
3 3 KE2 = 4φ = mv22
2
4K φ
K=
2 +
3 3 2φ
v1 m =1
=
Sol 3: (C) hν = φ + KE v2 8φ 2
m
KE = eV
V = stopping potential
hc
hν = φ + eV  … (i) Sol 6: (C) = φ + KE1
λ1
h2ν = φ + eV2  … (ii)
KE1 = eV1
By (i) and (ii)
hc  hc  1
2(φ + eV) = φ + eV2 = φ + eV1 ⇒ V
=1  − φ 
λ1  λ1 e
φ + 2eV = eV2
hc  hc  1
φ = φ + eV2 ⇒ V
=2  − φ 
V2 = 2V +
e
λ2  λ2 e
hc  hc  1
= φ + eV3 ⇒ V =3  − φ 
Sol 4: (A) E = φ + KE λ3 λ e
 3 
KE = eV, where V is stopping potential V1 V2 and V3 are in A.P.
E = 1.7 eV + 10.4 eV ⇒ V1 + V3 = 2V2
E = 12.1 eV  hc  1  hc  1  hc  1
1 1 2  − φ 
12.1 = 13.6 × (1)2  −   − φ  +  − =
φ

2 2  λ1  e  λ3 e  λ2 e
 n1 n2 
P hysi cs | 24.71

hc hc 2hc 2hc
+ = = φ + K2
λ1 λ3 λ2 λ1

1 1 2 By (i) and (ii)


+ =
λ1 λ3 λ2 2(φ + K1) = φ + K2 ⇒ φ + 2K1 = K2

Sol 7: (B) φ = 4.5 eV K2 φ


K1 = −
2 2
Wavelength of light = 2000Å
12400 124 Sol 13: (B) Distance between two successive maxima in
Energy of photon = = = 6.2eV
2000 20 λD
Young’s double slit experiment is
K.E. of emitted electron = hν – φ d
= 6.2 – 4.5 = 1.7 eV Distance will decrease if D will decreases.
As electrons are further accelerated by 2V
Sol 14: (C)
so final kinetic energy = 1.7 eV + 2eV
2.18 ×10−18
Energy required = = 2.42 × 10–19 J
= 3.7 eV 9

Sol 8: (B) hν = hn0 + KE Sol 15: (D) In some Hydrogen like atom
Kinetic energy depends only on the energy of incident  13.6 13.6  2
photon. Number of emitted electrons ∝ intensity of light. E4n – E2n =  − + Z
2 2 
 (4n) (2n) 
h h
Sol 9: (C) λp= = 13.6 Z 2  1 
pp 2mp (KE)p  
=  − + 1
4n2  4 
h h
λe= = 3 ×13.6 Z 2
pe 2me (KE)e  =
16n2
mp > me
As proton and electron both are accelerated by same  13.6 13.6  2
E2n – En =  − + Z
potential difference so KEp = KEe  (en)
2
n2 
lp < le
13.6 z 2  1  3 ×13.6 Z 2
  =  − + 1  =
n2  4  4n2
Sol 10: (A) Initial KE is 100 eV
After accelerating through potential difference of 50 v E4n − E2n 1
Ratio = =
final KE is 150 eV. E2n − En 4

150 150 Ratio is independent of Z and n.


λd = = =1 Å
V 150
Sol 16: (A) n’ = n + 1
h h
λ = ; p = 1012 h
Sol 11: (D) = n’ = 3
p 0.01 × 10−10
(n + 1 − 3) ( (n + 1 − 3) + 1 )
hc No. of lines =
Sol 12: (C) = φ + K1  … (i) 2
λ1
hc (n − 2)(n − 1)
= φ + K2  =
… (ii) = 10
λ2 2

λ1 (n – 2) (n – 1) = 20,
l2 =
2 n=6
2 4 . 7 2 | Modern Physics

Sol 17: (A) Ten different wavelengths are emitted so So P.E. will decrease
n(n − 1) −13.6 Z 2
= 10 ⇒ n (n – 1) = 20 T.E. =
2
n2
⇒n=5
T.E. will decrease with decrease in n.
1 1
Energy of incident radiation is 13.6  −  nh
 12 52  Sol 21: (B) Angular momentum = = mvr

24 1
= ×13.6 = 13.056 eV f ∝
25 n3
12400
λ= = 949.75 Å r ∝ n2
13.056
1
f rL ∝ × n2 × n =1 ⇒ independent of n.
3
Sol 18: (C) Five dark line corresponds to transitions so n
highest state of electron is n = 6
n(n − 1) 0.529n2 0.529 × 4
So no of lines in emission spectrum = Sol 22: (B) r = = = 1.058 Å
2 Z 2
6× 5
= = 15 −13.6
2 Sol 23: (A) Energy En = = – 3.4 (n = 2)
n2
n=6 nh 2h h
n=5 angular momentum = = =
n=4 2π 2π π
n=3 −13.6
Sol 24: (B) En = −3.4 =
= T.E.
n=2 n2
n=2
n=1
Kinetic energy = |T.E.| = 3.4 eV
1 1 
Sol 19: (C) Energy of photon = 13.6  −  h 6.6 ×10−34
 12 25  =λ =
2mKE 2 × 9.1 ×10−31 × 3.4 × 1.6 × 10−19
24
= × 13.6 eV
25 6.6 ×10−34 25
= ×10
= 6.6 × 10 –10 m
Momentum of photon 9.9

h h E 24 13.6 ×1.6 ×10−19


= = = = × Sol 25: (A) Since some photons have energy greater
λ hc c 25 3 ×108
than 13.6 eV so electrons in hydrogen atoms will come
E
out of hydrogen atom. So only Lyman series absorbtion
By momentum conservation will be observed. Electron will not excite in other excited
mv = 6.96 × 10–27 states as there are only few photons of required energy
for the transition. So Balmer series will not be observed.
1.67 × 10–27 × v = 6.96 × 10–27
v = 4.169 m/s Sol 26: (B) Difference of energy between any shell is
independent of the reference level.
1
Sol 20: (A) Velocity ∝
n T.E. = K.E. + P.E.
So kinetic energy will increase KE at ground state = 13.6 eV

−2 ×13.6 Z 2 So T.E. at ground state = 13.6 + 0 = 13.6 eV


P.E. =
n2 1
Sol 27: (A) r ∝
m
P hysi cs | 24.73

Mass of dueterium > mass of hydrogen Sol 6: (D) For a given plate voltage, the plate current
in a triode valve is maximum when the potential of the
⇒ rd < rh
grid is positive and plate is positive
Velocity is same for both.
Energy of dueterium > Energy of hydrogen Sol 7: (C) The X-ray beam coming from an X-ray tube
will be having all wavelengths larger than a certain
⇒ Wavelength of dueterium < wavelength of
minimum wavelength
hydrogen
nh Sol 8: (B) Cut-off wavelength depends on the
Angular momentum =mvr =
2π accelerating voltage, not the characteristic wavelengths.
is independent of mass. Further, approximately 2% kinetic energy of the
electrons is utilized in producing X-rays. Rest 98% is
hc 12420 lost in heat.
Sol 28: (B) lmin = = Å
eV V
Sol 9: Minimum voltage required is corresponding to n
12420
=V = 18.75kV = 1 to n = 2. Binding energy of the innermost electron
0.663 is given as 40 keV i.e., ionization potential is 40 kV.
Therefore,
12420
Sol 29: (D) λmin =
V 1 1 
40 × 103  2 − 2 
If V increase lmin will decrease. Vmin
= 1 2 
= 30 × 103 V
1 1
 2 − 
1 ∞

Previous Years’ Questions The energy of the characteristic radiation will be


30 × 103 eV .
n
Sol 1: (B) Shortest wavelength or cut-off wavelength 1
Sol 10: R = R 0  
depends only upon the voltage applied in the Coolidge 2
tube.
Here R 0 = initial activity
Sol 2: (C) Beta particles are fast moving electrons which = 1000 disintegration/s
are emitted by the nucleus.
and n = number of half-lives.
At t = 1 s, n = 1
Sol 3: (A) The maximum number of electrons in an
orbit are 2n2 . If n > 4, is not allowed, then the number 31
of maximum electrons that can be in first four orbit are:
∴ R 10
= =   500 disintegration/s
2
2(1)2 + 2(2)2 + 2(3)2 + 2(4)2 At t = 3 s, n = 3
= 2 + 8 + 18 + 32 = 60 3
1
R = 103   = 125 disintegration/s
Therefore, possible elements can be 60. 2

Sol 4: (D) Shortest wavelength will correspond to Sol 11: Kmax = hν − W


maximum energy. As value of atomic number (Z)
increases, the magnitude of energy in different energy Therefore, Kmax is linearly dependent on frequency of
states gets increased. Value of Z is maximum for doubly incident radiation.
ionized lithium atom (Z = 3) among the given elements.
Hence, wavelength corresponding to this will be least. Sol 12: Number of α-particles emitted
238 − 206
n1 = =8
Sol 5: (C) During fusion process two or more lighter 4
nuclei combine to form a heavy nucleus.
and number of β -particles emitted are say n2 , then
2 4 . 7 4 | Modern Physics

92 − 8 × 2 + n2 =82 Sol 19: (A) 4 x 103 = 1020 x hf


∴ n2 = 6 4 × 103
f=
1020 × 6.023 × 10 –34
Sol 13: (B)
f = 6.03 x 1016 Hz
The obtained frequency lies in the band of X–rays.

i Sol 20: (C) KEmax = hυ – hυ0

hυ – hυ0 = e × ∆v
2d cos i = nλ
hυ hυ0
h V0 = –
2d cos i = e e
2meV
'υ ' is doubled
v = 50 volt
KEmax = 2hυ – hυ0
Sol 14: (B) 2d cos i = nλdB 2hυ hυ0
V0' = ( ∆V)' = –
e e
Sol 15: (D) Diffraction pattern will be wider than the
KEmax
slit. may not be equal to 2
KEmax

Sol 16: (D) W = QdV = Q(Vq- VP) V0 '


⇒ may not be equal to 2
= -100 ×(1.6 ×10-19) ×(– 4 – 10) V0

= + 100 ×1.6 ×10-19×14 = +2.24 10-16 J. Ke = max = hυ – hυ0

hυ hυ0
Sol 17: (B) V= –
e e
1 2 hc 1240evnm
mv= eV= 1.68 eV ⇒ h=
ν = = 3.1 eV ⇒ 3.1 eV= W + 1.6 eV
2 o
λ 400nm Sol 21: (C)o Davisson – Germer experiment showed that
1240evnm electron beams can undergo diffraction when passed
= 3.1 eV ⇒ 3.1 eV
= Wo + 1.6 eV through atomic crystals. This shows the wave nature
400nm
of electrons as waves can exhibit interference and
∴ Wo = 1.42 eV
diffraction.

Sol 18: (D) Since the frequency of ultraviolet light is m2 f m1r


Sol 22: (D) r1 = ; r2 =
less than the frequency of X–rays, the energy of each m1 + m2 m1 + m2
incident photon will be more for X–rays.
nh
K.Ephotoelectron = hv - φ (l1 + l2 )ω = n
=

1 (m1 + m2 )n2h2
Stopping potential is to stop the fastest photoelectron K.E.
= ( I
2 2 2
+ I ) ω
=2

hv φ 2m1m2r 2
V=
0 −
e e
So, K.Emax and V0 both increases. Sol 23: (D) As λ is increased, there will be a value of λ
above which photoelectrons will be cease to come out
But K.E ranges from zero to K.Emax because of loss of so photocurrent will become zero. Hence, (D) is correct
energy due to subsequent collisions before getting answer .
ejected and not due to range of frequencies in the
incident light.
P hysi cs | 24.75

Sol 24: (D) ∆E = hν 4 41  1


⇒ w +  mv 2  =w + m(v')2
3 32  2
∆E  1 1 k2n
ν=
= k = – 
h  (n– 1)
2
n  n (n– 1)2
2 2 1 w 41
⇒ m(v')2 =+ mv 2
2 3 32
2k 1
≈ ∝ 1 41 
n 3
n3 ⇒ m(v')2 >  mv 2 
2 32 
mυ 4
Sol 25: (B) r = ⇒ v' > v
qB 3
2meV
=
eB
JEE Advanced/Boards
1 2m Br e 2 2
= = V ⇒=
B e V
2m
= 0.8 V Exercise 1

For transition between 3 to 2, Sol 1: (a) Stopping potential is a property of material so


it will remain same.
1 1 13.6 × 5
E = 13.6  –  = = 1.88 eV 1
4 9 36 (b) Saturation current ∝
Work function = 1.88 eV – 0.8 eV r2
r is thrice of initial distance so
= 1.08 eV = 1.1 eV
1
Saturation current = ×18mA =
2mA
1 9
1 1
Sol 26: (C) = RZ 2  – 
λ  n12 n2  Sol 2: λ = 540 nm
1
⇒λ∝ for given n1 & n2 12400 62
Z2 Energy of photon
= E = eV
5400 27
⇒ λ1 = λ2 = 4λ3 = 9λ 4
Power = 663 mW
663 ×10−3 27 × 663
Sol 27: (C) (i) Frants – Hertz Experiment is associated No. of photon per sec = = ×1016
6.2 −19 62 ×1.6
with Discrete energy levels of atom ×1.6 ×10
27
(ii) Photo electric experiment is associated with particle
27 × 663 ×1016
nature of light. No. of it e¯ per sec =
62 ×1.6 ×5 ×109
(iii) Davison – Germer experiment is associated with
wave nature of electron. = 3.61 × 108 e¯/sec =3.61 × 108 × 1.6 × 10–19 A
= 5.776 × 10–11 A
hc 1
Sol 28: (D) = w + mv 2  … (i)
λ 2
Sol 3: λ = 330 nm
hc 1 hn1 = hn0 + KE
= w + m(v ')2
λ' 2
KE = eV0
hc 1 hn1 = hn0 + eV0
= w + m(v ')2  … (ii)
 3λ  2
  hn2 = hn0 + 2eV0
 4
h(n2 – n1) = eV0
 4
Equation (i) ×  – (ii)
 3  h(ν2 − ν1 ) E2 − E1
V0 = =
4hc 4 hc 4 41  1 e e
– = w +  mv 2  – w – m(v')2
3λ 3 λ 3 32  2
2 4 . 7 6 | Modern Physics

12400 12400 Sol 6: φ = 1.17 eV


E2 = eV ; E1 = eV
2200 3300 d = 2m

62 41.3 λ = 4.8 × 10-7 m = 4800 Å


E2 = eV ; E1 = eV
11 11 P = 1W
p 1 1
62 − 41.3 Intensity of light = = =
V0 = eV = 1.88 V 4 πd 2 4π × 4 16 π
11e
12400
Energy of 1 photon = = 2.58 eV
4800
Sol 4: λ = 990 Å
d = 0.1 m Number of photons striking per square meter per sec

r = 0.05 nm = 5 × 10–11 m 1
= = 4.81 × 1016
10 −11 2 16π × 2.58 ×1.6 ×10−19
(i) Intensity of light = × π (5 × 10 )
4 π (0.1)2 hν = hn0 + KE
= 250 × 25 × 10–22
E = hν
= 6250 × 10–22
E = hn0
= 6.25 × 10–19 J
1
12400 mv 2 = hν – hn0 = E – E0
Energy of photon = = 12.52 eV 2
990 12400
− 1.17
  = 20 × 10–19 J 4800

6.25 ×10−19 1
5 m v 2 = 2.58 – 1.17 = 1.41 eV …. (i)
Average no. of photon = = 2 e
20 ×10−19 16
2.82 ×1.6 ×10−19
v2 =
10 1 1 9.1 ×10−31
(ii) No. of electron = × ×
4 π (0.1)2 −19
(20 × 10 J) 100 v2 = 0.495 × 1012

10 ×1018 5 1019 v = 7.04 × 105 m/s


= = ×1018 =
4 π× 2 4π 8π mv 2
Magnetic force = eVB =
R
Sol 5: hν = hn0 + KE By (i)

KE = eV0 mv2 = 2.82 eV

hν = hn0 + eV0 2.82eV


eVB =
R
eV0 = hν – hn0
2.82
y =V0 ; x = n 7.04 × 105 m/s × 10–4 =
R
ey = hx – hn0 R = 4 cm
hx φ
y
= −
e e Sol 7: Z = 3
Work function φ = 2eV 1 1 
Energy of E1 = 13.6 × 9  − 
h 2  16 25 
Slope = =
e 0.49 ×1015
E1 = 2.754 eV
−19
2 ×1.6 ×10
h= = 6.53 × 10–34 J-s 1 1 
0.49 ×10 15 Energy E2 = 13.6 × 9  − 
 9 16 
P hysi cs | 24.77

E2 = 5.95 eV Sol 10: P = 3.2 × 10–3 W


E2 = φ + KE2 (a) Energy of photons = 5 eV;
KE2 = 3.95 eV 12400
∴λ = = 2480 Å
φ = E2 – KE2 5
Distance = 0.8 m
= 5.95 – 3.95
φ = 3 eV
φ = 2 eV
Radius = 8 × 10–3 m
E1 = φ + KE1
1
KE1 = 2.754 – 2 Efficiency = electrons per photon
106
eV = 0.754 eV Power incident on atom
V = 0.754 Volts
3.2 ×10−3
= × π (8 ×10−3 )2
12400 4 π (0.8)2
Sol 8: l1 = 4144 Å; E1 = eV = 2.99 eV
4144
10−3
12400 = × 0.8 × 0.8 ×10−4
l2 = 4972 Å; E2 = eV = 2.49 eV 0.8
4972

12400 n=5
l3 = 6216 Å; E3 = eV = 1.99 eV E1
6216 n=4
E2
φ = 2.3 eV n=3
E2 > E1
−3
3.6 ×10
Intensity I1 = I2 = I3 = Wm–2 × 10–4
3 = 0.8 × 10–7 = 8 × 10–8 W
= 1.2 × 10–7 W
8 ×10−8
No electrons will be emitted by 6216 Å wavelength No. of photons =
5 ×1.6 ×10−19
photons as E3 < φ.
N = No. of electrons
No. of photons in light of wavelength l2 is
1.2 ×10−7
= 3 × 1011 photons / sec 8 ×10−8
2.49 ×1.6 ×10 −19 = = 105 s−1
106 × 5 ×1.6 ×10−19
No of photons in light wavelength l1 is
h
(b) ld =
1.2 ×10 −7 mv
= 2.5 × 1011 photons / sec
2.99 ×1.6 ×10−19 p2
KE = 2eV =
2m
No of electrons liberated in 2 seconds
p2 = 2 × 1.6 × 10–19 × 2 × 9.1 × 10–31
= 2 (3 + 2.5) × 10 11
p2 = 58.24 × 10–50
= 11 × 10 electrons.
11
p = 7.63 × 10–25

Sol 9: (i) Refer Sol 4 Exercise-I JEE Advanced 6.6 ×10−34


ld = = 0.86 ×10−9
−25
5 7.63 ×10
(ii) No. of photons = per second
16
λ 2480 ×10−10 248
5 1 = = = 286.76
No. of electrons = × per second λd 0.86 ×10 −9 0.86
16 100
5 (c) After some time sphere gets positively charged and
= per second
1600 it will create electric field which will stop the further
emission of electrons.
2 4 . 7 8 | Modern Physics

(d) KE = hν – hφ =5 eV – 3 eV = 2 eV EB = φB + Tb
K.E. = eV = 2 eV φB = 4.7 – 0.5 = 4.2 eV
V = 2 volts
Sol 12: Force on electron = eE
Potential at the surface of sphere
eE
Kq K(Nt) ×1.6 ×10−19 9 ×109 × 105 t ×1.6 ×10−19 Acceleration =
= = = m
r 8 × 10−3 8 × 10−3 eE
Velocity = t ; p = eEt
= 9 × 0.2 × 10–2 × t m
h
2 = 1.8 × 10–2 × t ld =
p
2
t
So time required= ×100 = 111.1 sec dλ d h dp −heE −heE −h
1.8 =– = = =
dt p 2 dt (eEt)2 2 2 2
eE t eEt2
Sol 11: For metal A
13.6
Energy of photons = 4.25 eV Sol 13: B.E. = 0.85 eV =
n2
Maximum KEA = Ta
n=4
de Broglie wavelength = la
For metal B 1  1 1  1 1 
= R − = R − 
2 
λ 2 2
4   4 16 
KEB max = Tb = Ta – 1.5
Energy of photons = 4.7 eV 1 12R 3R 16R
= = ⇒ λ= = 487 nm
De-Broglie wavelength = lb = 2la λ 4 ×16 16 3
2
p2  h  1
KE =
=   × Sol 14: 5th excited state
2m  λd  2m

 h 
2 ⇒n=6
1
=Ta   ×
 λ  2m m = mass of atom
 a
2 v = velocity of atom
 h  1
=Tb  ×
 2λ  2m (mv 2 ) hc 13.6
 a + = E6 − E0 = – + 13.6
2m λ 36
Tb = Ta – 1.5 35
= ×13.6
2 36
 h  1 h2 1 h
  × − × −1.5
= Momentum conservation, = mv
 2λa  2m λa 2m
2
λ
2

 h 
2

  ×
1 3
× =−1.5
1  h  hc
  +
2m  λ  λ
=
35
36
×13.6 × 10−19 ×1.6 J ( )
λ
 a 2m 4
⇒ λ = 939.4 Å
Ta ÷ 2 = 1
Energy = 13.2 eV
Ta = 2 eV
h
Tb = 2 – 1.5 = 0.5 eV v= ⇒ v = 4.26 m/s

From metal A
13.6
Sol 15: Energy of series limit of Balmer is = 3.4
EA = φA + Ta 4
φA = 4.25 – 2 = 2.25 eV hc
λB =
For metal B 3.4
P hysi cs | 24.79

1 1  h 6.6 ×10−34
Energy of first line of Paschen is 13.6  2 − 2  λ1 = = = 8.6 pm
3 4  mv1cm 1.67 × 10−27 × 4.6 ×10 4
hc 13.6 × 7
= = 0.661 h 6.6 ×10−34
λp 9 ×16 l2= = =8.6 pm
2mv 2cm 2 × 1.67 ×10−27 × 2.3 ×10−4
hc
λP =
0.661 nλ
Sol 17: = 2Å
2
λB 0.661 7
Ration = =
λP 3.4 36 (n + 1)λ λ
= 2.5 Å ⇒ = 0.5 Å
2 2
1 h
Sol 16: 25 eV = m v2 ⇒ λ = 1Å ⇒ λ =
2 n p
6.6 ×10−34
V p= = 6.6 × 10–24
−10
m 2m 1 ×10

p2 (6.6)2 ×10−48 2.39 ×10−17


Energy = = = = 148.4eV
2m 2 × 9.1 ×10−31 1.61 ×10−19
50 ×1.6 ×10−19
v2 = = 47.9 × 108
1.67 ×10−27 Sol 18: (D) Energy of photon from He+
V = 6.92 × 104 m/s
1 1
= 13.6 Z2  − 
m1 v1 + m2 v 2 m× V + 0 V 1
2
22 
Vcm = = =
m1 + m2 3m 3
3
= 13.6 × 4 × = 13.6 × 3 eV
= 2.3 × 10 m/s 4
4
By momentum conservation Energy of photon from H = 13.6 eV
Energy of photoelectron = 13.6×3-13.6 = 13.6×2 eV
V1 V2 1
m 2m K.E. = mv2 = 13.6 × 2 × 1.6 × 10–19
2

mV = mV1 + 2mV2 4 ×13.6 ×1.6 ×10−19


v2 = ⇒ v2 = 9.56 × 1012
−31
9.1 ×10
V = V1 + 2V2
By energy conservation v = 3.09 × 106 m/s

1 1 1 n(n − 1)
=mV 2 mV12 + 2mV22 Sol 19: (i) =6
2 2 2 2

V2 = V12 + 2V22 ⇒ V2 = (V – 2V2)2 + 2V22 ⇒ n (n – 1) = 12 ⇒ n = 4


V2 = V2 + 6V22 – 4V2V ⇒ 6V2 = 4V since emitted photons are of energy less, equal and
2V more than 2.7 eV
V2 =
3 So level B must be n = 2
4V V
V1 = V – 2V2 = V – =– 1 1 
3 3 (ii) 2.7 = 13.6 Z2  − 
 22 42 
−2V
Velocity of 1 w.r.t. C.M. is V1cm = = – 4.6 × 104 m/s
3 13.6 Z 2 3
V 2.7 =  
Velocity of 2 w.r.t. CM is V2cm = =2.3 × 104 m/s 4 4
3
2 4 . 8 0 | Modern Physics

2.7 ×16 13.6 Z 2 ×5


Ionisation energy = 14.4 eV = 14.4×1.6×10–19 J 47.2 × 2 =
3 36
= 23.04 × 10–19 J Z2 = 25 ⇒ Z = 5

1 1  1 1
(iii) Maximum energy= 13.6 Z2  −  ; 4 → 1 Sol 26: 47.2 = 13.6 Z2  − 
2
 12 42  2 32 

(i) Z = 5
1 1 
Minimum energy = 13.6 Z2  −  ; 4 → 3
 32 42  1 1 
(ii) E = 13.6 × 25  −  = 16.5 eV
 9 16 
Sol 20: λ = 500 Å
1 1 
12400 (iii) E = 13.6 × 25  − 
Energy = = 24.8eV  1 ∞2 
2
500
Energy required to take out electron from atom = 13.6 E = 13.6 × 25 eV
eV E = 340 eV
KE = 24.8 – 13.6 = 11.2 eV 12400
λ= = 36.4 Å
340
Sol 21: Energy of photon = 13.6 Z2 (iv) KE = |T.E.|
= 13.6 × 4 = 54.4 KE = 13.6 Z2 = 13.6 × 25 = 340 eV
12400 eV P.E. = – 2 |T.E.|
Wavelength = = 227.94 Å = 22.8 nm
54.4 eV
P.E. = – 2 × 340 = – 680 eV
h
Sol 22: E3 – E2 + E2 – E1 = E3 – E1 (v) Angular momentum = mvr = Iω = n

12400 12400 12400 0.529n2
+ = Radius = = 0.1058 Å
λ1 λ2 λ3 z

1 1 1 λ λ
+ = ⇒ λ3 = 1 2 Sol 27:
λ1 λ2 λ3 λ1 + λ2
n n
n1 n2
Sol 23: Energy of new atom = 2 × energy of hydrogen
n=2 n=3
atom
Energy gap between quantum states n and 2 is
hc 1 1
=13.6 × 2  −  22.95 + 5.15 = 28.1 eV
λ  22 32 
Energy gap between quantum state n and 3 is
1 1 1 18
= 2R  −  ⇒ λ = 2.4 + 8.7 = 11.1 eV
λ 2 2
2 3  5R
Energy gap between n = 2 and n = 3 is

Sol 24: r = 0.5 Å ; ω = 2π × 1016 rad / sec. 28.1 – 11.1 = 17 eV


1 1 13.6 Z 2 ×5
qVr 17 = 13.6 Z2  −  =
Magnetic moment = 2 2 36
2 2 3 
Z2 = 9 ⇒Z=3
eωr 2 1.6 ×10−19 × 2π×1016 1
= = × ×10 −20 = 0.4 × 10–23 1 1
2 2 4 28.1 = 13.6 × 9  − 
 22 n2 
= 1.25 × 10–23 Am2 1 1 1
− = 0.229 ⇒ = 0.25 − 0.229
1 4 n 2
n2
1
Sol 25: 47.2 = 13.6 Z2  − 
22
32  ⇒ n2 = 48.96 ⇒ n = 7
P hysi cs | 24.81

1 1 Exercise 2
Sol 28: E = 13.6 Z2  2 − 2 
2 5 
Single Correct Choice Type
12400 1 1
= 13.6 Z 2  −  10−3
1085  22 52  Sol 1: (B) No. of photons =
Energy of 1photon
12400  21  12400
= 13.6 Z 2   Energy of 1 photon = = 2.48 eV
1085  100  5000

⇒ Z2 = 4 ⇒ Z = 2 10−3
np = No. of photons =
Binding energy = 13.6 Z2 = 13.6 × 4 = 54.4 eV 2.48 ×1.6 ×10−19

−13.6 × 32 = 0.25 × 1016 = 2.5 × 1015 photon


Sol 29: Energy of n state of lithium ion =
th

n2 0.16 µA 1.6 ×10−7


= – 13.6 eV (n = 3) ne = No. of electron = =
1.6 ×10−19 1.6 ×10−19
0.529n2
Radius = = 1012 electron
2
ne 1012
0.529 × 9 Efficiency = ×100 = × 100
R 2+ np 2.5 ×1015
=Li
= 3 3 4
1
RH 0.529 = = = 0.04%
2.5 ×10 100

Sol 30: Total energy of the photons is 20 KeV


Sol 2: (B) KEmax = 2eV
Let their wavelength be λ 1, l2 Å
E1 = φ + KEmax 1
12400 12400
+ 20000
=
λ1 λ2 5eV = φ + 2eV
1 1 φ = 3eV
+ 1.613 
= … (i)
λ1 λ2 E2 = φ + KEmax 2 ; KEmax 2 = eV2
6 eV = 3 eV + eV2 ⇒ 3 eV = eV2
λ2 − λ1 =1.3  … (ii)
V2 = 3V
By (i) and (ii)
So stopping potential of A w.r.t. C is – 3V
1 1
+ = 1.613 2π2mk 2 Z 2e4
λ1 1.3 + λ1 Sol 3: (A) En =
n2h2
1.3 + 2λ1
= 1.613 Z 2 . 4 π2 mk 2e4
λ1 (1.3 + λ1 ) Frequency ν =
n3 .h3
1.3 + 2l1 = 1.613 × 1.3 l1 + 1.613 l12
1.6l12 + 0.096 l1 – 1.3 = 0 ν Z 2 × 4 π2 mk 2e4 n2h2 ν 2
= ⇒ =
En 2 3 2
n h 2π mk Z e 2 2 4 En nh
−0.096 ± (0.096)2 + 4 × 1.3 × 1.6
λ1 = 2En
3.2 ⇒ ν=
nh
l1 = 0.871 Å
l2 = 2.17 Å n2
Sol 4: (D) Bohr radius = × 0.529
Z

rn+1 – rn = [(n + 1)2 – n2] 0.529 = rn–1


2 4 . 8 2 | Modern Physics

= (2n + 1) 0.529 = (n – 1)2 × 0.529 field as in magnetic field kinetic energy does not
change. Kinetic energy of electron in electric field may
⇒ (n – 1)2 = 2n + 1
increase, remain same or decrease that’s why l2 can
n2 + 1 – 2n = 2n + 1 ⇒ n2 = 4n increase or decrease.
⇒n=4
λ1 > λ2 or λ2 < λ1 or λ1 = λ2
2 2 2
n h e0 0.529n
Sol 5:=
(A) rn =
pme2 Z Z Sol 11: (A, C) Minimum energy required for transition
to happen from ground state is 10.2 eV.
n =1, Z = 1
If the total loss in energy is less than 10.2 eV no transition
0.529n2 o will occur. Either there can be loss of energy greater
For mean r’n = = 2.56 × 10–3 A than 10.2 eV or no loss in energy since the energy of
207 Z
neutron is 20.4 eV the maximum loss in energy due
to inelasticity will be less then 10.2 eV. Therefore only
Sol 6: (C) Hydrogen emit a photon corresponding to
option is no loss in energy which means elastic collision.
the largest wavelength of the Balmer series. This implies
So (A and C).
electron was excited to n = 3
Energy required for transition n = 1 → 3 is
Sol 12: (A, C, D) Photon of energy 2.55 eV is emitted
 1 1  13.6 × 8 when transition is from n = 4 to n = 2
13.6  −  = = 12.088 eV
2 2
1 3  9 So other photon corresponds to n = 2 → n = 1

Minimum kinetic energy = 12.1 eV Energy absorbed by hydrogen atom = 10.2 + 2.55 =
12.75 eV

Sol 7: (A) Refer Q-11 (in Exercise II JEE Advanced) Minimum Kinetic energy of photon is when
collision is perfectly inelastic i.e. when
1 1  5 K = 25.5 eV
Sol 8: (B) n = 3 → 2; E1 = 13.6  −  = ×13.6
 4 9  36
Refer Q. 11
 1 3
n = 2→ 1; E2 = 13.6 1 −  = ×13.6
 4  4 Sol 13: (A, C, D) 13.6 Z2 = 122.4

hc hc Z=3
λ1 = ; λ2 =
E1 E2 For n = 1, E1 = – 122.4 eV
λ1 E2 3 27 n = 2 E2 = – 30.6 eV
x= = = × 36 =
λ2 E1 4 ×5 5 E2 – E1 = 91.8 eV
E1 5 If 125 eV energy electron collides with this atom then
Z
= =
E2 27 122.4 eV will be used to take out the electron and
kinetic energy of electron will be 125 – 122.4 = 2.6 eV
p1 h / λ1 λ2 5
y
= = = =
p2 h / λ2 λ1 27
Sol 14: (A, C, D) Some incident wavelengths will be
absent in A as some of them will be absorbed by
Multiple Correct Choice Type the hydrogen atom. B will emit photons of Energy
Corresponding to transitions in the hydrogen atom.
Sol 9: (A, C) Stopping potential ∝ kinetic energy This energy will lie in visible and infrared region.

Kinetic energy depends on the frequency of light


Sol 15: (A, B, C) Having electrons of same speed won’t
hν = hn0 + KE matter because electrons get decelerated to different
velocities ( just like electrons with random velocities)
h giving photons of different wavelength. (Read theory).
Sol 10: (A, D) λd =
p

de Broglie wavelength l1 will not change in magnetic


P hysi cs | 24.83

Assertion Reasoning Type


= 0.0364 × 10−7 m = 36.4 Å

Sol 16: (D) For frequency less than no no electrons are (d) In first orbit, total energy = –340eV
emitted. so Statement-I is/false. Kinetic energy = +340 eV

h Potential energy = –2 × 340 eV = –680 eV


Sol 17: (C) Momentum of photon is p =
λ h
and angular momentum =
Sol 18: (D) All emitted electrons do not have same K.E. 2π

There K.E. range from 0 to (hν – φ). 6.6 × 10−34


= = 1.05 × 10−34 kg- m2 /s

Sol 19: (D) If electron will not emit as only one single n2
photon should have energy more than work function. (e) rn ∝
Z
h h Radius of first Bohr orbit
Sol 20: (A) λe = ; λp =
2me (KE) 2mp (KE)
r1H 5.3 × 10−11
mp > me r1 = = = 1.06 × 10−11 m
Z 5
⇒ le > lp
Sol 2: Energy corresponding to given wavelengths:
Sol 21: (A) By Boltzmann’s law (randomization increases
12375 12375
with temperature) electron’s occupy more number of E(in eV) = = = 12.69eV
excited levels at higher temperature. λ (in Å) 975
Now, let the electron excites to nth energy state. Then,
En − E1 =12.69
Previous Years’ Questions
( −13.6)
or – (–13.6) = 12.69
(n2 )
Sol 1: (a) Given E3 − E2 = 47.2 eV
∴n≈ 4
Z2
Since En ∝ (for hydrogen like atoms) i.e., electron excites to 4th energy state. Total number of
n2
lines in emission spectrum would be:
 Z2    Z2  n(n − 1) 4×3
or(–13.6)   –  −(13.6)    =47.2 = =6
 9    4  2 2
    
Longest wavelength will correspond to the minimum
Solving this equation, we get Z = 5
energy and minimum energy is released in transition
(b) Energy required to excite the electron from 3rd to from n = 4 to n = 3.
4th orbit: −13.6 −13.6
E4 −=
3 E4 − E3 = – =0.66 eV
E3−=
4 E4 − E3 (4)2
(3)2

 25    25   ∴ Longest wavelength will be,


= (–13.6)   – ( −13.6)    = 16.53eV
16
    9 
12375 12375
λmax = = Å= 1.875 × 10−6 m = 1.875 µm
(c) Energy required to remove the electron from first E(in eV) 0.66
orbit to infinity (or the ionization energy) will be:
E = (13.6) (5)2 = 340 eV Sol 3: Number of proton = atomic number = 11

The corresponding wavelength would be, Number of neutron = mass number – atomic number
= 13
hc 6.6 × 10−34 × 3 × 108 But note that in the nucleus number of electron will be
λ= =
E 340 × 1.6 × 10−19 zero.
2 4 . 8 4 | Modern Physics

photon required is,


238 α−decay β−decay
Sol 4: 92 U → 90 X 234 →91 Y 234
( −13.6)(3)2  ( −13.6)(3)2 
E1−3 = E3 − E1 = –  
During an α -decay atomic number decreases by 2 and (3)2  (1)2 
mass number by 4. During a β -decay, atomic number
increases by 1 while mass number remains unchanged. = 108.8 eV
Corresponding wavelength will be,
Sol 5: When 800 Å wavelength falls on hydrogen atom
(in ground state) 13.6 eV energy is used in liberating 12375 12375
λ (in Å)= =
the electron. The rest goes to kinetic energy of electron. E(in eV) 108.8
Hence, K = E – 13.6 (in eV) or = 113.74 Å
hc (b) From nth orbit total number of emission lines can be ,
(1.8×1.6× 10−19 )= –13.6×1.6× 10 −19 ….(i)
−10
800 × 10 n(n − 1)
.
Similarly for the second wavelength: 2
3(3 − 1)
∴ Number of emission lines = =3
hc 2
(4.0×1.6× 10−19 )= –13.6×1.6× 10 −19 …(ii)
700 × 10−10
2K
Sol 8: Speed of neutrons =
Solving these two equations, we get m

h ≈ 6.6 × 10 −34 J s− s  1 2
 From K = mv 
 2 
Sol 6: (a) 1 Rydberg = 2.2 × 10−18 J = Rhc
2 × 0.0327 × 1.6 × 10−19
Ionisation energy is given as 4 Rydberg or v = ≈ 2.5 × 103 m / s
1.675 × 10−27
8.8 × 10−18 Time taken by the neutrons to travel a distance of 10 m:
= 8.8 × 10−18 J = = 55 eV
1.6 × 10−19
d 10
t= = = 4.0 × 10−3
∴ Energy in first orbit E1 = −55eV v 2.5 × 103

Energy of radiation emitted when electron jumps from Number of neutrons decayed after time t
first excited state (n = 2) to ground state (n = 1):
N = N0 (1 − e−λt )
E 3
E21 =1 − E1 =− E1 =41.25eV
(2)2 4 ∴ Fraction of neutrons that will decay in this time
interval
∴ Wavelength of photon emitted in this transition
ln(2)
would be, N − × 4.0 × 10−3
= = (1 − e−λt ) = 1 – e 700 = 3.96 × 10−6
12375 N0
λ= = 300 Å
41.25
(b) Let Z be the atomic number of given element. Then Sol 9: If we assume that mass of nucleus >> mass of
mu-meson, then nucleus will be assumed to be at rest,
( 13.6)(Z 2 ) or Z ≈ 2
E1 = ( −13.6)(Z 2 ) or −55 =− only mu-meson is revolving round it.
1 (a) In nth orbit, the necessary
Now, as r ∝
Z centripetal force to the mu-
Radius of first orbit of this atom, meson will be provided by the
rH electrostatic force between the
0.529 +Ze - m = 208 me
r1 = 1 = = 0.2645 Å nucleus and the mu-meson.
Z 2

Z2
Sol 7: Given Z = 3 : En ∝
n2
(a) To excite the atom from n = 1 to n = 3, energy of
P hysi cs | 24.85

mv 2 1 (Ze)(e) Sol 10: (C)


Hence, =  … (i)
r 4 πε0 r 2 n=2 -3.4eV
E = 10.2eV
Further, it is given that Bohr model is applicable to this n=1 -13.6eV
system also. Hence, H-atom
n=4 -3.4eV
nh
Angular momentum in n orbit = th
2π n=3 -6.04eV
h E = 10.2eV
or mvr = n  … (ii) n=2 -13.4eV

We have two unknowns v and r (in nth orbit). After -54.4eV
n=1
n2h2 ε0 He+
solving these two equations, we get r = Z=2
Zπme2
Substituting Z = 3 and m = 208 me , we get Energy given by H-atom in transition from n = 2 to n
= 1 is equal to energy taken by He+ atom in transition
n2h2 ε0 from n = 2 to n = 4.
rn =
624 πmee2

(b) The radius of the first Bohr orbit for the hydrogen Sol 11: (C) Visible light lies in the range, λ1 = 4000 Å to
λ2 = 7000 Å. Energy of photons corresponding to these
h2 ε0
atom is: wavelength (in eV) would be:
πmee2
12375
E1 =
Equating this with the radius calculated in part (a), we 4000 = 3.09 eV
n2 ≈ 624 or n ≈ 25
12375
mv 2 Ze2 E2 =
(c) Kinetic energy of atom = = 7000 = 1.77 eV
2 8πε0r
From energy level diagram of He+ atom we can see that
Ze2 in transition from n = 4 to n = 3, energy of photon
and the potential energy = −
4 πε0r released will lie between E1 and E2.
−Ze2
∴ Total energy En = ∆E43 =−3.4 − ( −6.04) = 2.64 eV
8πε0r

Substituting value of r, calculate in part (a), Wavelength of photon corresponding to this energy,
12375
1872  mee 
4
λ=
264 Å = 4687.5 Å = 4.68×10 m
-7
=En − 
n2  8ε02h2 

 m e4  Sol 12: (A) Kinetic energy K ∝ Z 2


But  − e  is the ground state energy of hydrogen
 8ε20h2  2
KH 1 1
atom and hence is equal to – 13.6 eV. ∴ = =
K + 2 4
He
−1872 25459.2
∴ En = (13.6) eV = – eV
2
n n2 Sol 13: A → p, q; B → p, r; C → p, s; D → q

1 1 
∴ E3 − E1 =−25459.2  −  =22630.4 eV Sol 14: (C, D) For photoemission to take place,
9 1
wavelength of incident light should be less than the
∴ The corresponding wavelength, threshold wavelength. Wavelength of ultraviolet light <
5200Å while that of infrared radiation > 5200 Å.
 12375 
λ(ln)=
A = 0.546 A
22630.4 nλ 2a
Sol 15: (A) a = ⇒λ=
2 n
2 4 . 8 6 | Modern Physics

h h
λdeBroglie = λ=
p mv

2a h nh 4.2 × 10−15 × 1.6 × 10−19 4.2 × 4.8 × 10−34


= ⇒p= =
λ = = 0.07 × 10−13
n p 2a 5
× 10−27 × 331.776 × 1013 57.6 × 5 × 10−21
3
p2 n2h2 λ = 7 × 10 −15 = 7 fm
E= =
2m 8a2m
Sol 21: (B) t = 100 × 10-9 sec, P = 30 × 10-3
⇒ E ∝ 1 / a2
Watt, C = C × 108 m/s

Sol 16: (B) Pt 30×10−3×100×10−9


Momentum
= = = 1.0 × 10−17 kg ms−1
h2 (6.6 × 10−34 )2 C 3 × 108
E= = = 8 meV.
8a2m 8 × (6.6 × 10−9 )2 × 10−30 × 1.6 × 10−19
Sol 22: Slope of graph is h/e = constant ⇒ 1
nh
Sol 17: (D) mv =
2a Sol 23: Ephoton= Eionize atom + Ekinetic energy
nh 1242 13.6
v= ⇒ v ∝n
= + 10.4
2am 90 n2
from this, n = 2
Sol 18: P1 = 2m(100 eV)

h h Sol 24: (A) For photoelectric emission


λP = ⇒ λα =
2m(100 eV) 2(4m)(100 eV)
 hc  1 φ
V0 =   –
λP  e λ e
= 8
λα
hc
λP Sol 25:(B) KEmax
= – φ eV0
=
⇒ The ratio to the nearest integer, is equal to 3. λ
λα
hc hc
⇒ – = e(V1 – V2 )
1  1 1  5R λ1 λ2
Sol 19: (A) =R –  =
6561  4 9  36
 1 1 
⇒ hc  – =1.6 × 10 –19 × 10 –6
1  1 1  3R × 4  0.3 0.4 
= 4R  – =
λ  4 16  16
 0.1  –25
 ⇒ hc  =  1.6 × 10
λ =1215 A  0.12 
1 K(Q)e K(120 e)e
2
Sol 20: 0 + mv= = 1.6 × 10−25 × 1.2
2 10 × 10 −15
10 × 10−15 ⇒ h= =0.64 × 10−33 =6.4 × 10−34
3 × 108

1 5 9 × 109 × 120 × (1.6 × 10−19 )2  1 1 


× × 10−27 v 2 = ⇒ hc  – −19 −6
2 3 10 × 10−15  = (1.6 × 10 ) × 0.6 × 10
 0.4 0.5 
9 × 6 × 109 × 120 × 2.56 × 10−38 0.20 1
⇒ v= ⇒ hc = (0.96 × 10−25 ) × ×
50 × 10−42 0.10 3 × 108

⇒ v
= 331.776 × 1013 1.92
⇒ h = × 10−33 =6.4 × 10−34
3
P hysi cs | 24.87

Sol 26: (A), (B), (D) Orbital radius rn = n2c [c = constant] m2d m1d
r1 = and r2 =
Angular momentum = nh = L m1 + m2 m1 + m2

∆r (n+ 1)2 – n2 2 ∆Ln 1 I = m1 r12 + m2 r22


= = .....[B]; = .... [D]
rn n2 n Ln n
∴ d = 1.3 ×10−10 m.
(A) is correct since it will get cancelled in calculation of
relative charge. hc
Sol 33: (7) Stopping potential = –W
λ
Sol 27: [6] Photon Energy = 6.2 eV −4.7 eV = 1.5 eV

hc 1.237 × 10 –6 1237 Kq
== = × 10eV V
= = 1.5
λ 970 × 10 –10 970 r

Absorption of this photon changes the energy to = – = 1.5 × 10−2


n = 1.05 × 107
13.6 + 12.75 = – 0.85 eV 9 × 10 × 1.6 × 10−19
9

Number of possible transitions from the 4th quantum Z=7


state = 4C2 = 6
−2GMm 1
Sol 34: (B) + mv 2 =
0
Sol 28: (B) P1 = pressure just inside the bubble at the L 2
4T
end 2 = P0 + GM
R ⇒v=
2
L
P2 = pressure just inside the bubble at the end
Note: The energy of mass ‘m’ means its kinetic energy
4T
1 = P0 + (KE) only and not the potential energy of interaction
r
between m and the two bodies (of mass M each) –
R > r ⇒P2< P1 ⇒ Air will flow from end 1 to end 2 which is the potential energy of the system.

Sol 29: (A) VB= (1/e)[(hc/λ) − φ ] hc


–φ
VP= (1/e)[(1240/550) −2]eV = 0.2545 V λ1 u12
Sol 35: (A) =
hc u22
Vq= (1/e)[(1240/450) −2.5]eV = 0.255 V –φ
λ2
Vr = (1/e)[(1240/350) −3]eV = 0.5428 V φ = 3.7 eV
If n is the number of photons in unit time then nhc/λ= I
hc
⇒iP : iq: ir= nP : nq : nr = λP : λq : λr Sol 36: (A) K=
max – φ + eV [Kmax = maximum
λph
energy e– reaching the anode]
nh
Sol 30: (D) L =  hc 
2π h2
⇒ =  – φ  + eV  … (i)
2 2mλ 2e λ 
L2  nh  1  ph 
K.E. = = 
2I  2π  2I From Equation (i) (A) follows
 hc 
if φ increases and λph increases then  – φ
 λph 
Sol 31: (B) hv= k.En=2 −kEn=1 decreases  
As a result λc increases λe is independent of ‘d’ and
I = 1.87 × 10-46 kg m2 clearly λe and λph do not increase at the same rate.

Sol 32: (C)

You might also like